You are on page 1of 218

OBS

GYN-System Wise 1700-by Sush and Team. 2016


Susmita, Asad, Manu, Saima, Zohaib, Savia, Shanu, Mona, Manisha, Sitara, Samreena, Sami and Komal


Dear Plabber,


• This first ever System Wise 1700 document was created thanks to 3 months of daily hard work by the
PLAB Skype group ‘Unity’ which was brought together by Dr Susmita Chowdhury.
• Please ignore the old versions posted by my new skype member Murtaza as he did so without
permission.

The team members were:


& Susmita (Lead/most ignorant as she is working full time in public health for 13 years)

& Asad (Invaluable in IT and all types of support/the heart of the group)
& Manu (Volunteered to solve more questions/pathologist/amazing genuine person)

& Saima (Most concise clear notes/ photographic memory)
& Zohaib (Great research/a surgeon)
& Savia (Great research/multi-tasker with two little ones)
& Shanu (Very helpful after her March exam for those appearing in June)
& Mona (Great contributor in discussions)
& Manisha (Gyne/great discussion contributor)
& Sitara (Good discussion contributor)
& Samreena (Stayed a shorter time but great)
& Sami (Contributed the most early on but too brilliant for the group/still great friends)

& Komal (Knowledgeable sweet supportive girl)



• The main purpose was to break down the 1700 Q Bank System wise.



• We did our own reliable research for the options (OHCM/Patient info etc.) and concluded these keys
below on skype. This can save you 100s of hours of research. But I suggest you also do your own.

• 90% of the document consists of Unity research. We also added information from other circulating
documents and they are referenced as Dr Khalid/Dr Rabia (and her Team).

• However, several keys may be ‘incorrect’ and so please use your own judgment as we take no
responsibility. I suggest cross checking with Dr Khalid’s latest keys (a few of which are still debatable).
Finally decide on your own key.

• Sorry if some members failed to make their answers thorough. The highlights are mostly as per what
the team members wanted to highlight. Blank tables to be ignored.

• Note that some 1700 Questions are missing from here (when members did not do their share).
Questions may not be in order due to merging of documents and there is excess information than
required. Read as much as needed.

• This has been circulated by our team as a generous contribution to the Plabbers’ success and must
not be ‘sold’.



Good luck and best wishes: Sush and Team


1
OBS GYN-System Wise 1700-by Sush and Team. 2016
Susmita, Asad, Manu, Saima, Zohaib, Savia, Shanu, Mona, Manisha, Sitara, Samreena, Sami and Komal


Q: 287 A pregnant woman with longterm hx of osteoarthritis came to the antenatal
clinic with complaints of restricted joint movement and severe pain in her
affected joints. What is the drug of choice?
a. Paracetamol
b. Steroid
c. NSAID
d. Paracetamol+dihydrocoiene
e. Pethadine



Clincher(s)
A · Preconception and pregnancy

• Paracetamol is the analgesic of choice for women who are trying to


conceive, or who are pregnant.
• Paracetamol can be used at the usual dose at any stage of pregnancy


B Corticosteroids are generally safe to use during pregnancy. However, they're
not usually recommended unless the potential benefits outweigh the risks.
C · Paracetamol is the analgesic of choice during pregnancy.
· If paracetamol is ineffective and an NSAID is clinically indicated during the
first or second trimester of pregnancy:

• Ibuprofen is preferred, but should only be taken up to 30 weeks of


gestation (this use is off-label).
• Use the lowest effective dose, for the shortest duration possible.
• If regular use is required (for example if the woman has rheumatoid
arthritis), seek specialist advice.

· NSAIDs must not be used from week 30 of pregnancy onwards, without


specialist advice and regular fetal monitoring.
D · Codeine is preferred if a weak opioid is required.

• Codeine can be used short term in all trimesters of pregnancy.


However, use of codeine near to the end of gestation may cause
neonatal respiratory depression. Long-term use may cause withdrawal
symptoms in the baby.
• Information on the safety of dihydrocodeine and tramadol in
pregnancy is lacking.


E
KEY A
Additional Rheumatoid arthritis is usually alleviated by pregnancy (but exacerbations

2
OBS GYN-System Wise 1700-by Sush and Team. 2016
Susmita, Asad, Manu, Saima, Zohaib, Savia, Shanu, Mona, Manisha, Sitara, Samreena, Sami and Komal

Information may occur in the puerperium). Methotrexate use is contraindicated
(teratogenic);
sulfasalazine may be used (give extra folate). Azathioprine use may
cause intrauterine growth restriction and penicillamine may weaken fetal
collagen. Non-steroidal anti-infl ammatories can be used in the fi rst and
second trimesters but are not recommended in the third as they can cause
premature closure of the ductus arteriosus and late in pregnancy have been
associated with renal impairment in the newborn. Anti-tumour necrosis factor
TNF-alpha therapies have not shown problems; but experience is limited. 87
Congenital heart block is a rare fetal feature. Deliver babies with heart block
as below
Reference ohcs
Dr Khalid/Rabia


Q: 288 A 24yo 18wk pregnant lady presents with pain in her lower abdomen for the
last 24h. She had painless vaginal bleeding. Exam: abdomen is tender, os is
closed. What is the most probable dx?
a. Threatened miscarriage
b. Inevitable miscarriage
c. Incomplete miscarriage
d. Missed miscarriage
e. Spontaneous miscarriage



Clincher(s) Os is closed
A If symptoms are mild and the cervical os is closed it is a threatened
miscarriage.
Rest is advised but probably does not help. 75% will settle.
B If symptoms are severe and the os is open it is an inevitable miscarriage
C if most of the products have already been passed, an incomplete miscarriage
D Missed miscarriage: The fetus dies but is retained. There has usually been
bleeding and the uterus is small for dates. Confi rm with ultrasound.
Mifepristone
and misoprostol may be used to induce uterine evacuation if the uterus
is small but 50% will require surgical evacuation if uterine products are >5cm2
in the transverse plane, >6cm2 in the sagittal plane. Surgical evacuation is
required
for larger uteruses, if scar (previous caesar), by senior staff .
E Miscarriage is the loss of a pregnancy before 24 weeks’ gestation. 20–40% of
pregnancies miscarry, mostly in the fi rst trimester. Most present with bleeding
PV. Diagnosis may not be straightforward (consider ectopics p262): have a
low threshold for doing an ultrasound scan. Pregnancy tests remain +ve for
several days after fetal death. Heavy/persistent bleeding >2 weeks needs ERPC
KEY A

3
OBS GYN-System Wise 1700-by Sush and Team. 2016
Susmita, Asad, Manu, Saima, Zohaib, Savia, Shanu, Mona, Manisha, Sitara, Samreena, Sami and Komal

Reference ohcs
Dr Khalid/Rabia wikipedia






Q: 290 A 32yo female who has had 3 prv miscarriages in the 1st trimester now comes
with vaginal bleeding at 8wks. US reveals a viable fetus. What would be the
most appropriate definitive management?
a. Admit
b. Aspirin
c. Bed rest 2 weeks
d. Cervical cerclage
e. No tx




Clincher(s)
A
B
C
D

E

KEY B
Additional
Information Recurrent Spontaneous Miscarriage:
This is loss of 3 or more consecutive pregnancies before 24 weeks’ gestation.
It aff ects 1% of women. Prognosis for future successful pregnancy is
aff ected by the previous number of miscarriages, and maternal age. (Rates
of miscarriage are greatest when maternal age is ≥35 years, and paternal
age ≥ 40 years.)
Possible causes
Endocrine: Polycystic ovaries are thought to be associated via insulin
resistance.
Metformin has been shown in small uncontrolled trials to reduce RSM.
The role of progesterone in RSM is still being studied.
Infection: Bacterial vaginosis (p284) is associated with 2nd trimester loss.
Screening (and treatment) was previously recommended for those with
previous
mid-trimester miscarriage or pre-term birth (benefi t unproven).
Parental chromosome abnormality: 2–5% of those with RSM. It is usually a

4
OBS GYN-System Wise 1700-by Sush and Team. 2016
Susmita, Asad, Manu, Saima, Zohaib, Savia, Shanu, Mona, Manisha, Sitara, Samreena, Sami and Komal

balanced reciprocal or Robertsonian translocation (p152). Refer to a clinical
geneticist. Genetic counselling off ers prognosis for future pregnancy, familial
chromosome studies, and appropriate advice for subsequent pregnancy.
Pre-implantation genetic diagnosis (p13—involving in vitro fertilization) has
lower rates of achieving healthy pregnancy outcome compared to natural
conception (30% vs 50%).
Uterine abnormality: It is uncertain how much abnormality is associated
with RSM or if hysteroscopic correction of abnormality contributes to
successful
pregnancy outcome, though septum division may help. 65 It is known
that open uterine surgery increases chance of uterine rupture in pregnancy.
Antiphospholipid antibodies: (lupus anticoagulant, phospholipid and
anticardiolipin antibodies) These are present in 15% of women with RSM. Most
women with antibodies miscarry in the fi rst trimester If. they are present,
giving aspirin eg 75mg/24h PO from the day of positive pregnancy test + low
molecular weight heparin, eg enoxaparin 40mg/24h sc) 66 as soon as the fetal
heart is seen (eg at 5 weeks on vaginal ultrasound) until 34 weeks’ gestation
helps. 67 Get expert advice. Resulting pregnancies are at high risk of repeated
miscarriage, pre-eclampsia, fetal growth restriction, and pre-term birth so
need special surveillance. Live birth rate is ~80%.
Thrombophilia: In those with inherited thrombophilia heparin helps those
who suff er from 2nd trimester losses but evidence is less certain for 1st
trimester
losses.
Alloimmune causes: The theory is that these women share human leucocyte
alleles (HLA) with their partners and do not mount the satisfactory protective
response to the fetus. Immunotherapy has not been found to increase
live birth rate, is potentially dangerous and should not be off ered.
Recommendation 68
• Off er referral to specialist recurrent miscarriage clinic.
• Test all women for antiphospholipid antibodies: positive if 2 tests +ve,
taken 12 weeks apart.
• Women with 2nd trimester losses test for thrombophilia.
• All women with recurrent 1st trimester losses, (or more than 1, 2nd
trimester
loss) should have pelvic ultrasound to assess uterus; further tests eg 3-D
ultrasound/laparoscopy/hysteroscopy if anatomy abnormal.
• Karyotype fetal products (3rd and subsequent fetal losses). If an unbalanced
chromosome abnormality is identifi ed in the products of conception
then karyotype the peripheral blood of both parents
Reference ohcs
Dr Khalid/Rabia



5
OBS GYN-System Wise 1700-by Sush and Team. 2016
Susmita, Asad, Manu, Saima, Zohaib, Savia, Shanu, Mona, Manisha, Sitara, Samreena, Sami and Komal

Q:292 A 27yo 34wk pregnant lady presents with headache, epigastric pain and
vomiting. Exam: pulse=115, BP=145/95mmHg, proteinuria ++. She complains
of visual disturbance. What is the best medication for the tx of the BP?
a. 4g MgSO4 in 100ml 0.9%NS in 5mins
b. 2g MgSO4 IV bolus
c. 5mg hydralazine IV
d. Methyldopa 500mg/8h PO
e. No tx


Only when 160/100 then intervention

When pre-eclamsia – hydralazine or methyldopa
With seizure- MgSO4

(if Bp was not mentioned then preeclamsia rx)

Clincher(s) Blood pressure 145/95
A
B
C
D
E
KEY E CHECK OHCM
Additional
Information Management of pre eclampsia:
This applies to those with BP >160/110 with proteinuria or BP ≥140/90 with
proteinuria plus one or more of:
• Seizures • Headache or epigastric pain • Platelets <100 ≈ 109/L
• Visual disturbance • Papilloedema • ALT >70U/L
• Clonus (>3 beats) • Liver tenderness or HELLP (p26)
Continuously monitor maternal oxygen saturation, and BP.
Use prophylactic magnesium sulfate: 4g (8mL of 50% solution) IVI over
15min in 100mL 0.9% saline; then maintenance as below.
Catheterize: measure urine output (eg use urometer) & T° hourly; FBC, U&E,
LFTs, creatinine every 12–24h. If platelets <100 ≈ 109/L do clotting studies.
Monitor fetal heart rate; assess liquor volume and fetal growth by scan;
umbilical cord Doppler if possible. Use monitoring in labour.
Delivery is the only cure for these women. When a decision is made to
deliver,
contact on-call consultant, anaesthetist, and senior labour ward midwife.
Deliver appropriately (eg <34 weeks usually by caesar). Give steroids
if <34–36 wks, (p51). At 3rd stage of labour give 5U oxytocin IM/IV slowly.
Treatment of hypertension: Beware: automated BP devices underestimate BP.
If BP >160/110mmHg or mean arterial pressure >125mmHg, use labetalol
20mg IV increasing after 10min intervals to 40mg then 80mg until 200mg

6
OBS GYN-System Wise 1700-by Sush and Team. 2016
Susmita, Asad, Manu, Saima, Zohaib, Savia, Shanu, Mona, Manisha, Sitara, Samreena, Sami and Komal

total is given. Aim for BP 150/80–100mmHg. Alternative is hydralazine 5mg
slowly/20min til 20mg given (unless pulse>120bpm ) after 500mL colloid IV.
Give prophylactic H2 blockers until normal postnatal care starts.
Restrict fl uids to 80mL/h. Hourly urine output. Renal failure is rare. Maintain
fl uid restriction until postpartum diuresis. Fluid restriction is inappropriate
if there is haemorrhage.
Pitfalls in the management of eclampsia and pre-eclampsia
• Belief that the disease behaves predictably, and that BP is a good marker.
• Ignoring mild proteinuria; even if 1+, the patient may be dead within 24h.
• Believing antihypertensives stop pre-eclampsia (they may stop stroke).
They don’t. Only delivery cures. Diuretics deplete plasma volume, and are
especially contraindicated (except in the rare left ventricular failure or
laryngeal
oedema complicating pre-eclampsia). Continue antihypertensives
postpartum; wean off slowly. Avoid methyldopa (depression).
• Believing that delivery removes risk. In the UK, 44% of eclamptic fi ts are
postpartum: half of these are >48h postpartum. Continue vigilance until
clinically and biochemically normal. Avoid discharge <5 postnatal days.
• Ergometrine should not be used for the 3rd stage (it  BP and risks stroke).
• Not replacing signifi cant blood loss meticulously. Risks are hypovolaemia or
fatal fl uid overload } acute respiratory distress syndrome. Have one person
(the most experienced) in charge of all the IVIs.
• Failure to inform anaesthetists early and use intensive care facilities.
• Not asking GP to check for proteinuria at 6 weeks (13% have renal disease
or underlying hypertension). Refer to physician if proteinuria still present
Reference ohcs
Dr Khalid/Rabia


Q: 293 A 24yo lady who is 37wk pregnant was brought to the ED. Her husband says a
few hours ago she complained of headache, visual disturbance and abdominal
pain. On arrival at the ED she has a fit. What is the next appropriate
management for this pt?
a. 4g MgSO4 in 100ml 0.9%NS in 5mins
b. 2g MgSO4 IV bolus
c. 2g MgSO4 in 500ml NS in 1h
d. 4g MgSO4 IV bolus
e. 10mg diazepam in 500ml 0.9%NS in 1h


4g MgSO4 in 14g MgSO4 in 100ml 0.9%NS in 5mins: in eclamsia
…. in 15mins: preeclamsia



Clincher(s)

7
OBS GYN-System Wise 1700-by Sush and Team. 2016
Susmita, Asad, Manu, Saima, Zohaib, Savia, Shanu, Mona, Manisha, Sitara, Samreena, Sami and Komal

A
B
C
D
E
KEY A
Additional Treatment of seizures (eclampsia):
Information Treat a first seizure with 4g magnesium sulfate in 100mL 0.9% saline IVI
over 5min + maintenance IVI of 1g/h for 24h. Beware of respiratory
depression.
If recurrent seizure give 2g IVI magnesium sulfate over 5 min.
Check tendon reflexes and respiratory rate every 15min.
Stop magnesium sulfate IVI if respiratory rate <14/min or tendon refl ex
loss, or urine output <20mL/h. Have IV calcium gluconate ready in case of
MgSO4 toxicity: 1g (10mL) over 10 min if respiratory depression.
Use diazepam once if fi ts continue (eg 5–10mg slowly IV). If seizures
continue, ventilate and consider other causes (consider CT scan).

Reference ohcs
Dr Khalid/Rabia

Q: 296 A 24yo girl comes to the woman sexual clinic and seeks advice for
contraception. She is on sodium valproate.
a. She can’t use COCP
b. She can use COCP with extra precaution
c. She can use COCP if anticonvulsant is changed to carbamezapin.
d. She can use COCP with estrogen 50ug and progesterone higher
dose
e. She can use COCP


Clincher(s) sodium valproate is not an enzyme inducer
A
B
C
D
E
KEY E
Additional
Information Contraception and epilepsy

Some anti-epilepsy medicines have a side-effect of increasing the speed in


which some contraceptive pills and injections are processed by the liver. These
medicines are known as liver enzyme inducers, as they speed up certain
processes in the liver cells.

8
OBS GYN-System Wise 1700-by Sush and Team. 2016
Susmita, Asad, Manu, Saima, Zohaib, Savia, Shanu, Mona, Manisha, Sitara, Samreena, Sami and Komal

The following anti-epilepsy medicines are liver enzyme inducers:

• carbamazepine
• eslicarbazepine
• oxcarbazepine
• phenobarbital
• phenytoin
• primidone
• topiramate

The other anti-epilepsy medicines, including sodium valproate, lamotrigine and


ethosuximide, are not liver enzyme inducers. If you are taking an anti-epilepsy
medicine which is not a liver enzyme inducer then your contraceptive choices,
doses, etc, are usually the same as for any other women (although see below
about lamotrigine).

However, if you are taking an anti-epilepsy medicine that is a liver enzyme


inducer, the following is recommended:

• If you take the combined oral contraceptive pill (COCP, or 'the pill') -
the dose of the oestrogen part needs to be at least 50 micrograms,
which is more than the usual dose. However, it is usually preferable to
use alternative contraception, if possible.
• The progestogen-only contraceptive pill (POCP) is not recommended.
• Progestogen implants are not recommended.
• The combined transdermal contraceptive patch is not recommended.
• If you use emergency contraception tablets - the initial dose of
levonorgestrel should be increased to 3 mg (you will need to take two
tablets instead of one).
• The progestogen injection called Depo-Provera® can be used but the
injections need to be given more frequently.


Reference Patient .info
Dr Khalid/Rabia



Q: 297 A 27yo lady came to the ED 10 days ago with fever, suprapubic tenderness and
vaginal discharge. PID was dx. She has been on the antibiotics for the last
10days. She presents again with lower abdominal pain. Temp=39.5C. what is
the most appropriate next management?
a. Vaginal swab
b. Endocervical swab
c. US
d. Abdominal XR

9
OBS GYN-System Wise 1700-by Sush and Team. 2016
Susmita, Asad, Manu, Saima, Zohaib, Savia, Shanu, Mona, Manisha, Sitara, Samreena, Sami and Komal

e. Laparoscopy



Clincher(s)
A
B
C
D
E
KEY C- US to rule out abscess
Additional Initial presentation was of PID. But recurring of symptoms may suggest
Information resistant conditions like abscess formation].

Reference
Dr Khalid/Rabia


Q: 301 A 27yo presents with abdominal pain, bleeding, vomiting and diarrhea. Her
LMP was 7wks ago. Exam: abdominal tenderness, BP=90/60mmHg. What is the
next appropriate management?
a. Immediate laparotomy
b. Laparoscopy
c. Salpingotomy
d. Salpingectomy
e. MTX



Clincher(s)
A
B
C
D
E
KEY A – susc ruptured ectopic
Additional In ectpic – gastritis is an unsual presentation
Information
Reference
Dr Khalid/Rabia


Q: 304 Which of the following is NOT a physiological change during pregnancy?
a. Tidal volume 500ml
b. RBC vol 1.64L
c. Cardiac output 6.5L/min

10
OBS GYN-System Wise 1700-by Sush and Team. 2016
Susmita, Asad, Manu, Saima, Zohaib, Savia, Shanu, Mona, Manisha, Sitara, Samreena, Sami and Komal

d. Uterus weight 1.1kg
e. ESR up by 4x



Clincher(s)
A Ventilation increases 40% (tidal volume rises from 500 to
700mL), the increased depth of breath being a progesterone effect. given tidal
volume is normal for non pregnant state..
In pregnancy it should rise to 700 m

B Red cell volume rises from 1.4 litres when non-pregnant to 1.64 litres at term if
iron supplements not taken (18%), or 1.8 litres at term ( 30%) if
supplements are taken—
C Cardiac output rises from 5 litres/min to 6.5–7 litres/min in
the first 10 weeks by increasing stroke volume (10%) and pulse rate (by ~15
beats/min)
D The 100g non-pregnant uterus weighs 1100g by term
E Erythrocyte sedimentation rate , ESR (whole blood)

Units Nonpregnant First Trimester Second Third Trimester


Female Trimester
mm/h 0 - 20 4 - 57 7 - 47 13 – 70



KEY A
Additional Albumin, gama globulin, U creatinne decrease because of increased plasma
Information volume
Reference
Dr Khalid/Rabia


Q: 313 A 45yo lady came to family planning clinic for contraception advice. She is not
keen to be pregnant for the next 3yrs. Her recent US showed multiple small
submucosal fibroid. What is the best method of contraception for her?
a. Etonogestrol
b. COCP
c. IUS
d. POP
e. IUCD



Clincher(s)
A
B

11
OBS GYN-System Wise 1700-by Sush and Team. 2016
Susmita, Asad, Manu, Saima, Zohaib, Savia, Shanu, Mona, Manisha, Sitara, Samreena, Sami and Komal

C
D
E
KEY IUS- because long term and in fibroids its safer
Additional
Information
Reference
Dr Khalid/Rabia

Q: 9 A pregnant lady came with pain in her calf muscle with local rise in temp to the
antenatal clinic.
What tx should be started?
a. Aspirin
b. LMWH
c. Paracetamol
d. Cocodamol
e. Aspirin and heparin


Clincher(s)
A Pregnancy contraindicated
B Correct Answer
C Can be safe, but can harm nursing baby through breast milk
D Can depress neonatal respiration
E
KEY B Safest drug in pregnancy

12
OBS GYN-System Wise 1700-by Sush and Team. 2016
Susmita, Asad, Manu, Saima, Zohaib, Savia, Shanu, Mona, Manisha, Sitara, Samreena, Sami and Komal

Additional Thromboembolism ! best choice of drug LMWH in pregnancy
Information Warfarin choice of drug (anti-coagulant) in non-pregnant state

DVT Wells’ Algorithm
























Investigation
Initial " DUPLEX USG
Gold Standard " INVASIVE VENOGRAPHY
Reference
Dr Khalid/Rabia During pregnancy :
Start LMWH and continue throughout pregnancy. Stop the injections 24 hours
before labour and then restart them 4 hours post op. Warfarin is
Contraindicated in pregnancy.
If NO pregnancy :
the protocol is different.
- LMWH stat
- Start Warfarin within 24 hours
- Monitor INR and withdraw LMWH when value is 2.0
- Depending on provoked or non provoked, give Warfarin for 3 and 6 months
respectively and then stop.
- IVC filter is used when anticoagulants fail
- Compression stockings to all patients to prevent 'Post-phlebitic limb changes'

13
OBS GYN-System Wise 1700-by Sush and Team. 2016
Susmita, Asad, Manu, Saima, Zohaib, Savia, Shanu, Mona, Manisha, Sitara, Samreena, Sami and Komal




Q: 24 A 40yo woman notices increasing lower abdominal distention with little/no
pain. On examination, a lobulated cystic mass is felt and it seems to be arising
from the pelvis. What is the most appropriate inv?
a. CA 125
b. CA 153
c. CA 199
d. CEA
e. AFP


Clincher(s) Lobulated Cyst
A Ovarian Cancer
B Breast Cancer
C Pancreatic Cancer
D CEA- colorectal
E AFP- germ cell tumor and hepatocellular
KEY A CA-125 test
Additional
Information




14
OBS GYN-System Wise 1700-by Sush and Team. 2016
Susmita, Asad, Manu, Saima, Zohaib, Savia, Shanu, Mona, Manisha, Sitara, Samreena, Sami and Komal

15
OBS GYN-System Wise 1700-by Sush and Team. 2016
Susmita, Asad, Manu, Saima, Zohaib, Savia, Shanu, Mona, Manisha, Sitara, Samreena, Sami and Komal

16
OBS GYN-System Wise 1700-by Sush and Team. 2016
Susmita, Asad, Manu, Saima, Zohaib, Savia, Shanu, Mona, Manisha, Sitara, Samreena, Sami and Komal


Reference OHCS , OHOBGYN 703 , Pass Medicine
Dr Khalid/Rabia DIAGNOSIS : Ovarian CA
Age - 60 to 80 women

SYMPTOMS :
• Early symptoms are often vague, such as abdominal discomfort,
abdominal distension or bloating, urinary frequency or dyspepsia.
Constitutional symptoms include fatigue, weight loss, anorexia
and depression.
• It most commonly presents with a pelvic or abdominal mass that
may be associated with pain. Abdominal, pelvic or back pain is
usually a late sign
• It may cause abnormal uterine bleeding.
• Often associated with ascites. One third of patients with ascites
also have a pleural effusion.

INVESTIGATIONS :

• CA-125 test.
• If this is reported as raised (35 IU/mL or greater) arrange pelvic
and abdominal ultrasound scans.
• CT is the investigation of choice in the UK

Treatment :

The standard comprehensive surgical staging approach consists of a total


abdominal hysterectomy and bilateral salpingo-oophorectomy (TAH and
BSO)

The standard regime is paclitaxel and carboplatin given intravenously every


three weeks for six cycles.

Radiotherapy.

PRGNOSIS :

• Stage I: 92%
• Stage II: 55%
• Stage III: 21.9%
• Stage IV: 5.6%




Q: 45 A 32yo woman of 39wks gestation attends the antenatal day unit feeling very

17
OBS GYN-System Wise 1700-by Sush and Team. 2016
Susmita, Asad, Manu, Saima, Zohaib, Savia, Shanu, Mona, Manisha, Sitara, Samreena, Sami and Komal

unwell with sudden onset of epigastric pain associated with nausea and
vomiting. Her temp is 36.7C. Exam: she is found to have RUQ tenderness. Her
blood results show mild anemia, low platelets, elevated liver enzymes and
hemolysis. What is the most likely dx?
a. Acute fatty liver of pregnancy
b. Acute pyelonephritis
c. Cholecystitis
d. HELLP syndrome
e. Acute hepatitis


Clincher(s)
A Only elevated enzymes
B Site of pain, no enzymes
C
D Correct answer
E Only enzymes and hx of hep A or E
KEY D H – haemolysis, EL - elevated liver enzymes, LP - low platelets
Additional HELLP syndrome
Information
This is a serious complication regarded by most as a variant of severe
pre-eclampsia which manifests with haemolysis (H), elevated liver
enzymes (EL), and low platelets (LP).

18
OBS GYN-System Wise 1700-by Sush and Team. 2016
Susmita, Asad, Manu, Saima, Zohaib, Savia, Shanu, Mona, Manisha, Sitara, Samreena, Sami and Komal




Reference OHCS 26 & PassMedicine
Dr Khalid/Rabia Ans. 2. The main treatment is to deliver the baby as soon as possible [as early
as after 34 weeks if multisystem disease is present].

HELLP syndrome is a group of symptoms that occurs in pregnant women who
have pre-eclampsia or eclampsia and who also show signs of liver damage and
abnormalities in blood clotting.
H hemolysis
EL (elevated liver) enzymes

19
OBS GYN-System Wise 1700-by Sush and Team. 2016
Susmita, Asad, Manu, Saima, Zohaib, Savia, Shanu, Mona, Manisha, Sitara, Samreena, Sami and Komal

LP (low platelet) count




Q: 52 A female with T1DM would like to know about deficiency of vitamins in
pregnancy that can be harmful. A deficiency of which vitamin can lead to
teratogenic effects in the child?
a. Folic acid
b. Vit B12
c. Thiamine
d. Riboflavin
e. Pyridoxine



Clincher(s)
A Correct asnwer
B
C
D
E
KEY A Folic Acid (B6) deficiency " Neural Tube Defects
Additional Neural tube defects (NTDs) are the second most common severe disabling
Information human congenital defects.

















20
OBS GYN-System Wise 1700-by Sush and Team. 2016
Susmita, Asad, Manu, Saima, Zohaib, Savia, Shanu, Mona, Manisha, Sitara, Samreena, Sami and Komal


Reference OHCS 140 , OHOBGYN

21
OBS GYN-System Wise 1700-by Sush and Team. 2016
Susmita, Asad, Manu, Saima, Zohaib, Savia, Shanu, Mona, Manisha, Sitara, Samreena, Sami and Komal

Dr Khalid/Rabia Diet: To prevent neural tube defects (NTD) and cleft lip, all should have folate
rich foods + folic acid 0.4mg daily >1 month pre-conception till 13wks
(5mg/day if past NTD, on anti epileptics, obese (BMI ≥30), HIV+ve on co-
trimoxazole prophylaxis, diabetic or sickle cell disease.
Smoking: decreases ovulations, causes abnormal sperm production (± less
penetrating capacity), rates of miscarriage (≈2), and is associated with
preterm labour and lighter-for-dates babies placenta praevia and abruption.
Reduced reading ability in smokers’ children up to 11yrs old shows that long
term effects are important.
Alcohol consumption: High levels of consumption are known to cause the fetal
alcohol syndrome. Mild drinking eg 1–2U/wk has not been shown to adversely
affect the fetus. Especially harmful in weeks 3-8.Miscarriage rates are higher
among drinkers of alcohol



Q: 100 A 27yo lady has had an uncomplicated pregnancy so far. She came to the
hospital 2h ago after her water broke. The midwife is looking at her now. She
has regular contractions. P.V exam revealed 2cm dilated cervix. Vital signs are
normal. What stage of labour is she in?
a. Second stage
b. First stage
c. Latent stage
d. Third stage
e. Active phase


Clincher(s) 2 cm dilated cervix
A
B Dilates till 4cm
C No such thing as latent stage , ‘latent phase’ part of 1st stage. Dil to 4cm.
D
E
KEY B First stage

22
OBS GYN-System Wise 1700-by Sush and Team. 2016
Susmita, Asad, Manu, Saima, Zohaib, Savia, Shanu, Mona, Manisha, Sitara, Samreena, Sami and Komal

Additional
Information











































23
OBS GYN-System Wise 1700-by Sush and Team. 2016
Susmita, Asad, Manu, Saima, Zohaib, Savia, Shanu, Mona, Manisha, Sitara, Samreena, Sami and Komal










24
OBS GYN-System Wise 1700-by Sush and Team. 2016
Susmita, Asad, Manu, Saima, Zohaib, Savia, Shanu, Mona, Manisha, Sitara, Samreena, Sami and Komal





Reference OHCS 58
Dr Khalid/Rabia Ans. The key is B. First stage starts with softening of cervix with start of
opening of cervix and ends when cervix is fully dilated (i.e. 10 cm dilated).
[There is nothing named “latent stage” but latent phase which is up to 4cm
dilatation. So, the preferred option is first stage here].
Stages of Labour

25
OBS GYN-System Wise 1700-by Sush and Team. 2016
Susmita, Asad, Manu, Saima, Zohaib, Savia, Shanu, Mona, Manisha, Sitara, Samreena, Sami and Komal

First stage
The first stage begins with regular contractions (when the fetal presenting part
has descended into the true pelvis), or on admission to hospital with obvious
signs of labour.
The first stage ends when the cervix is fully dilated (10 cm).
First stage can be divided into:

Latent or quiet phase: Contractions are not particularly painful and at 5- to 10-
minute intervals. Contractions become stronger with shorter intervals,
although the cervix is still dilating relatively slowly, with membranes possibly
breaking later in this phase.

Active phase:Starts with the cervix 3-4 cm dilated and is associated with more
rapid dilatation normally at 0.5-1.0 cm/hour. Once the cervix is dilated to 9 cm,
towards the end of the active phase, contractions may be more painful and
women may want to push. Pushing is undesirable at this stage; there is the
need to establish by vaginal examination whether the cervix is fully dilated.
During this time the fetal head descends into the maternal pelvis and the fetal
neck flexes.

While the length of established first stage of labour varies between women,
first labours last on average 8 hours (unlikely ≥18 hours). Second and
subsequent labours last on average 5 hours (unlikely ≥12 hours). However if
the first stage does not appear to be progressing, the cause needs to be
determined.

Second stage:
This starts when the cervix is fully dilated and ends with the birth of the baby:

Contractions are stronger, occur at 2- to 5-minute intervals and last 60-90


seconds.
The fetal head descends deeply into the pelvis and rotates anteriorly so that
the back of the fetal head is behind the mother's symphysis pubis (98% of
cases).
The second stage is said to be active once the baby is visible and the woman
usually also wants to assist what have become expulsive contractions by
pushing.
The fetal head becomes more visible with each contraction until a large part of
the head can be seen.
The head is now born with first the forehead, then the nose, mouth and chin.
The head rotates to allow the shoulders to be born next, followed by the trunk
and legs.
After this, the baby should start to breathe and to cry loudly.

26
OBS GYN-System Wise 1700-by Sush and Team. 2016
Susmita, Asad, Manu, Saima, Zohaib, Savia, Shanu, Mona, Manisha, Sitara, Samreena, Sami and Komal

Third stage:
This stage starts with the birth of the baby and ends with the delivery of the
placenta and membranes:

Separation of the placenta occurs immediately after birth due to forceful


uterine contractions along with retraction of the uterus, thus greatly reducing
the size of the placental bed.
It normally takes up to 5 minutes, but can take longer.
Haemorrhaging is prevented by the contraction of uterine muscle fibres closing
off the blood vessels that were supplying the placenta.
Without active management, after 10-20 minutes, separation is shown by a
gush of blood, prominence of the fundus in the abdomen and apparent
lengthening of the umbilical cord.


Q: 102 A 29yo woman had just delivered a still born vaginally, following a major
placental abruption. Choose the single most likely predisposing factor for
developing PPH in this lady?
a. Retained product
b. DIC
c. Fibroid uterus
d. Uterine infection
e. Large placental site


Clincher(s) PPH , Major Placental Abruption
A
B Correct answer (common cause) DIC for abruptio and retain product for
placenta previa..

C
D
E
KEY B DIC (Disseminated intravascular coagulation) ! common cause – Placental
Abruption
Additional Eclampsia, retain placenta or retained dead fetus, sepsis , and amniotic fluid
Information embolism can predispose to DIC


Reference http://patient.info/doctor/postpartum-haemorrhage
Dr Khalid/Rabia Ans. 1. The key is B. DIC.
Ans. 2. Pregnancy itself is a risk factor for DIC. Placental abruption is a more
common cause of DIC. Other causes of pregnancy related DIC are: eclampsia,
retention of a dead fetus, amniotic fluid embolism, retained placenta or

27
OBS GYN-System Wise 1700-by Sush and Team. 2016
Susmita, Asad, Manu, Saima, Zohaib, Savia, Shanu, Mona, Manisha, Sitara, Samreena, Sami and Komal

bacterial sepsis.



Q: 103 A 28yo woman has delivered with rotational forceps after an 8h labor and 3h
second stage. Choose the single most likely predisposing factor for PPH for this
pt?
a. Atonic uterus
b. Cervical/vaginal trauma
c. Retained product
d. Preterm labor
e. Uterine infection


Clincher(s) Rotational forceps
A
B Correct answer
C
D
E
KEY B B Complication of forceps delivery

28
OBS GYN-System Wise 1700-by Sush and Team. 2016
Susmita, Asad, Manu, Saima, Zohaib, Savia, Shanu, Mona, Manisha, Sitara, Samreena, Sami and Komal

Additional
Information


Reference OHCM 84 , OHOBGYN 322
Dr Khalid/Rabia Primary PPH is the loss of greater than 500mL (defi nitions vary) in the first
24h after delivery
Causes: uterine atony (90%), genital tract trauma (7%), clotting disorders—
(3%)
Risks: Antenatal • Previous PPH or retained placenta BMI>35kg/m2 •
Maternal Hb<8.5g/dl at onset of labour • Antepartum haemorrhage
Multiparity 4+ • Maternal age 35y+ • Uterine malformation or fibroids A large
placental site (twins, severe rhesus disease, large baby) • Low placenta,
Overdistended uterus (polyhydramnios, twins) • Extravasated blood in the
myometrium (abruption).
In labour • Prolonged labour (1st, 2nd or 3rd stage) • Induction or oxytocin use
• Precipitant labour • Operative birth or caesarean section. Book mothers with
risk factors for obstetric unit delivery.
Treatment: Give oxytocin 5U slowly IV for atonic uterus.
Attach oxygen, Give IV fluids, maintain systolic >100mmHg, Transfuse blood.
Is the placenta delivered? If it is, is it complete? If not, explore the uterus. • If
the placenta is complete, put the patient in the lithotomy position with
adequate analgesia and good lighting. Check for and repair trauma.

29
OBS GYN-System Wise 1700-by Sush and Team. 2016
Susmita, Asad, Manu, Saima, Zohaib, Savia, Shanu, Mona, Manisha, Sitara, Samreena, Sami and Komal

• If the placenta has not been delivered but has separated, attempt to deliver
it by controlled cord traction after rubbing up a uterine contraction. If this fails,
ask an experienced obstetrician to remove it under general
anaesthesia.Beware renal shut down.



Q: 128 A 38yo woman has delivered after an induced labor which lasted 26h. choose
the single most likely predisposing factor for postpartum hemorrhage?
a. Atonic uterus
b. Cervical/vaginal trauma
c. Rupture uterus
d. Fibroid uterus
e. Age of mother


Clincher(s) Induced labour lasting 26 hours
A Correct answer
B
C
D
E
KEY A A: Labour lasted 26 hours " uterus lost its tone therefore cant contract "
excessive bleeding = PPH
Additional
Information


Reference http://www.gpnotebook.co.uk/simplepage.cfm?ID=-644218879
Dr Khalid/Rabia


Q: 133 A 25yo woman presented to her GP on a routine check up. Upon vaginal exam,
she was fine except for finding of cervical ectropion which was painless but
mild contact bleeding on touch. What is the next management?
a. Endometrial ablation
b. Cervical smear
c. Colposcopy

30
OBS GYN-System Wise 1700-by Sush and Team. 2016
Susmita, Asad, Manu, Saima, Zohaib, Savia, Shanu, Mona, Manisha, Sitara, Samreena, Sami and Komal

d. Antibiotics
e. Vaginal US
f. Pack with gauze and leave to dry


Clincher(s)
A
B to exclude cervical cancer
Key; painless, mild Contact bleeding
GP

C
D
E
KEY B
Additional
Information
Reference
Dr Khalid/Rabia Rabia "
There is a red ring around the os because the endocervical epithelium has
extended its territory over the paler epithelium of the ectocervix. Ectropions
extend temporarily under hormonal influence during puberty, with the
combined Pill, and during pregnancy. As columnar epithelium is soft and
glandular, ectropion is prone to bleeding, to excess mucus production, and to
infection. Treatment: Once a normal cervical smear has been confirmed, it is
actively managed only if there are symptoms. After stopping any oestrogen-
containing contraceptive, treatment options are controversial but include
diathermy, cryotherapy, surgery with laser treatment and microwave therapy.

SO THE CORRECT ANSWER IS B.




Q:1326 A homeless person is found wandering on the street. He had ataxic gait,
nystagmus and
opthalmoplegia. He looked unkempt and his clothes had a sweaty odour. He
had a dry mucous membrane with a BP=118/70mmHg and PR=90bpm. Blood
sugar level=8. Alcohol breath test= -ve. What would the most imp initial inv?
a. IV insulin
b. Vit B complex
c. Bolus IV 0.9%NS
d. IV dextrose
e. Antibiotics

31
OBS GYN-System Wise 1700-by Sush and Team. 2016
Susmita, Asad, Manu, Saima, Zohaib, Savia, Shanu, Mona, Manisha, Sitara, Samreena, Sami and Komal

Clincher(s) Alcohol breath test negative with ataxic gait n nystagmus
A Sugars are almost normal

B Yes as patient has few of the symptoms
C Vitals stable
D No signs of hypoglycaemia
E No evidence of any infection
KEY B
Additional Via B complex deficiency leads to symptoms :
Information Extreme tiredness or fatigue.
· A lack of energy or lethargy.
· Being out of breath.
· Feeling faint.
· Headache.
· Ringing in the ears (tinnitus)
· Lack of appetite.
· Peripheral neuropathy
· confusion, anxiety, paranoia
Reference Rabia
Dr Khalid/Rabia


Q:1351 A 4yo boy ingested his grandmother’s medicine and has developed dilated
pupil. What is the
cause?
a. Amitryptiline
b. Paracetamol
c. Iron
d. Digoxin


Clincher(s) Dilated pupil
A Amitryptiline has side effects similar to antocolinergics and causes dilated
pupil
Anticholinergics -- atropine, antihistaminics, Amanita mushrooms.
Toxicity from drugs possessing anti-muscarinic activity presents with a distinct
profile that has been described as "red as a beet" (flushing); "hot as a hare"
(hyperthermia); "dry as a bone" (decreased sweating and mucous
membranes); "blind as a bat" (blurred vision); and "mad as a hatter"
(behavioural effects including delerium, hallucinations, and confusion). Other
signs and symptoms include tachycardia, arrhythmias, hypertension, pupillary
dilatation (mydriasis), muscle twitching, and urinary retention (chronic
overingestion may also cause severe constipation). Overdose with
antihistaminics may also cause seizures.
Treatment is primarily supportive (evaporative cooling, catheterisation). In
patients with profound behavioural effects, benzodiazepines or antipsychotics

32
OBS GYN-System Wise 1700-by Sush and Team. 2016
Susmita, Asad, Manu, Saima, Zohaib, Savia, Shanu, Mona, Manisha, Sitara, Samreena, Sami and Komal

such as haloperidol may be given. The specific antidote is physostigmine
(eserine) which inhibits acetylcholinesterase. This drug will increase circulating
levels of acetylcholine which may then counteract the effects of the reversible
blockade by the anti-cholinergic drug. However, it should be administered in
small (0.5-1 mg, IV) doses only with careful monitoring, since bradycardia and
seizure may result if given too rapidly or in too high a dose.


B No effect on pupil as such
Paracetamol toxicity will present in four distinct phases
1) Within the first day of overdose -- anorexia, nausea, vomiting, malaise
2) 1-2 days post-ingestion -- patient feels better, may eat, get up, but liver
enzymes are elevated
3) 3-5 days -- hepatic necrosis, liver enzymes may peak
4) 7-8 days -- either hepatic failure and death OR if appropriate measures were
taken or the overdose not extensive, improvement and recovery.
Treatment -- N-Acetylcysteine -- This compound may either scavenge the toxic
intermediate directly and/or regenerate additional GSH. It is given IV or PO as
a 5% solution within 36 hours of ingestion. NOTE that it is most effective if
given within 10 hours of ingestion. Either route of administration will require
extemporaneous compounding of the drug from the solution for aerosol that is
currently available. If the parenteral route is chosen, the pharmacist must
observed aseptic technique in compounding the solution for injection. Oral
solutions are often diluted in cola and should be administered within one (1)
hour of preparation. The loading dose is 140 mg/Kg, then 70 mg/Kg q4h for 17
doses or until the risk of hepatoxicity has passed. NOTE that this is based upon
the blood level of APAP. The liver is at risk for irreversable damage if the
plasma level of APAP is approximately 200 mcg/ml 4 hours post ingestion. The
risk for damage continues linearly over time as a function of the log plasma
concentration such that 24 hours post ingestion, the risk is still present when
plasma APAP is over 5 mcg/ml.

C No effect on pupil
The first indication of iron poisoning by ingestion is a stomach pain, as iron is
corrosive to the lining of the gastrointestinal tract including the stomach.
Nausea and vomiting are also common symptoms and bloody vomiting may
occur. The pain then abates for 24 hours as the iron passes deeper into the
body resulting in metabolic acidosis, which in turn damages internal organs,
particularly the brain and the liver. Iron poisoning can cause hypovolemic
shock due to iron's potent ability to dilate the blood vessels.[citation needed]
Death may occur from liver failure.[citation needed]

If intake of iron is during a prolonged period of time, symptoms are likely
similar to other causes of iron overload.
Treatment is by chelation agent desferoxamine and dialysis

33
OBS GYN-System Wise 1700-by Sush and Team. 2016
Susmita, Asad, Manu, Saima, Zohaib, Savia, Shanu, Mona, Manisha, Sitara, Samreena, Sami and Komal


D Causes pinpoint pupil
Digitalis glycosides
Overdose with digoxin or other cardiac glycosides causes severe vomiting,
visual disturbances, fatigue, confusion, hyperkalaemia, and numerous
arrhythmias including AV block and ventricular arrhythmias. (Refer to the
Pharmacology II notes for a compleat discussion of digoxin toxicity).
Treatment depends upon the clinical situation. Uncomplicated arrhythmias
(normal digoxin and potassium levels) may be treated with lidocaine (the
drug of choice) or phenytoin. Hypokalaemia-induced digoxin toxicity may be
treated with parenteral potassium. The antidote DigibindR (digoxin immune
fab fragments -- antibodies to the digoxin molecule) should only be used in
cases of supra-therapeutic plasma concentrations of digoxin. Note that if the
drug is digitoxin, ouabain, or strophantine, Digibind may not be as effective
(incompleat cross reactivity) and higher doses may need to be administered.
E
KEY A

34
OBS GYN-System Wise 1700-by Sush and Team. 2016
Susmita, Asad, Manu, Saima, Zohaib, Savia, Shanu, Mona, Manisha, Sitara, Samreena, Sami and Komal

Additional
Information

Reference NHS pat .co.uk


Dr Khalid/Rabia


Q:

Clincher(s)
A

35
OBS GYN-System Wise 1700-by Sush and Team. 2016
Susmita, Asad, Manu, Saima, Zohaib, Savia, Shanu, Mona, Manisha, Sitara, Samreena, Sami and Komal

B
C
D
E
KEY
Additional
Information
Reference
Dr Khalid/Rabia


Q:

Clincher(s)
A
B
C
D
E
KEY
Additional
Information
Reference
Dr Khalid/Rabia


Q:

Clincher(s)
A
B
C
D
E
KEY
Additional
Information
Reference
Dr Khalid/Rabia

Q:

Clincher(s)
A
B

36
OBS GYN-System Wise 1700-by Sush and Team. 2016
Susmita, Asad, Manu, Saima, Zohaib, Savia, Shanu, Mona, Manisha, Sitara, Samreena, Sami and Komal

C
D
E
KEY
Additional
Information
Reference
Dr Khalid/Rabia

Q:

Clincher(s)
A
B
C
D
E
KEY
Additional
Information
Reference
Dr Khalid/Rabia

Q:1283 77yo publican was admitted for an appendectomy. Post-op he becomes
confused, agitated and starts to pick at things. He is then given an IV drug
which settles this confusion. Which of the following drugs was given for his
confusion?
a. Diazepam
b. Chlordiazepoxide
c. Thiamine
d. Vit B


Incomplete question
Clincher(s) Symptoms of alcohol withdrawal
A Is a second line drug for alcohol withdrawal
B First line drug for alcohol withdrawal (D Tremens)
C Deficiency can cause Wernicke's encephalopathy, which if left untreated, can
lead to Korsakoff's syndrome.
Oral thiamine is poorly absorbed in dependent drinkers. For this reason, all
those undergoing detoxification in the community should be considered for
admission for parenteral high-potency B complex vitamins (Pabrinex®) as
prophylactic treatment. However, because of the risk of anaphylaxis,
resuscitation facilities need to be available at the time of administration. The
risk of anaphylaxis is lower if the drug is given intramuscularly (IM).

37
OBS GYN-System Wise 1700-by Sush and Team. 2016
Susmita, Asad, Manu, Saima, Zohaib, Savia, Shanu, Mona, Manisha, Sitara, Samreena, Sami and Komal

As prophylactic treatment, one pair of ampoules of Pabrinex® should be given
IM or intravenously (IV) once a day for three to five days. A pair of ampoules
contains 250 mg of thiamine.
D Same as above
E
KEY For alcohol withdrawal, chlordiazepoxide is 1st line. Diazepam is 2nd line.
Benzodiazepines are the recommended drugs for detoxification. They have a
slower onset of action and therefore are less likely to lead to abuse. A reducing
dose of chlordiazepoxide over 5-7 days is commonly used.
Diazepam is an alternative.
• Symptoms typically present about eight hours after a significant fall in blood
alcohol levels. They peak on day 2 and, by day 4 or 5, the symptoms have
usually improved significantly.
• Minor withdrawal symptoms (can appear 6-12 hours after alcohol has
stopped)
o Insomnia and fatigue.
o Tremor.
o Mild anxiety/feeling nervous.
o Mild restlessness/agitation.
o Nausea and vomiting.
o Headache.
o Excessive sweating.
o Palpitations.
o Anorexia.
o Depression.
o Craving for alcohol.
• Alcoholic hallucinosis (can appear 12-24 hours after alcohol has stopped)
o Includes visual, auditory or tactile hallucinations.
• Withdrawal seizures (can appear 24-48 hours after alcohol has stopped)
o These are generalised tonic-clonic seizures.
• Alcohol withdrawal delirium or 'delirium tremens' (can appear 48-72 hours
after alcohol has stopped).

Additional British Medical Journal (BMJ) review has suggested the following regimen for
Information moderate alcohol dependence in the community or as an inpatient:

Day 1: 20 mg chlordiazepoxide four times daily.

Day 2: 15 mg chlordiazepoxide four times daily.

Day 3: 10 mg chlordiazepoxide four times daily.

Day 4: 5 mg chlordiazepoxide four times daily.

Day 5: 5 mg chlordiazepoxide twice daily.

38
OBS GYN-System Wise 1700-by Sush and Team. 2016
Susmita, Asad, Manu, Saima, Zohaib, Savia, Shanu, Mona, Manisha, Sitara, Samreena, Sami and Komal

Is medication always needed for detoxification?

Medication may not be needed for detoxification if:

A male patient is drinking <15 units/day or a female patient is drinking <10


units/day AND they do not report any recent withdrawal symptoms or recent
drinking to prevent withdrawal symptoms.

The patient has no alcohol on breath test and no withdrawal signs or


symptoms.


Reference NHS
Dr Khalid/Rabia


Q:1284 A 65yo lady presents with dyspareunia. What will you give her for her
condition?
a. HRT
b. COCP
c. Estrogen gel
d. Testosterone gel

c. Estrogen gel


Clincher(s) Post menopausal with dyspareunia
The diagnosis here is VVA I.e vulvo vaginal atrophy
And remember patient presented with only local problem that is vaginal
dryness not systemic problems like hot flushes or osteoporosis
So for local complaints local application is treatment and for systemic needs
oral treatment
A It's very vague option in that many varieties are there
Oestrogen ,progesterone a ,tobolone and testosterone
B Irregular break through bleeding is a complication generally not advisable
C first-line management for VVA should include non-hormonal vaginal lubricants
and moisturizers and if ineffective then options include several forms of local
therapy. After being fully educated about the available choices
local vaginal estrogen asserts that non-hormonal lubricants and moisturizers in
combination of regular sexual activity should be considered first-line therapies
D Given to improve the libido
E
KEY Oestrogen gel
Additional In older women vaginal dryness due to hormonal deficiency mainly oestrogen
Information Atrophic vaginitis

39
OBS GYN-System Wise 1700-by Sush and Team. 2016
Susmita, Asad, Manu, Saima, Zohaib, Savia, Shanu, Mona, Manisha, Sitara, Samreena, Sami and Komal

Current treatment guidelines for vaginal atrophy recommend the use of
minimally absorbed local vaginal oestrogens, along with non-hormonal
lubricants or moisturisers, coupled with maintenance of sexual activity
Reference Oestrogens:
estrogens used in HRT include oestradiol, oestrone, and oestriol, which,
although chemically synthesized from soya beans or yams, are molecularly
identical to the natural human hormone. Conjugated equine oestrogens
containing about 50–65% oestrone sulphate, with the remainder being
equine oestrogens (mainly equilin sulphate), are also used. Progestagens The
progestagens used in HRT are almost all synthetic and derived
from plant sources. They are structurally different from progesterone.
17-hydroxyprogesterone and 19-nortestosterone derivatives are the pro-
gestagens used most commonly in HRT. 17-Hydroxyprogesterone D
ydrogesterone. • Medroxyprogesterone acetate. •
19-Nortestosterone derivatives Norethisterone. • LEvonorgessterol
Other hormones used at the menopause
• Tibolone: is a synthetic steroid compound that itself is inert, but
on absorption is converted to metabolites with oestrogenic,
progestagenic, and androgenic actions. It is classifi ed as HRT in the
British National Formulary. It is used in post-menopausal women.
• Testosterone: patches and implants may be used to improve libido.
Composition Product name Dosing
Vaginal creams 17β-estradiol conjugated estrogens (formerly conjugated
equine estrogens) Estrace® Vaginal Cream Initial: 2–4 g/d for 1–2 wk
Premarin® Vaginal Cream Maintenance: 1 g/d (0.1 mg active ingredient/g)
0.5–2 g/d (0.625 mg active ingredient/g
Vaginal rings 17β-estradiol Estring® Device containing 2 mg releases
7.5 μg/d for 90 d
Vaginal tablet estradiol hemihydrate Vagifem® Initial: 1 tablet/d for 2 wk
Maintenance: 1 tablet twice/wk (tablet containing 25.8 μg estradiol
hemihydrate equivalent to 25 μg of estradiol)

Dr Khalid/Rabia OhOGchapter 21,page 644


Q:1285 A 35yo lady with subserosal fibroid=4cm and submural fibroid=6cm is planning
for a child.
Which way will you remove the fibroids?
a. Laproscopy
b. Vaginal myomectomy (no adnexenal pathology- if uterine is lower down)
c. Abdominal myomectomy
d. Drugs
e. Reassure


40
OBS GYN-System Wise 1700-by Sush and Team. 2016
Susmita, Asad, Manu, Saima, Zohaib, Savia, Shanu, Mona, Manisha, Sitara, Samreena, Sami and Komal

Clincher(s) Submural fibroid planning for pregnancy
A For subserous fibroids (unless it creates problems)
B Generally done who have completed family and has complaints of
menorrhagia or so with fibroid
C For intramural or mixed fibroids
D Are given who have completed family and not suitable for surgery or wanted
medical treatment generally GNrh analogues like goserilin and buserilin or
nafirilin are given
E Can't reassure the patient as intramural fibroids or submural fibroids
associated with increase chances of bleeding patient may not conceive or
may have increase chances of abortion if she conceives
KEY C
Additional Guys subserosal fibroid causes no harm to conceive it's only submural fibroid
Information because it's closer to endometrium increase chances of bleeding and
abortion hence we should treat submural fibroid here
So treatment option is abdominal myomectomy
Reference For any fibroid related complications if patient has completed family other
newer treatment now a days given is uterine artery embolisation
Dr Khalid/Rabia


Q:1286 A 32yo presents with heavy blood loss, US: uterine thickness>14mm. What is
the best possible
management for her?
a. COCP
b. UAE (mainly for fibroids)
c. Hysteroscopy myomectomy (>5mm- only in post menopausal women)
d. Abdominal myomectomy
e. Endometrial ablation (not done as patient is young and fertility will not be
preserved)


With 32 age, ablation or myomectomy cannot be done
Clincher(s) 32 years lady with heavy bleeding and thickness more than 3 mm
A Can't be given as oestrogen is present will increase bleeding
B Is the best treatment option in this patient as thickness is more than 3 mm and
fertility needs to be preserved
C Myomectomy is not needed in this patient as there is no fibroid
D Same again no fibroid so no myomectomy
E Ablation is done in older patient in whom preservation of fertility is not
required and thickness is less than 3 mm
KEY UAE
Additional UAE can shrink the endometrial bulk and resolve menorrhagia. So preferred
Information here.
for Menorrhagia in young patient , childbearing age, UAE is preferred choice.
If fertility is not needed, thickness is < 3mm then endometrial ablation.

41
OBS GYN-System Wise 1700-by Sush and Team. 2016
Susmita, Asad, Manu, Saima, Zohaib, Savia, Shanu, Mona, Manisha, Sitara, Samreena, Sami and Komal

If fertility is required and >3mm then UAE
If fertility is not needed and >3mm then hysterectomy...
Since here the pt is 32yo so we assume we need to preserve her fertility and
size is 14 mm so UAE is the answer.
Uterine artery embolism UAE would be a very good option here as it would
help reduce the endometrial thickness, I assume she has endometrial
hyperplasia here, myomectomy is treatment of choice for uterine fibroids in
women who are subfertile, COCP contain estrogen which would further
worsen the hyperplasia,

Uterine fibroids
Features
• may be asymptomatic
• menorrhagia
• lower abdominal pain: cramping pains, often during menstruation
• bloating
• urinary symptoms, e.g. frequency, may occur with larger fibroids
• subfertility

Diagnosis
• transvaginal ultrasound

Management
• medical: symptomatic management e.g. with combined oral contraceptive
pill. GnRH agonists may reduce the size of the fibroid but are typically useful
for short-term treatment
• surgery is sometimes needed: myomectomy, hysterscopic endometrial
ablation, hysterectomy
• uterine artery embolization

Reference Khalid sir
Dr Khalid/Rabia


Q:1294 1294. An 84yo woman with drusen and yellow spots in the center of retina.
What is the single most likely dx?
a. Macular degeneration
b. HTN retinopathy
c. MS
d. DM background
e. Proliferative DM retinopathy


Clincher(s) Drusen (made of lipid and fatty protein) and yellow spots in the centre of
retina and age
A The above findings point towards dry form of macular degeneration it's the

42
OBS GYN-System Wise 1700-by Sush and Team. 2016
Susmita, Asad, Manu, Saima, Zohaib, Savia, Shanu, Mona, Manisha, Sitara, Samreena, Sami and Komal

key identifiers of dry type and most common in age of >70 years and is most
common than the wet form
Advanced dry macular degeneration is also known as geographic atrophy and
is the main cause of vision loss in dry type
And wet type effects only 15% of people but 2|3rd of people have significant
visual loss
Cause are as follows:
Age
White race
Common in more lightly coloured iris
Women
Smoking
Obesity
Sleep apnoea
Exposure to sun light
High degrees of myopia
Family history
Symptoms are :
Blurred central close up or distant vision
Scotomas
Straight lines look irregular called metamorphopsia
Micropsia objects look smaller in one eye than other eye
B Causes AV nipping ,ischaemic changes called cotton wool spots,ring of
exudates and optic disc oedema ,copper wire and silver wire arterioles
C Symptoms relating to eye are loss of vision
Colour blindness
Eye pain and flashes of light when moving eye
Nystagmus
Double vision and eye pain
D
E Micro aneurysms will be there on examination
KEY A
Additional dry (geographic atrophy) macular degeneration: characterised by drusen -
Information yellow round spots in Bruch's membrane
• wet (exudative, neovascular) macular degeneration: characterised by
choroidal neovascularisation. Leakage of serous fluid and blood can
subsequently result in a rapid loss of vision. Carries worst prognosis
Features
• reduced visual acuity: 'blurred', 'distorted' vision, central vision is affected
first
• central scotomas
• fundoscopy: drusen, pigmentary changes

Reference
Dr Khalid/Rabia

43
OBS GYN-System Wise 1700-by Sush and Team. 2016
Susmita, Asad, Manu, Saima, Zohaib, Savia, Shanu, Mona, Manisha, Sitara, Samreena, Sami and Komal

Q:1295 A pt presents with headache, blurring of vision and acuity loss. On fundoscopy,
dots and blots were noted with huge red swollen optic disc. What is the most
probable dx?
a. CRAO
b. Branch RAO
c. CRVO
d. Optic atrophy



Clincher(s) Red swollen optic disc
A This will have painless loss of vision
Fundoscopy shows cherry red spot caused by ischaemia
Most common cause is carotid artery atherosclerosis
B At presentation only a part of vision is lost the defect is usually altitudinal or
sectoral
Cotton wool spots may occur
Needs referenc in 24 hours anti coagulation is required
C Described below
D On fundoscopy appears as pale shrunken disc
E
KEY C CRVO
Additional Branch RVO: unilateral, painless blurred vision, metamorphopsia (image
Information distortion) ± a field defect. Fundoscopy will reveal vascular dilatation and
tortuosity of the affected vessels, with associated haemorrhages in that area
only (look for an arc of haemorrhages, like a trail left behind a cartoon image
of a shooting star).

Retinal vein occlusion is one of the most common causes of sudden painless
unilateral loss of vision. Loss of vision is usually secondary to macular
oedema.

Artery occlusion- yellow spots

Vein occlusion- hemorrhages

Central retinal vein occlusion

• incidence increases with age, more common than arterial occlusion

• causes: glaucoma, polycythaemia, hypertension

• severe retinal haemorrhages are usually seen on fundoscopy


Reference

44
OBS GYN-System Wise 1700-by Sush and Team. 2016
Susmita, Asad, Manu, Saima, Zohaib, Savia, Shanu, Mona, Manisha, Sitara, Samreena, Sami and Komal

Dr Khalid/Rabia

Q:1296 A 64yo DM has come for a routine eye check up. Fundoscopy: new vessels all
over the retina. What is the most appropriate management?
a. Strict sugar control
b. Regular eye check ups
c. Non urgent referral to specialist
d. Laser photocoagulation
e. Insulin


Clincher(s) Newer vessels in retina diagnosis is CRVO
A It will be the part of the management as other options too
B
C
D This is the best treatment modality for DM patient with CRVO
E
KEY D
Retinal vein occlusion occurs when one of the tiny retinal veins becomes
blocked by a blood clot. This means that blood cannot drain away from the
retina as easily and there is a backlog of blood in the blood vessels of the
retina. This can lead to a build-up of pressure in the blood vessels. As a
result, fluid and blood start to leak from the blood vessels, which can
damage and cause swelling of the retina, affecting your eyesight.
It is of two types
Central retinal vein occlusion
Branched retinal vine occlusion

45
OBS GYN-System Wise 1700-by Sush and Team. 2016
Susmita, Asad, Manu, Saima, Zohaib, Savia, Shanu, Mona, Manisha, Sitara, Samreena, Sami and Komal

Additional
Information

Reference
Dr Khalid/Rabia Dr rabia

Q1273 A 34yo primigravida who is 16wk GA comes for routine antenatal check up.
Her BP=160/100mmHg. She has a hx of repeated childhood UTI. What is the
most likely cause of her high BP?
a. Essential HTN
b. Chronic pyelonephritis (as high BP)
c. Acute pyelonephritis
d. Pre-eclampsia
e. Chronic UTI (no high BP)


Clinchers 16wk gestation,bp 160/100,repeated childhood UTI
A
B Recurrent episodes of UTI point towards chronic pyelonephritis.
C
D Pre eclampsia has HTN with proteinuria/oedema after 20 weeks of gestation

E
KEY B
Additional Risk Factors:
Information •any structural renal tract anomalies, obstruction or calculi
•Children with vesicoureteral reflux
•Intrarenal reflux in neonates
•Genetic predisposition

46
OBS GYN-System Wise 1700-by Sush and Team. 2016
Susmita, Asad, Manu, Saima, Zohaib, Savia, Shanu, Mona, Manisha, Sitara, Samreena, Sami and Komal

•Any factors predisposing to recurrent
urinary infection
-eg, neurogenic bladd

Reference


Q:1274 A 24yo woman has had lower abdominal pain for 12h. She is otherwise well.
She is at 10wks GA in a planned pregnancy. What is the single most
appropriate test to inv the cause of acute abdomen in this lady?
a. Abdominal US
b. Anti phospholipid screen
c. CBC
d. Transvaginal US (sensitivity is more and can see ectopic and viability)
e. Laparoscopy


Clincher(s)
A
B For repeated miscarriages
C
D Transvaginal Usg is the investigation of choice to diagnose or rule out cervical
incompetence,ectopic pregnancy in early pregnancy
E
KEY D
Additional
information


Q.1275 A pt is at term and in labor, the membranes have ruptured, the liquor contains
meconium but the CTG is normal. The cervix is 3cm dilated. What is the single
most appropriate action?
a. BP monitoring
b. CTG
c. C-section
d. Fetal scalp blood sample
e. Internal rotation

First CTG>if normal CTG> continuous Heart rate monitoring>Fetal scalp blood
if abnormal CTG
Clincher(s) Meconium stained liquor,normal CTG
A
B Normal in this case
C If CTG non reassuring and fetal scalp blood sample show pH less than 7.21
then emergency C-section is recommended
D Since CTG is normal so there is no need to do FSBS

47
OBS GYN-System Wise 1700-by Sush and Team. 2016
Susmita, Asad, Manu, Saima, Zohaib, Savia, Shanu, Mona, Manisha, Sitara, Samreena, Sami and Komal

E
KEY There is no option of continuous CTG monitoring
Additional Management
Information
These recommendations are from the National Institute for Health and Care
Excellence (NICE), 2014.[5]

Intrapartum

• If significant meconium staining is noted in labour, there should be


continuous electronic fetal monitoring.
• This is defined as dark green or black amniotic fluid that is thick or
tenacious, or any amniotic fluid that contains lumps of meconium.
• Transfer mother to obstetric-led care, if it is safe to do so and delivery
is not imminent.
• If there are signs of fetal distress, a fetal blood sample should be
obtained. If pH is <7.21, there should be emergency delivery.
• Ensure that the advanced resuscitation unit and appropriately trained
staff are available.
• There should be no suction prior to delivery.

At delivery - healthy neonate

• If the baby is in good condition (Apgar score >5, based on colour, tone,
heart rate and breathing), there should be no suction.
• The baby should be observed for signs of respiratory distress in the first
hour of life, in the second hour and then two-hourly until 12 hours old.
• If there is blood or if there are lumps of meconium in the oropharynx,
suction should be used in the upper airways.
• Endotracheal intubation at birth in otherwise healthy, term meconium-
stained babies, is no longer recommended. All infants at risk for MAS
who show signs of respiratory distress should be admitted into the
neonatal intensive care units. Close monitoring is important since they
can deteriorate very quickly. Maintenance of adequate oxygenation,
optimal blood pressure, correction of acidosis, hypoglycaemia and
other metabolic disorders is the mainstay of treatment.[4]
• Therapeutic interventions in severe MAS include airway suctioning,
oxygen delivery, or ventilatory support.[9]
• Suction - the National Institute for Health and Care Excellence (NICE)
does not recommend routinely suctioning the nasopharynx and
oropharynx prior to birth of the shoulder and trunk. However, it advises
that the upper airways may be suctioned after the shoulders are
delivered, if thick or tenacious meconium is present in the oropharynx.
If the baby has depressed vital signs after delivery, laryngoscopy and
suction under direct vision should be carried out by a healthcare
professional trained in advanced neonatal life support.[10]
• Oxygen should be given to keep oxygen saturations at 95-98%.

48
OBS GYN-System Wise 1700-by Sush and Team. 2016
Susmita, Asad, Manu, Saima, Zohaib, Savia, Shanu, Mona, Manisha, Sitara, Samreena, Sami and Komal

Ventilation may be necessary. Pneumothoraces will need chest drain
insertion.
• High-frequency oscillation ventilation may be given in some cases.[11]
• Giving prophylactic antibiotics to neonates born through meconium-
stained amniotic fluid has not been shown to reduce the incidence of
MAS (or other complications).
• Surfactant - meconium flowing into the lung deactivates the activity of
surfactant, causes a rise in surface tension and presages the onset of
respiratory dist · Surfactant replacement can be beneficial for babies
with MAS, as it can rapidly improve oxygenation.
• · Surfactant replacement by bolus or slow infusion in infants with
severe MAS has also been shown to reduce the need for extracorporeal
membrane oxygenation.
• · The development of active synthetic surfactants is very complicated.
• · Anti-inflammatory drugs may be given to diminish the adverse action
of products of meconium-induced inflammation on both endogenous
and exogenously delivered surfactant.
• · Inhaled nitric oxide can be useful in the management of pulmonary
hypertension associated with MAS.It is thought to act by relaxing
smooth muscles in the pulmonary vessels, causing vasodilatation, as
well as promoting bronchodilation.
• · Enteral sildenafil may be used for the treatment of persistent
pulmonary hypertension resulting from MAS.
• · Extracorporeal membrane oxygenation (ECMO) may be needed in
those babies who deteriorate.
• · Steroids - inhaled or systemic - have been used to good effect in some
studies. Budesonide has been shown to improve the effects of
exogenous surfactant in experimental MAS.[9] ress


Reference Patient.info


Q.1276 A pt is at term and labor. The head has been delivered and you suspect
shoulder dystocia. What isthe single most appropriate action?
a. C-section
b. Episiotomy
c. External rotation
d. Fetal scalp blood sample
e. Instrumental delivery


Clincher(s)
A

49
OBS GYN-System Wise 1700-by Sush and Team. 2016
Susmita, Asad, Manu, Saima, Zohaib, Savia, Shanu, Mona, Manisha, Sitara, Samreena, Sami and Komal

B
C
D

KEY B
Additional RCOG Guidelines
Information
Elective caesarean section should be considered to reduce the potential
morbidity for pregnancies
complicated by pre-existing or gestational diabetes, regardless of treatment,
with an estimated fetal
weight of greater than 4.5 kg.

Get help. In addition to a senior obstetrician, an anaesthetist and paediatrician
should be called.
•Stop the
mother pushing. This may make impaction of the shoulders worse.
•McRoberts' manoeuvre
-the patient hyperflexes her hips so they are against her
abdomen. Mothers in labour may not have enough energy to do this by
themselves and may need the assistance of others in the roomwhich is usually
the case. Posterolateral pressure is applied suprapubically with traction on the
fetal head. This is the most effective procedure and should be performed first
(success rates are up to 90%).
•If this fails, an episiotomy
may be needed to facilitate the obstetrician trying
secondline manoeuvres -but the need for a caesarean section should be
considered.\

Rubin's manoeuvre

press on the posterior fetal shoulder, thereby creating more
space to allow the anterior shoulder to be delivered.

•Woods' screw manoeuvre
-turning the anterior shoulder to the posterior position.
•If these fail then delivery of the posterior shoulder may help.
•However, at all times the need for a caesarean section should be considered
and should not be delayed.
NB: fundal pressure should NOT be applied.





50
OBS GYN-System Wise 1700-by Sush and Team. 2016
Susmita, Asad, Manu, Saima, Zohaib, Savia, Shanu, Mona, Manisha, Sitara, Samreena, Sami and Komal

Reference RCOG guidelines

Q.1277 A 29yo female at 28wks GA presents to you with complains of hard stools and
constipation for last 2wks. CTG shows fetal tachycardia. What is the single
most appropriate tx?
a. Oral laxatives (increases hemorrhages etc complications)
b. Fiber diet
c. Phosphate enema (avoided for 28 preg)
d. Lactulose
e. Reassure
b. Fiber diet


Clincher(s)
A
B
C
D

KEY B
Additional
Information Constipation tends to occur as gut motility decreases. Adequate oral fl uids
and a high-fibre diet help combat it. Avoid stimulant laxatives—they increase
uterine activity in some women. Increased venous distensibility and pelvic
congestion predispose to haemorrhoids (if they prolapse, rest the mother
head down, apply ice packs and replace them) and varicose veins. Resting
with feet up and properly worn elastic stockings help.

Reference Ohcs page 17

Q.1278 A 16yo girl presents with heavy bleeding. What is the most appropriate initial
inv?
a. Endometrial sampling
b. Transvaginal US
c. Hysteroscopy
d. Pelvic US
e. Exam under anesthesia
d. Pelvic US


Clincher(s)
A
B
C
D Non invasive..to check for a miscarriage

51
OBS GYN-System Wise 1700-by Sush and Team. 2016
Susmita, Asad, Manu, Saima, Zohaib, Savia, Shanu, Mona, Manisha, Sitara, Samreena, Sami and Komal

E
KEY D
Additional
Information Abnormal uterine bleeding
1. Irregular menstruation, excessive menses (i.e., menorrhagia), or increased
duration of menses that may be the result of a variety of causes (e.g., uterine
fibroids, cancer, hypothalamic - pituitary dysfunction, bleeding diatheses [e.g.,
von
Willebrand disease], threatened abortion, molar pregnancy, ectopic
pregnancy)
2. H/P
a. Uterine bleeding that does not follow usual menstrual cycle or occurs in
postmenopausal women
b. Menses with _ 24-day or _ 35-day intervals, lasting _ 7 days, or blood loss
_ 80 mL are considered abnormal.
c. Associated symptoms (e.g., fever, abdominal pain, vaginal discharge, acne,
changes in bowel or bladder function), family history, history of medical
conditions
useful to making diagnosis
d. Visualization of bleeding site (e.g., cervix, vagina, anus, vulva), palpation of
pelvic masses important
3. Labs
a. _ -hCG used to rule out pregnancy
b. Complete blood count (CBC), coagulation studies, TSH, FSH, and LH are used
to rule out anemia, coagulopathy, and endocrine abnormalities.
c. Papanicolaou (Pap) smear and endometrial biopsy (possibly obtained during
dilation and curettage [D&C]) used to rule out cancer
d. Testing for STDs used to rule out infection
4. Radiology _ US may detect uterine lesions; hysteroscopy frequently
indicated to visualize lesions and perform D&C
5. Treatment
a. Treat underlying disorder (e.g., coagulopathies, thyroid disease, infection).
b. OCPs can be used for cycle irregularity.
c. Endometrial ablation may be performed for severe or recurrent bleeding.

Reference


Q.1279 A woman who is 7wks pregnant presents with excessive and severe vomiting
and put on IVfluids and anti -emetic (ondansteron). She is complaining of
severe headache and can’t take oral fluids. What is the most appropriate
management?
a. Termination of pregnancy
b. TPN (total per enteral nutrition)
c. Feeds via NGT
d. P6 acupressure

52
OBS GYN-System Wise 1700-by Sush and Team. 2016
Susmita, Asad, Manu, Saima, Zohaib, Savia, Shanu, Mona, Manisha, Sitara, Samreena, Sami and Komal

e. IV hydrocortisone


Clincher(s) GA 7weeks,excessive vomiting
A
B
C
D

KEY E
Additional Hyperemesis Gravidarum:
Information Treatment Admit to hospital. Give thromboprophylaxis (eg enoxaparin
40mg/24h SC) and anti-embolic stockings. Spend time optimizing psychological
wellbeing. Is she worried about how her other children are being cared for?
Most settle with due care and attention. If not too severe it may settle
with rest, ginger, pyridoxine, dry bland food, and carbonated drinks. Routine
thiamine supplementation is wise for all women admitted (eg thiamine
25–50mg/8h PO) or if IV required 100mg diluted in 100mL normal saline given
over 60min, repeated at weekly intervals. This is to prevent development of
Wernicke’s encephalopathy (see OHCM p707)—which is then associated with
40% fetal loss. Correct dehydration with IV infusion (eg with normal saline
infusion with potassium added to each bag as guided by U&E). Beware rapid
reversal of hyponatraemia which can cause fatal central pontine myelinosis. 54
If condition does not improve after rehydration anti-emetics may be needed
eg cyclizine 50mg/8h PO/IM or IV. Other recognized anti-emetics used:
metoclopramide,
prochloperazine, chlorpromazine, domperidone, ondansetron. 55
Phenothiazines can cause drowsiness, extrapyramidal side eff ects, and
oculogyric
crisis. Those resistant to conventional treatments may respond to
steroid treatment, eg hydrocortisone 100mg twice daily followed by 40mg
prednisolone/24h tapering down. Prednisolone can then usually be reduced to
2.5–10mg/24h by 20 weeks' gestation. If it is needed long-term screen for UTI
and gestational diabetes. Prednisolone is metabolized by the placenta, fetal
blood levels are low and adverse fetal eff ects have not been reported.
Parenteral nutrition may, very rarely, be needed—OHCM p574. If nutritional
support is required both nasojejunal tube feeding and percutaneous
endoscopic
gastrostomy have been successfully used. 56 Parenteral nutrition has been
found to be associated with serious complications (eg line sepsis). Get a
dietician’s help.

Reference Ohcs page 18

Q.1280 A young lady with primary amenorrhea has normal LH, FSH, estradiol and
prolactin. Choose the single most likely dx?

53
OBS GYN-System Wise 1700-by Sush and Team. 2016
Susmita, Asad, Manu, Saima, Zohaib, Savia, Shanu, Mona, Manisha, Sitara, Samreena, Sami and Komal

a. PCOS
b. POF
c. Absent uterus
d. Absent ovaries
e. Turner’s syndrom


Clincher(s) Primary amenorrhea,normal hormones
A PCOS raised LH:FSH ratio, Prolactin may be normal or mildly elevated.
Testosterone may be normal or mildly elevated

B
C An absent uterus or a rudimentary uterus with absent endometrium is rare.
They present with primary amenorrhoea.
D
E Turner syndrome should be considered in any girl with short stature or primary
amenorrhoea. LH and FSH may be elevated. (rudimentary overies)
KEY C
Additional Thin, rudimentary ‘streak’ ovaries are found in Turner’s syndrome
Information (p655). Ovaries are absent in testicular feminization syndrome, but primitive
testes are present (p134). Remnants of developmental tissue (eg the Wolffian
system) may result in cysts around the ovary and in the broad ligament.
Mayer-Rokitansky-Küster-Hauser (MRKH) syndrome is a disorder that occurs in
females and mainly affects the reproductive system. This condition causes the
vagina and uterus to be underdeveloped or absent.
Reference Ohcs 246



Q.1281 An obese lady presents with primary amenorrhea. She has high LH, normal FSH
and slightly high prolactin levels. Choose the single most likely dx?
a. PCOS (2:1 LH: FSH, but in normal : 1:2)
b. POF
c. Hypothyroidism (menorrhagia)
d. Pregnancy
e. Primary obesity

Clincher(s) Obese, amenorrhea. high LH, normal FSH and slightly high prolactin levels.
A Typical presesntation of PCOS
B Cessation of menstruation at 38years or below 38yr of age is termed as
premature ovarian failiure
C Hypothroidism presents with heavy menstruation and later light menstruation
alongwith other signs and symptoms.
D

KEY A

54
OBS GYN-System Wise 1700-by Sush and Team. 2016
Susmita, Asad, Manu, Saima, Zohaib, Savia, Shanu, Mona, Manisha, Sitara, Samreena, Sami and Komal

Additional
Information




Reference ohcs


Q.1282 A 38yo lady presents with amenorrhea has very high LH and FSH levels, normal
prolactin and low estradiol. Choose the single most likely dx?
a. PCOS

55
OBS GYN-System Wise 1700-by Sush and Team. 2016
Susmita, Asad, Manu, Saima, Zohaib, Savia, Shanu, Mona, Manisha, Sitara, Samreena, Sami and Komal

b. POF
c. Hypothyroidism
d. Pregnancy
e. Menopause

Clincher(s) 38years old,amenorrhea,very high LH,FSH,low estradiol.normal prolactin
A Prolactin usually raised,high LH:FSH
B
C
D Menopause after 40 years of age
KEY B
Additional Premature ovarian failiure:
information The age of 40yrs is used frequently as an arbitrary
limit below which the menopause is said to be premature. It affects 1% of
women younger than 40yrs and 0.1% of those under 30yrs. In most cases no
cause is found.


Presentation and assessment
The most common presentation is secondary amenorrhoea or oligo-
menorrhoea (which may not necessarily be accompanied by hot
flushes).
Co-existing disease may be detected, particularly:
• hypothyroidism
• Addison’s disease
• diabetes mellitus
• any chromosome abnormalities (especially those who have not achieved
successful pregnancy).
The diagnostic usefulness of ovarian biopsy outside the research setting has
yet to be proved.

56
OBS GYN-System Wise 1700-by Sush and Team. 2016
Susmita, Asad, Manu, Saima, Zohaib, Savia, Shanu, Mona, Manisha, Sitara, Samreena, Sami and Komal






Q:432 432. A 35yo woman has had bruising and petechiae for a week. She has also
had recent menorrhagia but is otherwise well. Blood: Hgb=11.1, WBC=6.3,
Plt=14K. What is the single most likely dx?
a. Acute leukemia
b. Aplastic anemia
c. HIV infection
d. ITP
e. SLE

Clincher(s) BRUISING , PETECHIAE, MENORRHAGIA, LOW PLATELETS.
A Acute Leukemia- WBC normal
B Aplastic Anemia- Only Platelets low, other two series are normal (To define
aplastic anaemia there must be at least two of the following (i) haemoglobin
<100 g/l (ii) platelet count <50 · 109/l (iii) neutrophil count <1.5 (International
Agranulocytosis and Aplastic Anaemia Study Group, 1987).
C HIV Infection- Aplastic Anemia
D ITP- Only Thrombocytopenia (only platelets are destroyed, body gets used to
low platelets)
E SLE- Only thrombocytopenia (no other clinchers; SLE may cause ITP, platelets
won’t be so low)
KEY D
Additional Gestational thrombocytopenia is the most common cause of
Information thrombocytopenia in pregnancy, affecting 5% of all pregnancies and
accounting for more than 75% of cases of pregnancyassociated
thrombocytopenia [4,6,10,25,83-85]. This disorder usually develops in the late
second or third trimester and generally affects women with no prior history of
ITP or autoimmune disease. It is characterized by mild thrombocytopenia not
accompanied by abnormal physical findings, such as hypertension, that would

57
OBS GYN-System Wise 1700-by Sush and Team. 2016
Susmita, Asad, Manu, Saima, Zohaib, Savia, Shanu, Mona, Manisha, Sitara, Samreena, Sami and Komal

implicate other causes of thrombocytopenia. While no absolute value of the
platelet count below which gestational thrombocytopenia can be excluded has
been defined, the American Society of Hematology (ASH) as well as the British
Committee for Standard in Hematology General Hematology Task Force (BCSH)
suggest that at platelet counts below 70,000/μl or 80,000/μl, respectively,
gestational thrombocytopenia becomes increasingly less likely and other
causes of thrombocytopenia should be more strongly considered [36,37].

Stavrou and McCrae Page 6

Hematol Oncol Clin North Am. Author manuscript; available in PMC 2010
December 1.

NIH-PA Author ManuscriptNIH-PA Author Manuscript



http://www.ncbi.nlm.nih.gov/pmc/articles/PMC2784425/pdf/nihms142639.pd
f
Reference
Dr Khalid/Rabia KEY- D
*Patient only presents with petechiae and menorrhagia, but is othwerwise
well. Hence all other options are unlikely. Also aplastic anaemia will result in
pancytopenia, but WBCs and Hb is normal here.
*What is ITP? Immune thrombocytopenic purpura.

-Autoimmune, destruction or decreased reduction of platelets. Hence
decreased platelets.

-Classified into primary (isolated) or secondary (in association with other
disease).

>Secondary ITP causes:
_Autoimmune disorders (Antiphospholipid AB syndrome, SLE)
_Viral ( CMV, VZ, HepC, HIV)<-- Most common in children at around 6 years.
_H.pylori
_Drugs

>Presentation:-
Petichae, epistaxis, hematuria or menorrhagia. Rarely intracranial bleeds.

>Investigations:

FBC, peripheral blood smear. Screen for HIV, HepC and other underlying cause.

Treatment:
-Only if symptomatic.
-Avoid NSAIDs and aspirin.
-First line tt is Prednisolone for 3 weeks, then taper off, IVIG and give IV anti-D

58
OBS GYN-System Wise 1700-by Sush and Team. 2016
Susmita, Asad, Manu, Saima, Zohaib, Savia, Shanu, Mona, Manisha, Sitara, Samreena, Sami and Komal

in Rh +ve and non-splenectomised people.
-Second line Splenectomy.
Complications- infection, bleeding, thrombosis, relapse.

-Refractory ITP- Romiplostim and Eltrombopag (thrombopoetin receptor
agonists)



Q:466 466. A 32yo woman of 38wks gestation complains of feeling unwell with fever,
rigors and abdominal pains. The pain was initially located in the abdomen and
was a/w urinary freq and dysuria. The pain has now become more generalized
specifically radiating to the right loin. She says that she has felt occasional
uterine tightening. CTG is reassuring. Select the most likely dx?
a. Acute fatty liver of pregnancy (LFT not mentioned)
b. Acute pyelonephritis
c. Roung ligament stretching (no fever)
d. Cholecystitis (shoulder)
e. UTI

Clincher(s) Fever, rigor, abdominal pain, urinary frequency.Dysuria, pain radiating to
loin. CTG normal
A Acute Fatty liver of pregnancy- No LFT mentioned
B Acute pyelonephritis- suggestive
C Round Ligament stretching- not associated with fever etc. Common in preg
women, radiation to legs
D Cholecystitis- radiation to right shoulder, pain RUQ
E UTI, will not cause radiation
KEY B
Additional
Information Pelvic joint pain (due to round lig stretching)

Some women develop pelvic pain in pregnancy. This is sometimes called


pregnancy-related pelvic girdle pain (PPGP) or symphysis pubis dysfunction
(SPD).

Symptoms of PPGP

Cause- misalignment or stiffness of pelvic joints at either the back or front of


pelvis.

Symptom-

cause severe pain around pelvic area pain over the pubic bone at the front in
the centre.

59
OBS GYN-System Wise 1700-by Sush and Team. 2016
Susmita, Asad, Manu, Saima, Zohaib, Savia, Shanu, Mona, Manisha, Sitara, Samreena, Sami and Komal

pain across one or both sides of your lower back

pain in the area between your vagina and anus (perineum)

Pain can also radiate to your thighs, and some women feel or hear a clicking or
grinding in the pelvic area. The pain can be most noticeable when you are:

• walking
• going upstairs
• standing on one leg (for example when you’re getting dressed or going
upstairs)
• turning over in bed

It can also be difficult to move legs apart, for example when getting out of a
car.

Who gets pelvic pain in pregnancy?

one in five pregnant women. linked to a number of issues, including previous


damage to the pelvis, pelvic joints moving unevenly, and the weight or position
of the baby.

Factors that may make a woman more likely to develop PPGP include:

• a history of lower back or pelvic girdle pain


• previous injury to the pelvis, for example from a fall or accident
• having PPGP in a previous pregnancy
• a hard physical job

When to get help for pelvic joint pain

diagnosed as early as possible can help to keep the pain to a minimum and
avoid long-term discomfort.

Treatment by a physiotherapist usually involves gently pressing on or moving


the affected joint, which helps it work normally again.

Treatment

Physiotherapy aims to relieve or ease pain, improve muscle function and


improve your pelvic joint position and stability, and may include:

• manual therapy to make sure the joints of your pelvis, hip and spine
move normally
• exercises to strengthen your pelvic floor, stomach, back and hip
muscles
• exercises in water

60
OBS GYN-System Wise 1700-by Sush and Team. 2016
Susmita, Asad, Manu, Saima, Zohaib, Savia, Shanu, Mona, Manisha, Sitara, Samreena, Sami and Komal

• advice and suggestions including positions for labour and birth, looking
after your baby, and positions for sex
• pain relief, such as TENS
• equipment if necessary, such as crutches or pelvic support belts

Coping with PPGP

• Be as active as possible within your pain limits, and avoid activities that
make the pain worse.
• Rest when you can.
• Get help with household chores from your partner, family and friends.
• Wear flat, supportive shoes.
• Sit down to get dressed – for example don’t stand on one leg when
putting on jeans.
• Keep your knees together when getting in and out of the car – a plastic
bag on the seat can help you swivel.
• Sleep in a comfortable position, for example on your side with a pillow
between your legs.
• Try different ways of turning over in bed, for example turning over with
your knees together and squeezing your buttocks.
• Take the stairs one at a time, or go upstairs backwards or on your
bottom.
• If you’re using crutches, have a small backpack to carry things in.
• If you want to have sex, consider different positions such as kneeling on
all fours.

ACPWH suggests that you avoid:

• standing on one leg


• bending and twisting to lift, or carrying a baby on one hip
• crossing your legs
• sitting on the floor, or sitting twisted
• sitting or standing for long periods
• lifting heavy weights, such as shopping bags, wet washing or a toddler
• vacuuming
• pushing heavy objects, such as a supermarket trolley
• carrying anything in only one hand (try using a small backpack)


Reference http://www.nhsdirect.wales.nhs.uk/doityourself/pregnancy/CommonProblem
s/?print=1
Dr Khalid/Rabia KEY- B
This is a case of UTI followed by ascending infection leading to pyelonephritis.
Fever, rigors and abdominal pain are typical symptoms.


61
OBS GYN-System Wise 1700-by Sush and Team. 2016
Susmita, Asad, Manu, Saima, Zohaib, Savia, Shanu, Mona, Manisha, Sitara, Samreena, Sami and Komal

Q:478 478. A 20yo pregnant 32wks by date presents to the antenatal clinic with hx of
painless vaginal bleeding after intercourse. Exam: P/A – soft and relaxed,
uterus=dates, CTG=reactive (normal- if ace or decc only- then abnormal).
Choose the single most likely dx?
a. Abruption of placenta 2nd to pre-eclampsia
b. Antepartum hemorrhage
c. Placenta previa
d. Preterm labor
e. Placenta percreta – (Prev

Clincher(s) Third Trimester, painless vaginal bleeding, soft abdomen, CTG reactive
A Abruption of placenta 2nd to pre-eclampsia- extreme Painful
B Antepartum haemorrhage-
C Placenta previa (painless vag bleeding in 3rd trimester)
D Preterm labor
E Placenta percreta- post pregnancy/delivery
KEY C
Additional Antepartum haemorrhage: determining cause
Information

Antepartum haemorrhage is defined as bleeding from the genital tract after 24
weeks pregnancy, prior to delivery of the fetus

Distinguishing placental abruption from praevia

Placental abruption

• shock out of keeping with visible loss


• pain constant
• tender, tense uterus*
• normal lie and presentation
• fetal heart: absent/distressed
• coagulation problems
• beware pre-eclampsia, DIC, anuria


Placenta praevia

• shock in proportion to visible loss


• no pain
• uterus not tender*
• lie and presentation may be abnormal
• fetal heart usually normal
• coagulation problems rare
• small bleeds before large

62
OBS GYN-System Wise 1700-by Sush and Team. 2016
Susmita, Asad, Manu, Saima, Zohaib, Savia, Shanu, Mona, Manisha, Sitara, Samreena, Sami and Komal

*vaginal examination should not be performed in primary care for suspected
antepartum haemorrhage - women with placenta praevia may haemorrhage

Reference
Dr Khalid/Rabia KEY- C
Clincher- painless bleeding, typical presentation of placenta previa. Uterus is
soft and relaxed and there’s no pain, so we rule out placental abruption


Q:483 483. A 65yo presents with dyspareunia after sex. She in menopause. She
complains of bleeding after sex. What is the most probably dx?
a. Cervical ca
b. Endometrial ca
c. Ovarian ca
d. Breast ca
e. Vaginal ca

Clincher(s) Post menopausal.
A Cervical ca- post coital bleeding, age younger
B Endometrial ca-postmenopausal bleeding
C Ovarian ca- abdominal distension
D Breast ca- breast lump
E Vaginal ca- contact bleeding
KEY B
Additional (anything to post meno wome=endo ca)
Information
Reference
Dr Khalid/Rabia KEY- B
RULE- Postmenopausal bleeding, be it post coital or not, is Endometrial cancer
unless proven otherwise.
Clinchers- Post menopausal, and age 65.
Cervical cancer is common in women aged 25-34 years, while 90% of women
with endometrial cancer are over 50 years of age. [Source- patient.co.uk]



Q:486 486. A 24yo primigravida who is 30wk pregnant presents to the labor ward
with a hx of constant abdominal pain for the last few hours. She also gives a hx
of having lost a cupful of fresh blood per vagina before the pain started.
Abdominal exam: irritable uterus, CTG=reactive (normal). Choose the single
most likely dx?
a. Abruption of placenta 2nd to pre-eclampsia (CTG will be abnormal, blood is
usually not fresh)
b. Antepartum haemorrhage (broader term; e.g due to ectropion, sex, polyP)
c. Placenta previa
d. Vasa previa (abn vessels under fetus, near cervix ruptured- fresh but there

63
OBS GYN-System Wise 1700-by Sush and Team. 2016
Susmita, Asad, Manu, Saima, Zohaib, Savia, Shanu, Mona, Manisha, Sitara, Samreena, Sami and Komal

will fetal distress)
e. Revealed haemorrhage- part abruotion hgs- CTG abnormal)

Clincher(s) Third trimester, constant abdominal pain, bleeding , reactive CTG
A Abruption of placenta 2nd to pre-eclampsia- no h/oBP raised, proteinuria
B Antepartum haemorrhage- Broader term
C Placenta previa- painless
D Vasa previa- (painless) fetal bradycardia ( CTG can not be normal) Vasa
praevia (vasa previa AE) is an obstetric complication in which fetal blood
vessels cross or run near the external orifice of the uterus. These vessels are at
risk of rupture when the supporting membranes rupture, as they are
unsupported by the umbilical cord or placental tissue.The term "vasa previa" is
derived from the Latin; "vasa" means vessel and "previa" comes from "pre"
meaning "before" and "via" meaning "way". In other words, vessels lie before
the baby in the birth canal and in the way. [1]
E Revealed haemorrhage
KEY B
Additional Placental abruption
Information
• Presents with sudden abdominal pain in the third trimester.
• On examination the mother can be seen to be in extreme pain and cold to
touch.
• Bleeding is present in 80% of cases.
• Absence of visible bleeding does not rule out this diagnosis.
• Risk factors include: maternal hypertension (common), cocaine, trauma,
uterine overdistension, tobacco and previous placental abruption.

Antepartum Haemorrhage:

• Defined as bleeding after 24 weeks gestation.


Common causes:

• Placental abruption
• Placenta praevia

IMP:

Placental abruption Placenta Praevia
Painful Painless
Dark bleeding (may not
Profuse red blood
bleed)
Tender 'woody' (hard)
Rarely tender
uterus
Normal USS Low placenta on

64
OBS GYN-System Wise 1700-by Sush and Team. 2016
Susmita, Asad, Manu, Saima, Zohaib, Savia, Shanu, Mona, Manisha, Sitara, Samreena, Sami and Komal

USS

Other causes:

• Uterine rupture
• Vasa praevia
• Genital tract pathology (e.g. cervical cancer, cervical polyps, cervical
ectropion, vaginal laceration; consider all other causes of non-cyclical per
vaginal bleeding)


Reference
Dr Khalid/Rabia KEY- B
Presentation indicates abruption of the placenta, but not confirmed yet.
Generally bleeding during this time is given a general diagnosis of antepartum
haemorrhage. There is no history or features suggestive of of hypertension or
pre-eclampsia so A is not the choice. Abruption can be either concealed or
revealed abruption.
Placenta praevia is painless bleeding.
Vasa Praevia- vessels before fetus, can rupture and cause fetal demise


Q:489 489. A 42yo female who is obese comes with severe upper abdominal pain
with a temp=37.8C. She has 5 children. What is the most probable dx?
a. Ectopic pregnancy
b. Ovarian torsion
c. Hepatitis
d. Biliary colic
e. Cholecystitis

Clincher(s) Fertile woman with five children, abdominal pain , no other history exc fever
A Ectopic Pregnancy- no sp hx, lower abdominal pain
B Ovarian torsion- Lower abdominal pain
C Hepatitis-
D Biliary colic
E Cholecystitis (5 Fs)
KEY E
Additional
Information
Reference
Dr Khalid/Rabia KEY- E
> This is cholecystitis, or non-alcoholic steatohepatitis. The 5 Fs of cholecystitis
are- Fat
Female
Fair
Forty

65
OBS GYN-System Wise 1700-by Sush and Team. 2016
Susmita, Asad, Manu, Saima, Zohaib, Savia, Shanu, Mona, Manisha, Sitara, Samreena, Sami and Komal

Fertile.
> Ovarian torsion and ectopic pregnancy will have lower abdominal pain.



Q:491 A primiparous woman with no prv infection with herpes zoster is 18wk
pregnant. She had recent contact with a young 21yo pt having widespread
chicken pox. What is the most suitable management for the pregnant lady?
a. Acyclovir PO (if with 24 hrs of presenting with rashes)
b. Acyclovir IV +IVIG
c. Acyclovir IV
d. Reassure
e. IVIG

Clincher(s)
A Acyclovir PO
B Acyclovir IV+IVIG
C Acyclovir IV
D Reassure
E IVIG (within 10 days presentations of contact)
KEY E
Additional Varicella in pregnancy
Information Pneumonitis and encephalitis are no commoner in
pregnancy, despite pregnancy being an immunocompromised state (1 in 400
and 1 in 1000, respectively).
Infection in the 1st 20 weeks (esp. 13–20 weeks) may cause varicella zoster
virus (VZV) fetopathy in 2%. 345

Signs of vzv fetopathy
are variable, eg cerebral cortical atrophy and cerebellar hypoplasia, manifested
by microcephaly, convulsions and IQ ; limb hypoplasia; rudimentary digits
} pigmented scars. Maternal shingles is not a cause.

If the mother is aff ected from 1 week before to 4 weeks after birth, babies
may suff er severe chickenpox.
Give the baby zoster immunglobulin 250mg IM at birth; if aff ected, isolate
from other babies, and give aciclovir.

Infection is preventable by pre-pregnancy vaccination with live varicella
vaccine, 346 but testing for antibodies pre-conceptually is expensive, and
costeff ectiveness depends on local rates of seronegativity.

~80% of those who cannot recall any previous chickenpox are, in fact,
immune. 347
Varicella zoster globulin prevents infection in 50% of susceptible contacts,
eg 1000mg IM (adults). Infection in pregnancy merits aciclovir (it’s probably

66
OBS GYN-System Wise 1700-by Sush and Team. 2016
Susmita, Asad, Manu, Saima, Zohaib, Savia, Shanu, Mona, Manisha, Sitara, Samreena, Sami and Komal

OK for the fetus). Chickenpox at birth is a problem. Barrier nursing mothers
causes distress and is of unproven value. Infant mortality: up to 20%. 348

161 ohcs
Oxford handbook of gynecology




A 24-year-old woman who is 18 weeks pregnant presents for review Earlier on
in the morning she came into contact with a child who has chickenpox. She is
unsure if she had the condition herself as a child. What is the most appropriate
action?

Advise her to present within 24 hours of the


rash developing for consideration of IV


aciclovir

Reassure her that there is no risk of fetal


complications at this point in pregnancy

Give varicella immunoglobulin


Check varicella antibodies


Prescribe oral acyclovir


Chickenpox exposure in pregnancy - first step is to check antibodies



If there is any doubt about the mother previously having chickenpox maternal
blood should be checked for varicella antibodies
Please rate this question:

67
OBS GYN-System Wise 1700-by Sush and Team. 2016
Susmita, Asad, Manu, Saima, Zohaib, Savia, Shanu, Mona, Manisha, Sitara, Samreena, Sami and Komal



Discuss and give feedback
Next question

Chickenpox exposure in pregnancy

Chickenpox is caused by primary infection with varicella zoster virus. Shingles


is reactivation of dormant virus in dorsal root ganglion. In pregnancy there is a
risk to both the mother and also the fetus, a syndrome now termed fetal
varicella syndrome

Risks to the mother

• 5 times greater risk of pneumonitis


Fetal varicella syndrome (FVS)

• risk of FVS following maternal varicella exposure is around 1% if occurs


before 20 weeks gestation
• studies have shown a very small number of cases occurring between
20-28 weeks gestation and none following 28 weeks
• features of FVS include skin scarring, eye defects (microphthalmia),
limb hypoplasia, microcephaly and learning disabilities


Other risks to the fetus

• shingles in infancy: 1-2% risk if maternal exposure in the second or


third trimester
• severe neonatal varicella: if mother develops rash between 5 days
before and 2 days after birth there is a risk of neonatal varicella, which
may be fatal to the newborn child in around 20% of cases


Management of chickenpox exposure

• if there is any doubt about the mother previously having chickenpox


maternal blood should be urgently checked for varicella antibodies
• if the pregnant women is not immune to varicella she should be given
varicella zoster immunoglobulin (VZIG) as soon as possible. RCOG and
Greenbook guidelines suggest VZIG is effective up to 10 days post
exposure
• consensus guidelines suggest oral aciclovir should be given if pregnant

68
OBS GYN-System Wise 1700-by Sush and Team. 2016
Susmita, Asad, Manu, Saima, Zohaib, Savia, Shanu, Mona, Manisha, Sitara, Samreena, Sami and Komal

women with chickenpox present within 24 hours of onset of the rash


Reference If the pregnant woman is not immune to VZV and she has had a significant
exposure, she should be offered varicella-zoster immunoglobulin (VZIG) as
soon as possible. VZIG is effective when given up to 10 days after contact (in
the case of continuous exposures, this is defined as 10 days from the
appearance of the rash in the index case).
Non-immune pregnant women who have been exposed to chickenpox should
be managed as potentially infectious from 8–28 days after exposure if they
receive VZIG and from 8–21 days after exposure if they do not receive VZIG.
When supplies are limited, issues to pregnant women may be restricted and
clinicians are advised to establish the availability of VZIG before offering it to
pregnant women.
https://www.rcog.org.uk/globalassets/documents/guidelines/gtg_13.pdf
Dr Khalid/Rabia KEY- E
*If the pregnant woman is not immune to VZV and she has had a significant
exposure, she should be offered varicella-zoster immunoglobulin (VZIG) as
soon as possible. VZIG is effective when given up to 10 days after contact (in
the case of continuous exposures, this is defined as 10 days from the
appearance of the rash in the index case).
*If she had no previous infection and develops a rash (got infected) and comes
within 24 hour of development of rash- acyclovir is given. [MRCOG Guideline].




Q:519 519. A 39yo woman has not had her period for 10months. She feels well but is
anxious as her mother had an early menopause. Choose the single most
appropriate initial inv?
a. Serum estradiol conc.
b. Serum FSH/LH (to rule out premature ovarian failure)
c. Serum progesterone conc.
d. None
e. Transvaginal US

Clincher(s)
A
B
C
D
E
KEY B
Additional
Information
Reference

69
OBS GYN-System Wise 1700-by Sush and Team. 2016
Susmita, Asad, Manu, Saima, Zohaib, Savia, Shanu, Mona, Manisha, Sitara, Samreena, Sami and Komal

Dr Khalid/Rabia Serum FSH/LH
Premature Menopause (ovarian faililure)shoud be ruled out.So FSH and
LH(very high)
Premature Ovarian Failure (menopause before 40 years of age).
• climacteric symptoms: hot flushes, night sweats
• infertility
• secondary amenorrhoea
• raised FSH, LH levels
serum estradiol reflects primarily the activity of the ovaries. useful in the
detection of baseline estrogen in women with amenorrhea or menstrual
dysfunction, and to detect the state of hypoestrogenicity and menopause.
Serum progesterone: indicates if failure to ovulate
7 days prior to expected next period



Q:527 527. A 23yo woman with painless vaginal bleeding at 36wks pregnancy
otherwise seems to be normal. What should be done next?
a. Vaginal US (not done during in late preg bleeding- for placenta previa)
b. Abdominal US
c. Vaginal exam
d. Reassurance

Clincher(s)
A Vaginal US- contraindicated in vaginal bleeding
B Abdominal US- assess fetal being and check placenta previa
C Vaginal Examination
D Reassurance
E
KEY B
Additional Vaginal usg is contraindicated in PV bleeding,if uterus is abdominal. Trans Pv
Information usg done in early preganancy. P/V in previa will cause further bleeding
Reference
Dr Khalid/Rabia b. Abdominal US
to assess fetal being and check placenta previa
vaginal US is more accurate but not initial when bleeding.
No PV until no PP


Placenta previa
Painless bleeding starting after the 28th week (although spotting may occur
earlier) is usually the main sign.

Acute bleeding

70
OBS GYN-System Wise 1700-by Sush and Team. 2016
Susmita, Asad, Manu, Saima, Zohaib, Savia, Shanu, Mona, Manisha, Sitara, Samreena, Sami and Komal

Admit the patient to hospital.

DO NOT PERFORM A VAGINAL EXAMINATION, as this may start torrential


bleeding in the presence of placenta praevia.

• Blood loss is assessed and cross-matched for possible transfusion.


• Resuscitation if indicated; the mother is the priority and should be
stabilised prior to any assessment of the fetus.
• Appropriate surgical intervention may be required:
o In severe bleeding the baby is delivered urgently
whatever its gestational age.
o Hysterectomy should also be considered in severe
cases.
• If immediate delivery is not likely, maternal steroids may be
indicated in order to promote fetal lung development and reduce
the risk of respiratory distress syndrome and intraventricular
haemorrhage.




Q:533 533. A 24yo woman has 8wk amenorrhea, right sided pelvic pain and vaginal
bleeding. She is apyrexial. Peritonism is elicited in the RIF. Vaginal exam
reveals right sided cervical excitation. (pt jumps with pain-due to fornix
tenderness- tp diff PID/ectopic and acute appendicitis)
What is the most probable dx?
a. Ectopic pregnancy
b. Salpingitis
c. Endometriosis
d. Ovarian torsion
e. Ovarian tumor

Clincher(s) 8 weeks amenorrhea, right sided pelvic pain, vaginal bleeding, apyrexial,
pertonism, cervical excitation, RIF pains
A Ectopic Pregnancy- Uninlateral Pain, amenorrohea, vaginal bleeding, apyrexial
B Salpingitis- No fever, amenorrhea..
C Endometriosis- recurrent
D Ovarian Torsion
E Ovarian Tumor
KEY A
Additional
Information
Reference
Dr Khalid/Rabia a. Ectopic pregnancy

71
OBS GYN-System Wise 1700-by Sush and Team. 2016
Susmita, Asad, Manu, Saima, Zohaib, Savia, Shanu, Mona, Manisha, Sitara, Samreena, Sami and Komal

amenorrea, pelvic pain n vaginal bleeding clinical triad of ectopic pregnancy
cervical excitation is sign in ectopic pregnancy and PID
cervical motion tenderness which is when bilateral we suspect PID n when
unilateral ectopic most likely
since the pt is apyrexial this rules out PID

The most common symptoms are
• Abdominal pain.
• Pelvic pain.
• Amenorrhoea or missed period.

Examination
• There may be some tenderness in the suprapubic region.
• Peritonism and signs of an acute abdomen may occur.
• Women with a positive pregnancy test and any of the following need to
be referred immediately to hospital:
o Pain and abdominal tenderness.
o Pelvic tenderness.
o Cervical motion tenderness.
• Vaginal bleeding (with or without clots).
The most accurate method to detect a tubal pregnancy is transvaginal
ultrasound.
Human chorionic gonadotrophin (hCG) levels are performed in women with
pregnancy of unknown location who are clinically stable.




Q:535 535. A 26yo woman with regular menses and her 28yo partner comes to the
GP surgery complaining of primary infertility for 2yrs. What would be the
single best investigation to see whether she is ovulating or not?
a. Basal body temp estimation
b. Cervical smear
c. Day2 LH and FSH
d. Day21 progesterone
e. Endometrial biopsy

Clincher(s) Regular menses, infertility for 2 years, single best investigation
A Basal Body Temp Estimation
B Cervical Smear
C Day 2 LH/FSH
D Day 21 Progesterone
E Endometrial Biopsy
KEY D
Additional Infertility
Information

72
OBS GYN-System Wise 1700-by Sush and Team. 2016
Susmita, Asad, Manu, Saima, Zohaib, Savia, Shanu, Mona, Manisha, Sitara, Samreena, Sami and Komal


Infertility affects around 1 in 7 couples. Around 84% of couples who have
regular sex will conceive within 1 year, and 92% within 2 years

Causes

• male factor 30%


• unexplained 20%
• ovulation failure 20%
• tubal damage 15%
• other causes 15%


Basic investigations

• semen analysis
• serum progesterone 7 days prior to expected next period


Interpretation of serum progestogen

Level Interpretation
Repeat, if consistently low refer to
< 16 nmol/l
specialist
16 - 30
Repeat
nmol/l
> 30 nmol/l Indicates ovulation

Key counselling points

• folic acid
• aim for BMI 20-25
• advise regular sexual intercourse every 2 to 3 days
• smoking/drinking advice


Reference
Dr Khalid/Rabia d. Day21 progesterone
To check for whether it is ovulatiry or anovulatiry cycles. Progesterone level
inceases and peaks 5 to six days post ovulation. Which is 21 day progesterone
levels


Infertility affects around 1 in 7 couples. Around 84% of couples who have
regular sex will conceive within 1 year, and 92% within 2 years

73
OBS GYN-System Wise 1700-by Sush and Team. 2016
Susmita, Asad, Manu, Saima, Zohaib, Savia, Shanu, Mona, Manisha, Sitara, Samreena, Sami and Komal

Causes
• male factor 30%
• unexplained 20%
• ovulation failure 20%
• tubal damage 15%
• other causes 15%

Basic investigations
• semen analysis
• serum progesterone 7 days prior to expected next period (day 21 of 28
day cycle)
indicates ovulation.

Interpretation of serum progestogen

Level Interpretation

< 16 nmol/l Repeat, if consistently low refer to


specialist

16 - 30 Repeat
nmol/l

> 30 nmol/l Indicates ovulation



Key counselling points
• folic acid
• aim for BMI 20-25
• advise regular sexual intercourse every 2 to 3 days
• smoking/drinking advice


Q:1073 An 8wk pregnant lady is brough to the ED due to severe vomiting. She was
administered IV fluids and oral anti-emetics. She still can’t tolerate anything
orally. What is the next best tx?
a. IV feeding
b. IV antiemetics
c. Termination of pregnancy
d. PPI
e. IV steroid

Clincher(s) Severe vomiting in 8 weeks pregnancy, iv done, oral antiemetics given , next
best
A IV feeding-
B IV antiemetics
C Termination of pregnancy

74
OBS GYN-System Wise 1700-by Sush and Team. 2016
Susmita, Asad, Manu, Saima, Zohaib, Savia, Shanu, Mona, Manisha, Sitara, Samreena, Sami and Komal

D . PPI

E IV steroid
KEY B
Additional Hyperemesis gravidarum
Information

Hyperemesis gravidarum describes excessive vomiting during pregnancy. It
occurs in around 1% of pregnancies and is thought to be related to raised beta
hCG levels. Hyperemesis gravidarum is most common between 8 and 12 weeks
but may persist up to 20 weeks*.

Associations

• multiple pregnancies
• trophoblastic disease
• hyperthyroidism
• nulliparity
• obesity


Smoking is associated with a decreased incidence of hyperemesis

Management

• antihistamines should be used first-line (BNF suggests promethazine as


first-line)
• ginger and P6 (wrist) acupressure: NICE Clinical Knowledge Summaries
suggest these can be tried but there is little evidence of benefit
• admission may be needed for IV hydration

then TPN (total perentral nutrion)


Complications

• Wernicke's encephalopathy
• Mallory-Weiss tear
• central pontine myelinolysis
• acute tubular necrosis
• fetal: small for gestational age, pre-term birth


*and in very rare cases beyond 20 weeks

Reference
Dr Khalid/Rabia Tx is IV anti emetics.

75
OBS GYN-System Wise 1700-by Sush and Team. 2016
Susmita, Asad, Manu, Saima, Zohaib, Savia, Shanu, Mona, Manisha, Sitara, Samreena, Sami and Komal

Woman may be progressing towards Hyperemesis Gravidarum.
IV feeding, IV steroids and PPIs are options if IV antiemetics fail to work
Termination is only the last option.

Q:1077 A pt came with dyskaryosis (abn cells in pap smear) to the OPD. She is a heavy
smoker and alcoholic. Cervical smear shows abnormal cells. What is the best
advice for her?
a. Colposcopy
b. Biopsy
c. Endocervical sample
d. Repeat after 4m
e. None
f. Cone biopsy

Clincher(s)
A Colposcopy- FIRST CHOICE
B Biopsy- DONE AFTER COLPOSCOPY
C Endocervical sample- AFTER COLPOSCOPY
D Repeat after 4m- NOT RECOMMENDED (rec after six months)
E E- None ,
F – CONE BIOPSY AFTER COLPOSCOPY
KEY A
Additional Mild and moder dyskaryosis- to HPV testing
Information
Severe: colposcopy – to examine cervix –abnormal areas/preneoplastic areas
can be confirmed- and then biopsy can be taken (endoscopy/camera can be
used)

Cervical cancer screening: interpretation of results

The table below outlines the management of abnormal cervical smears
(around 5% of all smears). Cervical intraepithelial neoplasia is abbreviated to
CIN

The original sample is tested for HPV*


Borderline or mild
• if negative the patient goes back to routine recall
dyskaryosis
• if positive the patient is referred for colposcopy

Moderate dyskaryosis Consistent with CIN II. Refer for colposcopy


Severe dyskaryosis Consistent with CIN III. Refer for colposcopy
Suspected invasive cancer Refer for urgent colposcopy (within 2 weeks)
Repeat smear - if persistent (3 inadequate samples), assessmen
Inadequate
colposcopy

76
OBS GYN-System Wise 1700-by Sush and Team. 2016
Susmita, Asad, Manu, Saima, Zohaib, Savia, Shanu, Mona, Manisha, Sitara, Samreena, Sami and Komal

*high-risk subtypes of HPV such as 16,18 & 33

Reference
Dr Khalid/Rabia If there is class 3 mild moderate or class 4 severe dyskaryosis on smear, the
next step is ro refer the lady for colposcopy, and if needed punch biopsy.
Class 1
Normal pap smear: repeat in 3 years
Class 2
Inflammatory pap smear: Take swab and treat infection. Repeat in 6 months.
Colposcopy after 3 abnormal smears
Mild atypia: repeat in 4 months. Colposcopy after 2 abnormal smears Class 3
Mild dyskaryosis: HPV test +/- colposcopy
Moderate dyskaryosis: colposcopy Class 4
Severe dyskaryosis: colposcopy Class 5
Suspected invasion and abnormal glandular cells: urgent colposcopy

Q:1082 1082. A lady came for OBGYN assessment unit with hx of 8wk pregnancy and
bleeding per vagina for last 2 days. On bimanual exam, uterus =8wks in size.
On speculum exam, cervical os is closed.
How do you confirm the viability of the fetus?
a. Transvaginal US
b. Serum BHCG
c. Urinary BHCG
d. Abdominal US
e. Per speculum exam

Clincher(s)
A Transvaginal US
B Serum BHCG- hcg maybe raised a few days until after death of fetus

C Urinary BHCG- hcg maybe raised a few days until after death of fetus
D Abdominal US
E Per speculum exam- CONTRAINDICATED
KEY A
Additional
Information
Reference
Dr Khalid/Rabia


Transabdominal ultrasound transvaginal ultrasound
PANORAMIC VIEW LIMITED VIEW
UNRELIABLE BEFORE 06 WEEKS PREG 4.5-5 WK


77
OBS GYN-System Wise 1700-by Sush and Team. 2016
Susmita, Asad, Manu, Saima, Zohaib, Savia, Shanu, Mona, Manisha, Sitara, Samreena, Sami and Komal

Transabdominal ultrasound will provide a panoramic view of the abdomen and
pelvis and is noninvasive, whereas transvaginal ultrasound provides a more
limited pelvic view and requires insertion of a probe into the vagina.
Transabdominal ultrasound cannot reliably diagnose pregnancies that are less
than 6 weeks gestation. Transvaginal ultrasound, by contrast, can detect
pregnancies earlier, at approximately 4 ½ to 5 weeks gestation. Prompt
diagnosis made possible by transvaginal ultrasound can, therefore, result in
earlier treatment.
Scenario is that of threatened miscarriage.
Serum and urine b hcg maybe raised a few days until after death of fetus, per
speculum exam is not done for miscarriage.

Q:1098 1098. A 21yo female in her first pregnancy at 38wks was brought to the ED
with generalized tonic clonic seizure. IV MgSO4 was given but fits was not
controlled. She is having fits again. What is the single most imp immediate
management of this pt?
a. IV MgSO4
b. IV diazepam
c. Immediate C-section
d. IV phenytoin
e. MgSO4 bolus
f. IV lorazepam

Clincher(s)
A
B
C
D
E
KEY E, in In eclampsia I/V mg SO4> bolus>diazepam>C/S after control
Additional Eclampsia, meaning literally “to shine forth”, complicates approximately one in
Information 2000 pregnancies and is one of the main causes of maternal death in the
United Kingdom.1 The cause is a pregnancy specific, underlying multiorgan
disorder involving vascular endothelial damage, intravascular coagulation, and
vasoconstriction leading to end organ ischaemia. There may be a variety of
presentations and classic features are not always present. Changes have
occurred in the recommended treatment for eclamptic seizures and are
considerably different from other seizure disorders (including management of
hypertension and careful fluid balance). As one third of cases occur out of
hospital, eclampsia should be considered in the differential diagnosis in any
pregnant woman presenting to the accident and emergency (A&E) department
with seizures.

Previous SectionNext SectionDefinition

78
OBS GYN-System Wise 1700-by Sush and Team. 2016
Susmita, Asad, Manu, Saima, Zohaib, Savia, Shanu, Mona, Manisha, Sitara, Samreena, Sami and Komal

An early definition of eclampsia was the occurrence of seizures in the presence
of pre-eclampsia (shown by hypertension, proteinuria, and oedema occurring
after 20 weeks' gestation). Current definitions place less reliance on the
presence of pre-eclampsia as eclampsia can develop without preceding
symptoms or signs in up to 38% of cases.2

The UK Eclampsia Trial1 definition consisted of: Seizures occurring in pregnancy


or within 10 days of delivery and with at least two of the following features
documented within 24 hours of the seizure:

Hypertension diastolic blood pressure (DBP) of at least 90 mm Hg (if DBP


less than 90 mm Hg on booking visit) or DBP increment of 25 mm Hg
above booking level.

Proteinuria one “plus” or at least 0.3 g/24 h.

Thrombocytopenia less than 100 000/μl.

Raised aspartate amino transferase (AST) greater than 42 IU/l.

Pre-eclampsia is a pregnancy specific, multi-organ disorder. The main features


are hypertension, proteinuria, and generalised oedema occurring after 20
weeks' gestation. Other common features include haemoconcentration,
hypoalbuminaemia, hepatic dysfunction, coagulation problems and
hyperuricaemia. Pre-eclampsia usually regresses within 48 hours of delivery.
Hypertension in this setting is usually taken as DBP 15 mm Hg higher than DBP
in early pregnancy. If previous values are not known, a blood pressure of
greater than140/90 mm Hg is considered significant.

Pathophysiology
Pre-eclampsia/eclampsia is thought to result from abnormal placental
development. Major pathological changes occur in the placental vascular bed
resulting in placental ischaemia. An alteration in the ratio of prostacyclin and
thromboxane occurs along with platelet aggregation, thrombin activation, and
fibrin deposition in maternal systemic vascular beds. Increased capillary
permeability and hypoalbuminaemia also occur. A combination of profound
vasospasm and thrombosis causes dysfunction of almost all organ systems.2

Pre-disposing factors for pre-eclampsia include nulliparity, multiple gestations,


extremes of age (teenagers three times more likely than older women),
diabetes mellitus, hydatidiform mole, fetal hydrops, and family history.1, 4

79
OBS GYN-System Wise 1700-by Sush and Team. 2016
Susmita, Asad, Manu, Saima, Zohaib, Savia, Shanu, Mona, Manisha, Sitara, Samreena, Sami and Komal

In pre-eclampsia, there are exaggerated responses to angiotensin II,
catecholamines, and vasopressin.2 Intravascular volume is reduced. Seizures
are thought to be the result of cerebral vasospasm and endothelial damage
leading to ischaemia, microinfarcts, and oedema.2, 5

Bleeding time is frequently increased in severe pre-eclampsia, although


standard coagulation tests such as prothrombin time and partial
thromboplastin time may be normal.2 The cause of this is uncertain but
increased levels of von Willebrand's factor and other unknown substances may
be implicated.6

Previous SectionNext SectionPresentation of eclampsia


In the UK Eclampsia Trial,1 18% of women suffering eclamptic seizures were
parous and had no previous history of pre-eclampsia or eclampsia. Seizures
were significantly more likely in teenagers and those with multiple
pregnancies—that is, twins, triplets, etc. Making a diagnosis of pre-eclampsia
in the A&E department may be difficult as the features of hypertension,
proteinuria, and oedema can occur individually as part of other diseases and in
normal pregnancy.4 Table 2 summarises the incidence of proteinuria,
hypertension, and symptoms before the onset of seizure. Thirty eight per cent
had their first fit outside the hospital and 49% of women with eclampsia had
multiple seizures. A recent case series suggested that all women at more than
20 weeks' gestation presenting with epigastric or right upper quadrant pain
should have their blood pressure checked and urine analysis performed.7

Preterm and antenatal eclampsia seem to be the most severe. Stillbirth and
neonatal death rates were 22.2 and 34.1 per 1000 deliveries respectively.
Overall, one in 14 offspring of women with eclampsia died.

In the UK, cerebral haemorrhage is the most common cause of death in


eclampsia and pre-eclampsia.8–10 The cerebral manifestations are similar to
hypertensive encephalopathy with thrombosis and fibrinoid necrosis of
cerebral arterioles, diffuse microinfarcts, and petechial haemorrhages in the
brain.2, 4, 5 However, approximately 20% of women with eclampsia have a
systolic blood pressure of less than 140 mm Hg or a DBP of less than 90 mm Hg
around the time of the seizure.11 Retinal changes of hypertensive
encephalopathy are rarely seen.2

As intravascular volume depletion is accompanied by intense vasospasm, these


patients are at increased risk of pulmonary oedema from excessive fluid
replacement. Conversely, they are at risk of hypovolaemia even from the

80
OBS GYN-System Wise 1700-by Sush and Team. 2016
Susmita, Asad, Manu, Saima, Zohaib, Savia, Shanu, Mona, Manisha, Sitara, Samreena, Sami and Komal

normal blood loss associated with delivery.2

Pulmonary oedema may be attributable to a combination of increased


capillary permeability, low colloid osmotic pressure, and pulmonary
endothelial damage. Renal complications include glomerular swelling and
fibrin deposition resulting in glomerular capillary endotheliosis. Oliguria is
common and this can progress to acute tubular necrosis.2

Hepatic dysfunction may result from periportal hepatic necrosis, subcapsular


haemorrhages or fibrin deposition in hepatic sinusoids. In very rare cases, fatal
hepatic rupture may occur.2 Hepatic dysfunction may form part of the HELLP
syndrome, which complicates 0.3% of all pregnancies and up to 20% of women
with severe pre-eclampsia. The syndrome comprises haemolysis, increased
liver enzymes, and low platelets with epigastric or right upper quadrant pain.12
This represents a life threatening complication and requires prompt delivery.

Disseminated intravascular coagulation occurs in 7% of patients with eclampsia


but the cause is unclear.2

Fetal complications are thought to arise as a result of placental hypoperfusion.


These include high fetal loss rate, intrauterine growth retardation, small for
dates infants and increased perinatal mortality. The oxyhaemoglobin
dissociation curve is shifted to the left in pre-eclampsia, reducing oxygen
delivery to the fetus.2

Placental abruption may accompany eclampsia.

Table 5
Immediate management of eclampsia

• Summon senior A&E and obstetric staff


• Secure airway and administer high flow oxygen
• Place wedge under right hip or nurse in left lateral position
• Secure intravenous access and draw blood for FBC, U&Es, LFTs, clotting screen, cross mat
and Kleihauer test if abruption suspected
• Control seizures
• Control hypertension
• Monitor vital signs including BP, ECG, RR, SaO2 and fetal heart rate
• Catheterise bladder, monitor urine output, and test urine for protein

emergency department

81
OBS GYN-System Wise 1700-by Sush and Team. 2016
Susmita, Asad, Manu, Saima, Zohaib, Savia, Shanu, Mona, Manisha, Sitara, Samreena, Sami and Komal

Table 6
Drug treatment in eclampsia 4

Drug Dose Onset Side effects


iv=intravenous.
Seizure control
5–10 mg slow iv
Diazepam bolus
Or
2–4 mg slow iv
Lorazepam bolus
4–6 g slow iv bolus
over 5 minutes
Magnesium then 1–2 g/h iv
sulphate infusion Loss of patellar reflexes
Drowsiness
Slurring of speech
Flushing
Muscle weakness
Respiratory depression
Blood pressure control

5 mg slow iv bolus Headache, tremor, nausea, vomiting,


Hydralazine every 20–30 min 10 min tachycardia
or
10 mg slow iv bolus
doubling every 10–
20 min to max 300
mg total or 1–2 Bradycardia (fetal), maternal flushing,
Labetalol mg/min iv infusion 5–10 min nausea
Fluids
1–2 ml/kg/h with monitoring of
Crystalloid urine output

Magnesium sulphate is administered in a loading dose of 4 to 6 g given as a


slow intravenous bolus (for example, four 2 ml ampoules of 50% solution made
up to 20 ml with 5% dextrose or sterile water, given over five minutes). This

82
OBS GYN-System Wise 1700-by Sush and Team. 2016
Susmita, Asad, Manu, Saima, Zohaib, Savia, Shanu, Mona, Manisha, Sitara, Samreena, Sami and Komal

loading dose should be followed by a maintenance infusion of 1 to 2 g per
hour. The patient should be monitored carefully for clinical signs of magnesium
toxicity, particularly loss of patellar reflexes, drowsiness, flushing, slurring of
speech, muscle weakness and respiratory depression, which may herald
respiratory (or cardiorespiratory) arrest. Level of consciousness, respiratory
rate and effort and the presence of patellar reflexes should be frequently and
regularly recorded during the infusion. If toxicity is suspected, the infusion
should be discontinued and if required, calcium gluconate (10 ml of 10%
solution) should be given. Magnesium sulphate increases sensitivity to non-
depolarising neuromuscular blocking agents such as vecuronium.
Fasciculations may not occur after suxamethonium.2

CONTROL OF BLOOD PRESSURE


Hydralazine is known to be effective for controlling hypertension in pre-
eclampsia/ eclampsia.11 After an intravenous bolus, effects are seen by 10
minutes and are maximal by 20 minutes. Its duration of action is six to eight
hours. Side effects include hypotension, tachycardia, tremor, headache,
nausea, and vomiting. Unfortunately, this may mimic impending eclampsia.
Neonatal thrombocytopenia has also been reported in association with the use
of hydralazine.

Labetalol is being used more frequently in this setting. Its onset of action is
more rapid than hydralazine, reflex tachycardia does not occur and there are
few maternal or neonatal side effects. It was found to safely lower mean
arterial pressure in a randomised controlled double blind trial of 152 women
with pregnancy induced hypertension.23

Under invasive blood pressure monitoring, blood pressure should be


controlled aiming for a DBP of 90 to 100 mm Hg. Give hydralazine 5 mg slow
intravenous bolus and repeat every 20 to 30 minutes as indicated, or, give
labetalol 10 mg slow intravenous bolus, doubling the dose every 10 minutes
(for example, 10, 20, 40, etc to a maximum total of 300 mg) or start an
intravenous infusion of 1–2 mg/min until the required DBP is reached.

FLUID MANAGEMENT
Pre-eclampsia/eclampsia seems to be a high cardiac output state associated
with an inappropriately high peripheral resistance. It is also associated with
haemoconcentration, reduction and central redistribution of plasma volume.24
Volume expansion seems to produce transient benefit but there are no studies
to suggest that this is accompanied by reduced maternal or fetal morbidity or
mortality.2

83
OBS GYN-System Wise 1700-by Sush and Team. 2016
Susmita, Asad, Manu, Saima, Zohaib, Savia, Shanu, Mona, Manisha, Sitara, Samreena, Sami and Komal

Controversy exists as to whether central venous pressure monitoring is helpful
as it may not accurately reflect pulmonary capillary wedge pressure. Most
cases are managed without such monitoring. One study, using invasive
monitoring in 49 patients with severe pre-eclampsia, demonstrated normal or
high cardiac output in the presence of normal wedge and central venous
pressure, and inappropriately high systemic vascular resistance.24 It concluded,
that because filling pressures were normal, fluid should be given cautiously to
avoid precipitating pulmonary oedema. Hypotension and fetal distress have
been reported in pre-eclamptic patients given epidural analgesia or
hydralazine without prior fluids. In view of the increased risk of pulmonary
oedema, a suggested fluid regimen is of crystalloids given at 1–2 ml/kg/h with
careful monitoring of urine output.2, 4

http://emj.bmj.com/content/17/1/7.full


Magnesium Sulphate Regimen: Magnesium Sulphate (MgSO4) is the


treatment of choice for the first fit.

Loading dose: Magnesium Sulphate 4 grams · 8mls of MgSO4 (50%) diluted


with 12mls Normal Saline (0.9%) = Total 20mls · Give IV over 20 minutes
using syringe driver rate of 60 mls/hour

Maintenance dose: Magnesium Sulphate 1 gram per hour · 20mls MgSO4 (10
gms) diluted with 30mls Normal Saline (0.9%) = Total 50mls · Give IV using
syringe driver at rate of 5mls/hour

Recurrent seizures whilst on Magnesium Sulphate · Further bolus of 4mls


MgSO4 (2 gms) diluted with 6mls Normal Saline (0.9%) Give IV over 5
minutes · If possible take blood for Magnesium levels before bolus · Notify
Obstetric and Anaesthetic Consultants

If further seizures occur · Inform Consultants

ECLAMPSIA AND SEVERE PRE-ECLAMPSIA – CLINICAL GUIDELINE Page 7 of 15

· Consider other causes of fits including intracranial haemorrhage · Consider


using other drugs, including general anaesthesia

http://www.rcht.nhs.uk/DocumentsLibrary/RoyalCornwallHospitalsTrust/Clinic
al/MidwiferyAndObstetrics/PreEclampsiaEclampsiaGuidelineForTheManagem

84
OBS GYN-System Wise 1700-by Sush and Team. 2016
Susmita, Asad, Manu, Saima, Zohaib, Savia, Shanu, Mona, Manisha, Sitara, Samreena, Sami and Komal

entOfSevere.pdf
Reference
Dr Khalid/Rabia Treatment of Eclampsia
Treat first seizure with 4g MGSO4 in 100 ml NS IVI over 5 min and
mantainenece IVI of 1g per hr for 24 hr.
If recurrent, give 2g ivi magnesium sulphate over 5 mins.
Use diazepam once if fits continue.
According to this regimen, the correct answer

Q:1099 1099. A 24yo lady with BMI=30 complains of facial hair growth and hx of
amenorrhea. FSH=10.9,prolactin=400IU, estradiol=177.8mmol/l,
progesterone=normal, LH=33.2. What is the most
probable dx?
a. PCOS
b. Pregnancy
. Cushi g s disease
d. CAH
e. POF
Older lady- POF (prem ov failure)
Clincher(s)
A PCOS- RAISED LH, FSH, NORMAL PRL (poly ov syndr)
B Pregnancy
C Cushings Disease- Raised ACTH/Cortisol
D CAH (cong adrenal hyperplasia)
E POF
KEY A
Additional Normal FSH level will differ depending on a person's age and gender.
Information
• Female:

o Before puberty: 0 - 4.0 mIU/ml


o During puberty: 0.3 - 10.0 mIU/ml
o Women who are still menstruating: 4.7 - 21.5 mIU/ml
o After menopause: 25.8 - 134.8 mIU/ml

High FSH levels in women may be present:

• During or after menopause, including premature menopause

• When receiving hormone therapy

• Due to certain types of tumor in the pituitary gland

• Due to Turner syndrome

85
OBS GYN-System Wise 1700-by Sush and Team. 2016
Susmita, Asad, Manu, Saima, Zohaib, Savia, Shanu, Mona, Manisha, Sitara, Samreena, Sami and Komal

Low FSH levels in women may be present due to:

• Being very underweight or having had recent rapid weight loss

• Not producing eggs (not ovulating)

• Parts of the brain (the pituitary gland or hypothalamus) not producing


normal amounts of some or all of its hormones

• Pregnancy

High FSH levels in men may mean the testicles are not functioning correctly
due to:

• Advancing age (male menopause)

• Damage to testicles caused by alcohol abuse, chemotherapy, or


radiation

• Problems with genes, such as Klinefelter syndrome

• Treatment with hormones

• Certain tumors in the pituitary gland

Low FSH levels in men may mean parts of the brain (the pituitary gland or
hypothalamus) do not produce normal amounts of some or all of its hormones.

High FSH levels in boys or girls may mean that puberty is about to start.

In women, FSH helps manage a women’s cycle and stimulates the ovaries to
produce eggs. The test is used to help diagnose or evaluate:

• Menopause

• Women who have polycystic ovary syndrome, ovarian cysts

• Abnormal vaginal or menstrual bleeding

• Problems becoming pregnant, or infertility

In men, FSH stimulates production of sperm. The test is used to help diagnose
or evaluate:

• Problems becoming pregnant, or infertility

86
OBS GYN-System Wise 1700-by Sush and Team. 2016
Susmita, Asad, Manu, Saima, Zohaib, Savia, Shanu, Mona, Manisha, Sitara, Samreena, Sami and Komal

• Men who do not have testicles or whose testicles are under-developed

In children, FSH is involved with the development of sexual features. The test is
ordered for children:

• Who develop sexual features at a very young age

• Who are delayed in starting puberty

In Women

LH and FSH levels can help to differentiate between primary ovarian failure
(failure of the ovaries themselves or lack of ovarian development) and
secondary ovarian failure (failure of the ovaries due to disorders of either the
pituitary or the hypothalamus).

Increased levels of LH and FSH are seen in primary ovarian failure. Some
causes of primary ovarian failure are listed below.

Developmental defects:

• Failure to develop ovaries (ovarian agenesis)


• Chromosomal abnormality, such as Turner syndrome
• Defect in steroid production by the ovaries, such as 17 alpha
hydroxylase deficiency

Premature ovarian failure due to:

• Exposure to radiation
• Chemotherapy
• Autoimmune disease

Chronic failure to ovulate (anovulation) due to:

• Polycystic ovary syndrome (PCOS)


• Adrenal disease
• Thyroid disease
• Ovarian tumor

In women who are trying to become pregnant, multiple LH tests can be used to
detect the surge that precedes ovulation. An LH surge indicates that ovulation
has occurred.

During menopause, a woman's ovaries cease to function; thus LH levels will

87
OBS GYN-System Wise 1700-by Sush and Team. 2016
Susmita, Asad, Manu, Saima, Zohaib, Savia, Shanu, Mona, Manisha, Sitara, Samreena, Sami and Komal

rise.

Low levels of LH and FSH are seen in secondary ovarian failure and indicate a
problem with the pituitary or hypothalamus. See the article on Pituitary
Disorders for more information.

high level of prolactin (hyperprolactinemia) is normal during pregnancy and


after childbirth while the mother is nursing.

A high level may also be seen with:

• Tumors that produce and release prolactin (prolactinomas)


• The eating disorder anorexia nervosa
• Diseases of the hypothalamus
• Hypothyroidism
• Kidney disease
• Liver disease
• Polycystic ovary syndrome
• Other pituitary diseases and tumors

Levels of prolactin that are below normal are not usually treated but may be
indicative of a general decrease in pituitary hormones caused by a pituitary
disorder such as hypopituitarism

Stress from illness, chest wall trauma, seizures, lung cancer, and use of
marijuana can cause moderate increases in prolactin.

Drugs that can cause an elevated prolactin include estrogen, tricyclic


antidepressants, risperidone, opiates, amphetamines, hypertension drugs
(reserpine, verapamil, methyldopa) and some drugs that are used to treat
gastroesophageal reflux (cimetidine). Nipple stimulation may cause a
moderate increase in prolactin blood level.

Low prolactin levels may be caused by drugs such as dopamine, levodopa, and
ergot alkaloid derivatives.

Prolactinomas are often small. Along with prolactin levels, a health practitioner
may do an MRI (magnetic resonance imaging) of the brain to locate and
determine the size of the tumor as well as the size of the pituitary gland


Cushing's syndrome: investigations

Investigations are divided into confirming Cushing's syndrome and then
localising the lesion. A hypokalaemic metabolic alkalosis may be seen, along

88
OBS GYN-System Wise 1700-by Sush and Team. 2016
Susmita, Asad, Manu, Saima, Zohaib, Savia, Shanu, Mona, Manisha, Sitara, Samreena, Sami and Komal

with impaired glucose tolerance. Ectopic ACTH secretion (e.g. secondary to
small cell lung cancer) is characteristically associated with very low potassium
levels. An insulin stress test is used to differentiate between true Cushing's and
pseudo-Cushing's

Tests to confirm Cushing's syndrome

The two most commonly used tests are:

• overnight dexamethasone suppression test (most sensitive)


• 24 hr urinary free cortisol


Localisation tests

The first-line localisation is 9am and midnight plasma ACTH (and cortisol)
levels. If ACTH is suppressed then a non-ACTH dependent cause is likely such
as an adrenal adenoma

High-dose dexamethasone suppression test

• if pituitary source then cortisol suppressed


• if ectopic/adrenal then no change in cortisol


CRH stimulation

• if pituitary source then cortisol rises


• if ectopic/adrenal then no change in cortisol


Petrosal sinus sampling of ACTH may be needed to differentiate between
pituitary and ectopic ACTH secretion

o

Reference
Dr Khalid/Rabia Young lady, obese, with facial hair and history of amenorrhea. Looks like PCOS
as cause seems to be ovarian indicated by a raised LH and FSH, and normal
prolactin.
It encompasses a syndrome of polycystic ovaries, in association with systemic
symptoms causing reproductive, metabolic and psychological disturbances.
These most commonly present with infertility, amenorrhoea, acne or
hirsutism.
The patient often presents in the peripubertal period through to her mid 20's.

89
OBS GYN-System Wise 1700-by Sush and Team. 2016
Susmita, Asad, Manu, Saima, Zohaib, Savia, Shanu, Mona, Manisha, Sitara, Samreena, Sami and Komal

Symptoms:
Oligomenorrhoea (defined as <9 periods per year)
Infertility or subfertility, Acne, Hirsutism, Alopecia, Obesity or difficulty losing
weight, Psychological symptoms[5] - mood swings, depression, anxiety, poor
self-esteem Sleep apnoea
Differential diagnosis * Thyroid disorder (particularly
hypothyroidism),Hyperprolactinaemia,Cushing's syndrome,Late-onset
congenital adrenal hyperplasia (very rare),Androgen-secreting ovarian or
adrenal tumours,Ovarian hyperthecosis (signs of virilism and biochemical
androgen excess are much more prominent in the latter three)
Investigations
* This may show LH elevated, LH:FSH ratio increased (>2), with FSH normalFree
testosterone levels may be raised, but if total testosterone is >5 nmol/L,
exclude androgen-secreting tumours and congenital adrenal hyperplasia.
* Laparoscopy or ultrasound shows characteristic ovaries (the average volume
is three times that of normal ovaries .Other blood tests, where indicated from
the clinical picture, to exclude other potential causes - eg, TFT (thyroid
dysfunction), 17-hydroxyprogesterone levels (congenital adrenal hyperplasia ),
prolactin (hyperprolactinaemia), DHEA-S and free androgen index (androgen-
secreting tumours), and 24-hour urinary cortisol (Cushing's syndrome).
* Fasting glucose
* Fasting lipid levels should be checked.
Diagnosis Two of the three following criteria are diagnostic of the condition
(Rotterdam criteria):[6]
* Polycystic ovaries (either 12 or more peripheral follicles or increased ovarian
volume (greater than 10 cm3)
* Oligo-ovulation or anovulation
* Clinical and/or biochemical signs of hyperandrogenism
MANAGEMENT
They should be advised on weight control and exercise.] It has been shown to
improve ovulation, pregnancy rate
Pharmacological treatment
There is no treatment which reverses the hormonal disturbances of PCOS and
treats all clinical features, so medical management is targeted at individual
symptoms, and only in association with lifestyle changes.
For women not planning pregnancy
* Co-cyprindrol: is licensed for treating hirsutism and acne, although not
specifically in PCOS. Combined oral contraceptive pill (COCP): is also used to
control menstrual irregularity.
* Metformin: has been increasingly used off-licence for PCOS; however, a
Cochrane review showed it to be less effective than the COCP for menstrual
irregularity, hirsutism and acne,[13] and the National Institute for Health and
Care Excellence (NICE) Evidence Summary suggests its side-effects and cost
outweigh its benefits and any, as yet unproven, long-term health benefits.[14]
* Eflornithine: may be used for hirsutism, as can cosmetic treatments
(electrolysis, laser, waxing, bleaching).

90
OBS GYN-System Wise 1700-by Sush and Team. 2016
Susmita, Asad, Manu, Saima, Zohaib, Savia, Shanu, Mona, Manisha, Sitara, Samreena, Sami and Komal

* Orlistat: can help with weight loss in women with PCOS and may improve
insulin sensitivity[15]
For women wishing to conceive and presenting with infertility
2013 NICE guidelines advise (after weight loss where indicated, and a full
fertility work-up) women should be treated with clomifene, metformin or a
combination of the two.[10] (A Cochrane review in 2012, however, found no
benefit in live birth rates from the use of metformin or other insulin-sensitising
drugs, although it did improve numbers of pregnancies.[16])
Complications
* Oligomenorrhoea or amenorrhoea are known to predispose to endometrial
hyperplasia and endometrial cancer in untreated cases. It has been suggested
that women with PCOS may have a higher cardiovascular risk than weight-
matched controls.
* Women presenting with PCOS, particularly if they are obese (BMI greater
than 30), have a strong family history of type 2 diabetes or are over the age of
40 years, are
atincreased risk of type 2 diabetes and should be offered screening. The risk
may be as high as 10-20%.
* Women diagnosed with PCOS (or their partners) should be asked about
snoring and daytime fatigue/somnolence and informed of the possible risk of
sleep apnoea. They should be offered investigation and treatment when
necessary.[20]
* Complications in pregnancy: there is a higher risk of gestational diabetes in
women with PCOS, which may be more than double..

Q:1101 1101. A lady comes with a missing IUCD thread. Her LMP was 2wks ago. What
is the single most appropriate next step in management?
a. Abdominal US
b. Prescribe contraceptives
c. CT
d. Serum BHCG
e. Vaginal exam

Clincher(s)
A
B
C
D
E
KEY
Additional
Information
Reference
Dr Khalid/Rabia key: A

cause: in case of lost thread we advise the pt with extra contraception like

91
OBS GYN-System Wise 1700-by Sush and Team. 2016
Susmita, Asad, Manu, Saima, Zohaib, Savia, Shanu, Mona, Manisha, Sitara, Samreena, Sami and Komal

condom then we start managing the case by : pregnancy test but in this case
her LMP was two weeks ago so no need for than , so the next step is to locate
the IUCD using imaging studies ( us first then xray)

lost thread : OCS pg 298

the IUCD may have been expelled, so advise extra contaception and exclude
pregnancy

seek coil on u/s ; if missing arrange x-ray to exclude extra-uterine coils


( surgical retrieval is advised)

- In case of a lost thread, a number of measures should be taken. They are:

- If threads are not visible, or if they are but the stem of the device is palpable,
the woman should be advised to use condoms or abstain from intercourse
until the site of the device (if present) can be determined.

- Perform a speculum examination to ensure the device is not in the posterior


fornix.

- Determine whether the woman is already pregnant.

- With consent, explore the lower part of the endocervical canal with narrow
artery forceps: threads which have been drawn a little way up are usually
found by this method.

- An experienced operator may, after appropriate analgesia (eg, mefenamic


acid 500 mg) explore the uterine cavity with a retriever hook.

- Hormonal emergency contraception may be indicated.

- Ultrasound should be arranged to locate the device.

- If ultrasound does not locate the device and there is no definite history of
expulsion then abdominal X-ray should be performed to look for an
extrauterine device.

- Expulsion should not otherwise be assumed.

- Hysteroscopy can be helpful if ultrasound is equivocal.

- Surgical retrieval of an extrauterine device is advised.

- The question asks for the most appropriate next step in the management of
this case. That step would be to exclude pregnancy via an abdominal
ultrasound. IUCDs increase the risk of ectopic pregnancy, for which again, the
next step would be an abdominal ultrasound.

92
OBS GYN-System Wise 1700-by Sush and Team. 2016
Susmita, Asad, Manu, Saima, Zohaib, Savia, Shanu, Mona, Manisha, Sitara, Samreena, Sami and Komal

Q:1102 1102. A 32yo woman presents with hx of lower abdominal pain and vaginal
discharge. She had her menses 4wk ago. She has a temp of 38.6C. What is the
most suitable dx?
a. Acute appendicitis
b. Acute PID
c. Endometriosis (hx of menstrual abnormalities)
d. Ectopic pregnancy
e. UTI

Clincher(s) LOWERABDOMINAL PAIN, DISCHARGE, FEVERNO AMENORRHOEA- PID
A
B
C
D
E
KEY
Additional
Information
Reference
Dr Khalid/Rabia key: B

cause: clinical signs and symptoms fit ( fever above 38 + bilatral lower
abdominal pain + vaginal discharge)

PID

Diagnosis of acute PID made only on clinical signs

signs & symptoms

• Bilateral lower abdominal pain.

• Deep dyspareunia.

• Abnormal vaginal bleeding (postcoital, intermenstrual or


menorrhagia).

• Vaginal or cervical discharge that is purulent.

• Fever above 38°C

Investigations:

• Pregnancy test (pregnant women with PID should be admitted;


ectopic pregnancy may be confused with PID).

• Cervical swabs for chlamydia and gonorrhoea

• Endocervical swabs for C. trachomatis and N. gonorrhoeae

93
OBS GYN-System Wise 1700-by Sush and Team. 2016
Susmita, Asad, Manu, Saima, Zohaib, Savia, Shanu, Mona, Manisha, Sitara, Samreena, Sami and Komal

Treatment:

• Mild or moderate disease can be managed in primary care or


outpatients, whereas clinically severe disease requires hospital
admission for intravenous (IV) antibiotics.

• The current outpatient treatment recommendation is ceftriaxone


500 mg as a single (IM) dose, followed by doxycycline 100 mg orally
twice daily and metronidazole 400 mg twice daily for 14 days

• Initial treatment is with doxycycline, single-dose IV ceftriaxone and


IV metronidazole, then change to oral doxycycline and
metronidazole to complete 14 days of treatment.

- Symptoms of PID

- Bilateral Lower/Pelvic abdominal pain that ranges from mild to severe.

- Pain during sex

- Abnormal vaginal discharge

- Abnormal vaginal bleeding (1 in 4 cases)

- Risk Factors

- A recent change of sexual partner. The risk goes up with the number of
partners.

- A previous episode of PID or sexually transmitted disease.

- A recent abortion.

- A recent operation or procedure on the womb (uterus).

- A contraceptive coil inserted recently.

- Tests to be done

- Endocervical Swab is the investigation of choice. High vaginal swab can also
be taken.

- If the above doesn’t show anything, an U/S can be done to look for inflamed
fallopian tubes.

- Complications

- Infertility

- Ectopic Pregnancy

- Miscarriage and Still Births

94
OBS GYN-System Wise 1700-by Sush and Team. 2016
Susmita, Asad, Manu, Saima, Zohaib, Savia, Shanu, Mona, Manisha, Sitara, Samreena, Sami and Komal

- Persistent pain (including pain during sex)

- Reiter’s Syndrome

- Abscess formation

- Treatment

- Antibiotics

- The partner must also be treated


Q:1103 1103. A 40yo female was on COCP which she stopped 6m ago. But she has not
had her periods since then. Labs: FSH=22, LH=24, prolactin=700, estradiol=80.
What is the most appropriate dx?
a. Hypothalamic amenorrhea
b. Post pill amenorrhea
c. Prolactinoma
d. Pregnancy
e. Premature ovarian failure
High prolactin >1000 (otherwise no –oma); more than 40yr- menopause;
Clincher(s) COCP, NO PERIODS AFTER THAT, FSH/LH RATIO<2- POF
A
B
C
D
E
KEY POF? (manisha to check)
Additional FSH- peri meno: 3-13
Information
in post menopausal FSH & LH >30

OHCS- see last page




Reference
Dr Khalid/Rabia Key E as per rabia (to post this)

reason: this is a case of secondary amenorrhea with elevated gonadotropins


indicating ovaian failure and the patient is 40 years so it's premature ovarian
failure

causes of secondary amenorrhea:

• with no androgen excess:

1-pregnancy(the most common cause of secondary


amenorrhea),lactation,menopause.

95
OBS GYN-System Wise 1700-by Sush and Team. 2016
Susmita, Asad, Manu, Saima, Zohaib, Savia, Shanu, Mona, Manisha, Sitara, Samreena, Sami and Komal

2-premature ovarian failure:

this is a poorly understood condition that may represent an autoimmune


phenomenon. It can also follow radiotherapy or chemotherapy. With all these
causes, menstruation and fertility can sometimes resume spontaneously.
Ovarian failure will cause elevation of gonadotrophins and so hot flushes are
likely. Premature menopause is defined as occurring before the age of 40.


3-depot and implant contraception

4-cevical stenosis and intrauterine adhessins >>> Asherman's syndrome

5-wt loss >>> especially if rapid

6-pitutary disease and hyperprolactinaemia

7-sheehan syndrome >>> the pituitary may be damaged by tumours, trauma,


cranial irradiation, sarcoidosis or tuberculosis

8- post pill amenorrhea

with androgen excess :

PCOS


Premature Ovarian Failure – Secondary Amenorrhea (E)

- Causes

- Hyperprolactinemia

- PCOS

- Premature Ovarian Failure

- Post-pill Amenorrhea

- Asherman’s Syndrome

- Investigations

- Pregnancy test (if appropriate)

- Follicle-stimulating hormone (FSH) and luteinising hormone (LH)

- PRL

- Total testosterone and sex hormone-binding globulin

- TSH

96
OBS GYN-System Wise 1700-by Sush and Team. 2016
Susmita, Asad, Manu, Saima, Zohaib, Savia, Shanu, Mona, Manisha, Sitara, Samreena, Sami and Komal

- A pelvic ultrasound may be useful in patients with suspected PCOS

- Treatment

Treatment is related to cause. Premature Ovarian Failure is irreversible but


hormone replacement is necessary for controlling the symptoms of Estrogen
deficiency and protection against Osteoporosis. Pregnancy can be achieved by
oocyte donation or in vitro fertilization techniques.

- What to look for in the question

- Secondary amenorrhea before or at the age of 40 (The age is 40 in this


question)

- A raised Gonadotropin level


Q:1104 1104. A 25yo woman presents with a single lump in the breast and axilla. The
lump is mobile and hard in consistency. The US, mammogram and FNA turn
out to be normal. What is the most appropriate inv to confirm the dx?
a. FNAC
b. MRI
c. Punch biopsy
d. Genetic testing and counselling
e. Core biopsy

Clincher(s)
A Fna- done (= needle biopsy)- usually for breast and other lumps- screening
B MRI- NO ROLE
C PUNCH BIOPSY- NOT DONE (on skin)
D GENETIC COUNSELLING- NOT RECOMMENDED
E CORE BIOPSY- IX OF CHOICE (on solid organ)- also or lumps and other liver
renal etc. – confirmatory
KEY
Additional
Information
Reference
Dr Khalid/Rabia key: E

reason : in palpable mass we do triple assessment : clinical


examination,imaging,FNAC >>>if not conclusive we do core biopsy then
excional biopsy if still non conclusive

Breast lump triple assessment

• examination

97
OBS GYN-System Wise 1700-by Sush and Team. 2016
Susmita, Asad, Manu, Saima, Zohaib, Savia, Shanu, Mona, Manisha, Sitara, Samreena, Sami and Komal

• imaging: mamogaraphy,u/s

• biopsy

* non palpable lesion:

1-core biopsy(image -guided)

2- open biopsy(needle localisation >>> radio opaque needles are used to guide
the boipsy0

*palpable lesion:

1-FNAC

2-core biopsy

3-excision biopsy (entire lesion is removed)

4-incision biopsy (part of the lesion)

All patients with breast lumps must undergo triple assessments.

1st Assessment: Clinical examination of the breast including axillary lymph


nodes

2nd Assessment: Imaging

- U/S for <35 years old

Mammography and THEN U/S for >35 years old patient.

3rd Assessment: Cytology

- If it is a cyst, perform FNAC

-Clear Fluid? Reassure the patient

- Blood-stained aspirate? Send to lab for Cytology

- Clear fluid but residual mass? Perform core biopsy

In this particular question:

The patient is a young woman with a single lump in the breast and the axilla.
The lump is hard and mobile which points towards Fibroadenoma but a
swollen lymph node in the axilla points towards a carcinoma. To find out which
one it is, we need a core biopsy since an ultrasound, a Mammogram and an
FNAC turned out to be normal. The usual typical order of investigations in such
cases is:

- Ultrasound/ Mammogram depending upon the age.

98
OBS GYN-System Wise 1700-by Sush and Team. 2016
Susmita, Asad, Manu, Saima, Zohaib, Savia, Shanu, Mona, Manisha, Sitara, Samreena, Sami and Komal

- Mammogram

- FNAC

- Core Biopsy


Q:1105 1105. A 37yo lady stopped taking COCP 18m ago and she had amenorrhea for
12m duration. Labs: FSH=8, LH=7, prolactin=400, estradiol=500. What is the
cause?
a. Hypothalamic amenorrhea
b. PCOS
c. Prolactinoma
d. Post pill amenorrhea
e. POF

Clincher(s) COCP, AMENORRHOEA, LOW FSH, LH, POST PILL AMENORRHOEA
A Hypothalamic amenorrhea
B PCOS- NORMAL /LOW LH, FSH
C Prolactinoma, prl n
D Post Pill Amenorrhoea
E POF
KEY D
Additional
Information
Reference
Dr Khalid/Rabia key:D

reason :this is a case of secondary amenorrhea following COCP use,,,

decreased gonadotropins >>> so it's not POF,,

and there are no symptoms of increased androgens>>> so it's not PCOS ,,

NO increased prolactin >>>so not prolactinoma,,,

the history of amenorrhea after COCP use + decreased gonadotropins fit with
>>> post pill amenorrhea

Post-pill amenorrhoea

this is when stopping oral contraceptives does not lead to a resumption of a


normal menstrual cycle. It usually settles spontaneously in around three
months but, if not, it requires investigation. The condition is probably not a
true entity but the cause of amenorrhoea started whilst taking the
contraceptives that induced an artificial cycle until they were stopped.

Post pill amenorrhea is described as the loss of menstrual periods for at least 6
months after stopping birth control pills. The incidence of post-pill amenorrhea

99
OBS GYN-System Wise 1700-by Sush and Team. 2016
Susmita, Asad, Manu, Saima, Zohaib, Savia, Shanu, Mona, Manisha, Sitara, Samreena, Sami and Komal

ranges from 0.2% to 3%.

- Cause of Post-pill Amenorrhea

Post-pill amenorrhea is believed to be due to suppression of the pituitary gland


by the birth control pills.

- Investigations

- The diagnosis of post-pill amenorrhea is usually made when there is loss of


periods after a prolonged history of taking birth control pills. - Ultrasonography
will reveal ovaries with no signs of developing follicles and ovulation even after
having stopped the pills for 6 months.

- Blood tests showing a low level of FSH, LH and estrogen is usually sufficient to
confirm the diagnosis.

- Treatment

- The first line of treatment in case of post-pill amenorrhea is waiting for a


spontaneous remission of the amenorrhea and a spontaneous occurrence of
periods.

- The time limit is usually six months. But if the woman is anxious to get her
periods, active treatment may be started after waiting for only three months.

- The standard treatment of post-pill amenorrhea is by stimulating the


pituitary to produce FSH and LH. This is done by the drug clomiphene citrate.


Q:1106 1106. A lady with a firm smooth breast lump in outer quadrant had a FNAC
done. Results showed borderline benign changes. She also has a fam hx of
breast cancer. What is the your next?
a. Mammography
b. US
c. Core biopsy
d. Genetic testing and counselling
e. Punch biopsy

Clincher(s) Smooth breast lump, FNA done, borderline benig, fam hx,
A Mammography- wont add
B US- FNA done already
C Core Bx- benign FNA
D Genetic testing and counselling to determine freq of follow up
E Punch Biopsy
KEY D
Additional
Information
Reference

100
OBS GYN-System Wise 1700-by Sush and Team. 2016
Susmita, Asad, Manu, Saima, Zohaib, Savia, Shanu, Mona, Manisha, Sitara, Samreena, Sami and Komal

Dr Khalid/Rabia key : D

reason: assessment of the case is complete (borderline benign change),,,next


step is genetic testing and counselling.

breast lump >>>discussed earlier

In this question a lady underwent an FNAC and the results have showed benign
breast changes. She also has a risk for developing breast cancer because of her
family history. All options but one are investigations, which we don’t need at
this point because we got all we could get from the investigations. What needs
to be done now is to assess the risk of breast cancer in this patient. Also, the
patient needs to be counselled about the disease. Therefore, we should for
genetic testing and counselling.


Q:1107 1107. A pt presents with mild dyskaryosis. 1y ago smear was normal. What is
the most appropriate
next step?
a. Cauterization
b. Repeat smear
c. Swab and culture
d. Cone biopsy
e. Colposcopy
Mild or moderate dyskaryosis: HPV first
Clincher(s)
A Cauterization - erosion
B Repeat smear- rec after mild dyskaryosis
C Swab and culture- no role
D Cone bx- not recommended
E Colposcopy- ?
KEY E
Additional If HPV given- then for mild dyskarosis- HPV, if HPV positive- then colposcopy
Information
Reference
Dr Khalid/Rabia key: E

Cervical screening

Cervical cancer is the third or fourth most common female malignancy


worldwide

The screening process is done using LBC (liquid based cytology) or older
method (PAP)

Interpreting smear results:

Cells are analysed to look for abnormalities in the appearance of the nucleus

101
OBS GYN-System Wise 1700-by Sush and Team. 2016
Susmita, Asad, Manu, Saima, Zohaib, Savia, Shanu, Mona, Manisha, Sitara, Samreena, Sami and Komal

and other aspects of cell morphology (dyskaryosis)

• Negative (normal) >>> treat incidental findings eg,infection & recall


as appropriate

• inadequate : insufficient or unsuitable material


sampled >>>>Repeat sample immediately after treating any
infection,,Repeat sample as soon as convenient if technically
inadequate,,,if persistent (three inadequate samples), advise
assessment by colposcopy

• Borderline changes and mild dyskaryosis>>>> HPV testing ,,if


positive >> colopscopy,,

• if negative >>> normal recall

• if unreliable >>> repeat HPV in six month and if +ve >> colopscopy

• moderate and severe dyskaryosis >>>> refer to colopscopy

Cervical Cancer – When to refer for Colposcopy (E)

- Any smear showing mild, moderate or severe Dyskaryosis

- Any suggestion of malignancy

- 3 consecutive inflammatory smears

- 2 consecutive atypical smears

- 3 consecutive borderline smears

- 3 consecutive inadequate smears

- Post-coital bleeding


Q:1108 1108. An African lady presents with heavy but regular periods. Her uterine size
correlates to 14wks pregnancy. What is the most appropriate dx?
a. Blood dyscrasia (bleeding disorder)
b. Hematoma (hx of injury)
c. Fibroids
d. Adenomyosis (endometriosis in the myomectomy- heavy irregular period,
infertility, bulky but not so big)> ut upto 8 wks size
e. Incomplete abortion

Clincher(s) African heavy regular periods, 14 wks uterus, - commonest fibroid
A
B
C
D

102
OBS GYN-System Wise 1700-by Sush and Team. 2016
Susmita, Asad, Manu, Saima, Zohaib, Savia, Shanu, Mona, Manisha, Sitara, Samreena, Sami and Komal

E
KEY C
Additional
Information
Reference
Dr Khalid/Rabia
Q:1109 1109. A 29yo at 38wks GA presents with a 2h hx of constant abdominal pain.
She then passes 100ml of blood per vagina. What is the next appropriate inv?
a. USS
b. CTG
c. Clotting screen
d. Hgb
e. Kleihauer Betke test (Rh – mother with rh + baby/ or group incompatibility–
detect fetal cells in maternal blood- to understand how much anti D should be
given; for every 4 ml of fetal, we give 500 IU of anti D ; koombs test is done in
fetal blood)

Clincher(s) 38 wks GA, abdominal pain, BPV-
A USS- TO KNOW CAUSE OF BLEEDING
B CTG- FETAL DISTRESS
C Clotting screen-
D HgB
E . Kleihauer Betke test
KEY A (suspect abruption)
Additional No vaginal examination should be attempted, at least until a placenta praevia
Information is excluded by ultrasound. It may initiate torrential bleeding from a placenta
praevia.


Reference
Dr Khalid/Rabia

key: A

reason : in case of antenatal bleeding we do u/s to investigate the cause of


bleeding

Antenatal bleeding

• never perform PV >> might increase bleeding

• dangerous causes: Abruption,placenta praevia,vasa praevia

• other caused:circumvallate placenta,placental sinuses cervical


polyps,erosions and carcinoma,cervicitis,vaginitis,vulvar varicosities

• placental abruption>>>> part of placenta becomes detached ,, the


outcome depends on the amount of detachment and blood

103
OBS GYN-System Wise 1700-by Sush and Team. 2016
Susmita, Asad, Manu, Saima, Zohaib, Savia, Shanu, Mona, Manisha, Sitara, Samreena, Sami and Komal

loss,,bleeding might be concealed

• associations>>>
preeclampsia,smoking,cocaine,IUGR,PROM,abdominal
trauma,multiple pregnancy,polyhydramnios,increased maternal
age,non vertex presentation,assisted reproductive techniques

• placenta praevia>>>placenta is in the lower uterine


symptoms,,bleeding is always revealed

Management:

• in severe bleeding>>> admission,,IV lines , O2 mask,, blood


transfusion if in shock,,catheterize,,if severe bleeding >> CS

• in mild cases>>> IVI,,Hb , cross match,check vitals >> establish dx by


u/s >>if placenta praevia >>>cs

Bleeding During Third Trimester – APH (A)

- Bleeding during the third trimester is either painful or painless bleeding.

- Painful bleeding points to Placental Abruption while painless bleeding points


to Placenta Previa

- In this case, the patient is experiencing painful vaginal bleeding in the third
trimester, so this appears to be the case of Placental Abruption

- The investigation of choice in this case is an Ultrasound

- Risk Factors for Abruption

- Hypertension

- Smoking

- Multiple Pregnancy

- Cocaine/Amphetamine Use

- Increased Maternal Age

- Trauma to the abdomen

- Polyhydramnios

- Investigation

- Diagnosis is clinical but U/S is done to exclude Placenta Praevia and to check
the well-being of the baby.

- Management

104
OBS GYN-System Wise 1700-by Sush and Team. 2016
Susmita, Asad, Manu, Saima, Zohaib, Savia, Shanu, Mona, Manisha, Sitara, Samreena, Sami and Komal

- Always admit the patient to hospital for assessment and management. Phone
999/112/911 if there are any major concerns regarding maternal or fetal well-
being.

- The mainstays of management are resuscitation and accurate diagnosis of


the underlying cause.

- Severe bleeding or fetal distress: urgent delivery of the baby, irrespective of


gestational age.

- Admit to hospital, even if bleeding is only a very small amount. There may be
a large amount of concealed bleeding with only a small amount of revealed
vaginal bleeding.

- No vaginal examination should be attempted, at least until a placenta praevia


is excluded by ultrasound. It may initiate torrential bleeding from a placenta
praevia.

- Resuscitation can be inadequate because of underestimation of blood loss


and misleading maternal response. A young woman may maintain a normal
blood pressure until sudden and catastrophic decompensation occurs.

- Take blood for FBC and clotting studies. Crossmatch, as heavy loss may
require transfusion.

- Gentle palpation of the abdomen to determine the gestational age of the


fetus, presentation and position.

- Fetal monitoring.

- Arrange urgent ultrasound.

- With every episode of bleeding, a rhesus-negative woman should have a


Kleihauer test and be given prophylactic anti-D immunoglobulin

- Complications

- Premature Labour

- DIC

- Renal Tubular Necrosis

- PPH

- Placenta Accreta

- Points to look for diagnosis of Abruption

- Shock is out of proportion from visible blood loss

- Constant pain

105
OBS GYN-System Wise 1700-by Sush and Team. 2016
Susmita, Asad, Manu, Saima, Zohaib, Savia, Shanu, Mona, Manisha, Sitara, Samreena, Sami and Komal

- Tender tense uterus

- Fetal heart sounds absent/distressed

- Coagulation problems like DIC


Q:1110 1110. A 26yo woman had amenorrhea for 10wks and is pregnant. She
experiences hyperemesis. Now she presents with vaginal bleed. Exam:
uterus=16wks, closed os. What is the most probable dx?
a. Thyrotoxicosis
b. Hyperemesis gravidarum
c. Twins
d. Wrong dates
e. Molar pregnancy

Clincher(s) 10 WK AMENORRHOEA, HYPEREMESIS VAGINALA BLEED, UTERUS- 16
WEEKS, CLOSED OS
A THYROTOXICOSIS-
B HYPEREMESIS GRAVIDARUM-
C TWINS
D WRONG DATES
E MOLAR PREGNANCY LARGE FOR DATES, BPV,
KEY E (Molar preg: large for date uterus, hyperemesis, p/V bleeding or not; HCG
very high, snow storm appearance: chorico ca is complication- so f/U B HCG –
(cannon ball) lung mets common)
Additional
Information
Reference
Dr Khalid/Rabia key : E

reason: increased uterine size for date + hyperemesis + vaginal bleeding

Molar pregnancy

• complete mole >>> MRI-guided transcutaneous focused


ultrasound,,the most common

• partial mile >>>the trophoblast cells have three sets of


chromosomes (triploid)

• risk factors>>> mother over 45ys,,previous molar


pregnancy,multiple pregnancy,menarche over 12 ys ,asian women ,
oral contraceptive pills

• presentation >>>vaginal bleeding in the first trimester ,


hyperemesis, abnormal uterine enlargement, hyperthyroidism,
anaemia, respiratory distress and pre-eclampsia are now rare as a

106
OBS GYN-System Wise 1700-by Sush and Team. 2016
Susmita, Asad, Manu, Saima, Zohaib, Savia, Shanu, Mona, Manisha, Sitara, Samreena, Sami and Komal

result of routine use of ultrasound in early pregnancy

Investigations

• Urine and blood levels of hCG >>> for follow up after evacuation

• histology>>>> Definitive diagnosis is made by histological


examination of the products of conception.

• u/s>>>>Ultrasound in the first trimester may not be reliable. The


typical 'snowstorm' appearance occurs mainly in the second
trimester

treatment :

• Suction curettage is the method of choice of evacuation for


complete molar pregnancies.

• Suction curettage is the method of choice of evacuation for partial


molar pregnancies except when the size of the fetal parts deters
the use of suction curettage and then medical evacuation can be
used.

• follow up :

• A urinary pregnancy test should be performed three weeks after


medical management of failed pregnancy if products of conception
are not sent for histological examination.

• complications: choriocarcinoma >>> follows a molar pregnancy and


should always be considered when a patient has continued vaginal
bleeding after the end of a pregnancy. It has the ability to spread
locally, as well as metastasise.

Molar Pregnancy

- Tumor consists of chorionic villi which have swollen and degenerated

- It makes a lot of HCG and therefore gives rise to exaggerated pregnancy


symptoms, e.g. Hyperemesis Gravidarum

- Signs

- Exaggerated pregnancy symptoms

- A larger for dates uterus

- Hyperthyroidism

- Most present with early pregnancy failure

- Investigation

107
OBS GYN-System Wise 1700-by Sush and Team. 2016
Susmita, Asad, Manu, Saima, Zohaib, Savia, Shanu, Mona, Manisha, Sitara, Samreena, Sami and Komal

- Snowstorm appearance on U/S

- Management

- Suction and curettage is the method of choice for evacuation

- Give anti-D prophylaxis

- Monitor HCG levels every two weeks until they are normal

- Monitor monthly for six months after they return to normal

- Pregnancy should be avoided for a year while HCG levels are being monitored

- Measure HCG 6-8 weeks after any future pregnancy regardless of the
outcome


Q:1111 1111. A pregnant woman of G2, GA 11wks presents with heavy vomiting,
headache and reduced urine output. Urine analysis shows ketonuria
(dehydration). Choose the next best step?
a. US
b. Oral fluid replacement
c. Serum BHCG
d. Parental anti-emetics
e. IV fluids

Clincher(s) GA- 11 WEEKS, VOMITINGS, HEADACHE, REDUCED UO, KETONURIA –
HYPEREMESIS GRAVIDARUM
A US
B ORAL FLUID REPLACEMENT
C SERIM B HCG
D PARENTAL ANTIEMETICS
E IV FLUIDS
KEY E
Additional
Information
Reference
Dr Khalid/Rabia key: E

reason : the pt has reduced urine output,vomiting ,headache ,ketonuria


>>which indicates hypovolemia >>> IV fluids

Vomiting in pregnancy

Presentation

• Symptoms usually start between 4 and 7 weeks of gestation and


resolve by 16 weeks in about 90% of women. Check for signs of

108
OBS GYN-System Wise 1700-by Sush and Team. 2016
Susmita, Asad, Manu, Saima, Zohaib, Savia, Shanu, Mona, Manisha, Sitara, Samreena, Sami and Komal

dehydration

Management

• Most cases are mild and do not require treatment

• Diet >>> Advise the patient to rest; eat small, frequent meals that
are high in carbohydrate and low in fat

• Anti-emetic drug treatment>>> This should only be given when


symptoms are persistent, severe and preventing daily
activities( cyclizine, metoclopramide, prochlorperazine,
promethazine, chlorpromazine, domperidone and ondansetron )

• Proton pump inhibitors and histamine H2-receptor antagonists may


be used in women who also have dyspepsia

• Hyperemesis gravidarum:

• intractable vomiting associated with weight loss of more than 5% of


pre-pregnancy weight, dehydration, electrolyte
imbalances( hyponatremia,hypokalemia), ketosis, and the need for
admission to hospital.

• management:

• dietary advice

• Fluid and electrolyte replacement >>> intravenous fluid and


electrolyte replacement

• Nutritional support (enteral or parenteral)

• Vitamin supplements >>> thiamine

• Thromboprophylaxis >>> LMWH

• Anti-emetic drugs

• Corticosteroids: may be used for intractable (failure to respond to


conventional treatment) cases of severe hyperemesis gravidarum in
secondary care.

• In cases of hyperemesis gravidarum: renal function and


electrolytes, LFTs, midstream urine and ultrasound (exclude
multiple or molar pregnancy).

Treating Dehydration due to Heavy Vomiting (E)

- This patient has heavy vomiting which causes dehydration.

- The immediate next step here would be fluid correction through IV route.

109
OBS GYN-System Wise 1700-by Sush and Team. 2016
Susmita, Asad, Manu, Saima, Zohaib, Savia, Shanu, Mona, Manisha, Sitara, Samreena, Sami and Komal


Q:1112 1112. A pt had inflammatory changes (means no dyskariosis) on cervical
smear. There is no vaginal discharge, no pelvic pain and no fever. What is the
next step?
a. Repeat smear in 6m- 3
b. Take swab (for infection)- 1st step
c. Treat with antibiotics- 2
d. Colposcopy
e. Cone biopsy (when colposcopy abnormal)

Clincher(s) INFLAMMATORY SMEARS- ASYMPTOMATIC
A REPEAT SMEARS- DYSKARYOSIS .
B SWAB- INFECTION
C TREAT WITH ANTIBIOTICS
D COLPOSCOPY
E CONE BIOPSY
KEY B
Additional
Information
Reference
Dr Khalid/Rabia key : b

reason : if inaccurate repeat the test ( full topic discussed earlier )

Inflammatory Changes on Cervical Smear (A)

OHCS on encountering inflammatory changes: Take swab. Treat infection.


Repeat in 6 months. Colposcopy after 3 abnormal.


NAAT test- chalmydia test; cultures are not done. Endocervical swab
taken

In Uk Kit given for first urine


Q:1113 1113. A 37yo infertile lady with 5cm subserosal and 3cm submucosal fibroid is
trying to get pregnant.
Which is the most suitable option?
a. Clomifen therapy (for Post pill amennnorhea)
b. IVF
c. Myomectomy
d. Hysterectomy
e. IU insemination (inter uterine)
If there is infertility problem: myomectomy first choice in fibroid
Clincher(s) Subserosal and submucosal fibroids- myomectomy
A

110
OBS GYN-System Wise 1700-by Sush and Team. 2016
Susmita, Asad, Manu, Saima, Zohaib, Savia, Shanu, Mona, Manisha, Sitara, Samreena, Sami and Komal

B
C
D
E
KEY C
Additional
Information
Reference
Dr Khalid/Rabia key : c

reason : submucosal fibroids cause infertility >> myomectomy ( full topic


discussed earlier)

- Refer to 1108. It is explained in detail over there.


Q:1114 1114. A young tall man and his wife are trying for babies and present at the
infertility clinic. On inv the man has primary infertilitiy and azoopermia. What
other inv should be done?
a. Testosterone
b. LSH
c. FSH
d. Estradiol
e. Karyotyping

Clincher(s) TALL MAN, INFERTILITY, PRIMARY MALE , AZOOSPERMIA- KLINEFELTER
A
B
C
D
E
KEY
Additional Connective tissue disorder: marfan; tall thin, no infertility, hyperflexibility
Information
Reference
Dr Khalid/Rabia key : e

reason : in primary azoospermia + tall man >>> karyotyping >>> klinefelter's


syndrome

Klinefelter's syndrome (tall young, infertility)

karyotype XXY is associated with hypogonadism and disorders of


spermatogenesis.

The classic clinical description

• Infertility and small firm testes are present in about 99% of

111
OBS GYN-System Wise 1700-by Sush and Team. 2016
Susmita, Asad, Manu, Saima, Zohaib, Savia, Shanu, Mona, Manisha, Sitara, Samreena, Sami and Komal

individuals ,decreased facial and pubic hair; loss of libido;
impotence.

• Tall and slender, with long legs, narrow shoulders, and wide hips.

• Gynaecomastia or history of gynaecomastia during puberty;


decreased libido; history of undescended testes.

• Learning disability; delayed speech development; behavioural


problems; psychosocial disturbances.

• tiredness, reduced muscle power and stamina, and truncal obesity

• Investigations

• XXY males may be diagnosed before birth, through amniocentesis


or chorionic villus sampling.

• Later, serum testosterone is low or low normal. FSH and LH are


elevated (FSH >LH).

• Diagnosis is confirmed by chromosomal analysis. The most common


indications for karyotyping are hypogonadism and infertility.

Management

• testosterone replacement

• fertility treatment : ICSI

• Surgical treatment : for gynaecomastia

- Young tall man with primary infertility and Azoospermia points in the
direction of Klinefelter’s Syndrome.

- The definitive diagnosis is chromosomal analysis also known as karyotyping.


Q:1115 1115. A woman who is on regular COCP presented to you for advice on what to
do as she has to now start to take a course of 7d antibiotics. What would you
advice?
a. Continue regular COC
b. Continue COCP and backup contraception using condoms for 2d
c. Continue COCP and backup contraception using condoms for 7d
d. Continue COCP and backup contraception using condoms for 2wks

Clincher(s) COCP, ANTIBIOTICS
A
B

112
OBS GYN-System Wise 1700-by Sush and Team. 2016
Susmita, Asad, Manu, Saima, Zohaib, Savia, Shanu, Mona, Manisha, Sitara, Samreena, Sami and Komal

C
D
E
KEY
Additional If rifampicin and rifambiutin have been given, extra contracives will be given
Information as these are enzyme inducers

And see below
Reference
Dr Khalid/Rabia key : D

reason : There are many commonly used medications which can affect the
efficacy of the pill

INTERACTIONS :

• *Non-enzyme-inducing antibacterial >>> Women should be


advised that no additional contraception is required.

• *Enzyme-inducing antibacterials rifampicin or rifabutin , and


anticonvulsants, St John's wort, (Short course two months or less )
>>> Women are advised to continue taking the COCP ,use
additional precautions AND should be continued for 28 days after
stopping the rifampicin/rifabutin.

• *Enzyme-inducing antibacterials rifampicin or rifabutin,and


anticonvulsants, St John's wort (Long-term course) >>> Should be
advised to use an alternative, non-hormonal method

• *Lamotrigine >>> Women should be advised not to take lamotrigine


with the COCP and should seek another form of contraception
(unless also taking a non-enzyme-inducing anticonvulsant such as
sodium valproate)

• *Antiretroviral therapies>>> women on ritonavir-boosted protease


inhibitors should be advised to use alternative methods of
contraception.

• *Ulipristal acetate>>>Women should use additional contraceptive


precautions for 14 days after taking ulipristal acetate

- Antibiotics like Rifampicin and Griseofulvin increase the breakdown of


estrogen and therefore can cause unwanted pregnancy.

- In such cases, backup contraception using condoms for two weeks should be
advised.

113
OBS GYN-System Wise 1700-by Sush and Team. 2016
Susmita, Asad, Manu, Saima, Zohaib, Savia, Shanu, Mona, Manisha, Sitara, Samreena, Sami and Komal

Q:1116 1116. A lady presents with hot flashes and other symptoms of menopause.
What is the tx option?
a. Raloxifen
b. HRT
c. Bisphosphonate
d. COCP
e. Topical estrogen

Clincher(s)
A RALOXIFEN (esogrogen receptor modulators)= tamoxifen?
B HRT
C BISPHOSPHONATE (given in osteoporosis)
D COCP
E TOPICAL ESTROGEN (senile or atrophic vaginitis)
KEY B
Additional
Information
Reference
Dr Khalid/Rabia key :B

reason : HRT is the gold standard treatment for hot flushes.

MENOPAUSE

The menopause is a natural phenomenon occurs in all women when the


number of ovarian follicles are depleted.

oestrogen & progesterone fall, and LH &FSH increase in response.


Menstruation becomes erratic and eventually stops

PRESENTATION

• Menstrual irregularity >>> which may last for up to four years,The


cycle may lengthen or shorten,A slight increase in the amount of
menstrual blood loss,10% of women have an abrupt cessation of
periods.

• Hot flushes and sweats

• Urinary and vaginal symptoms >>> dyspareunia, vaginal discomfort


and dryness, recurrent lower urinary tract infection and urinary
incontinence

• Sleep disturbance , Loss of libido AND Mood changes

Management

• Healthy lifestyle >>> Stopping smoking, losing weight and limiting


alcohol

114
OBS GYN-System Wise 1700-by Sush and Team. 2016
Susmita, Asad, Manu, Saima, Zohaib, Savia, Shanu, Mona, Manisha, Sitara, Samreena, Sami and Komal

• Hormone replacement therapy (HRT) >>> the most effective
treatment to completely relieve the symptoms
particularly Vasomotor symptoms (hot flushes/night
sweats) ,Mood swings ,Vaginal and bladder symptoms caused by
the menopause. It also prevents and reverses bone loss.

• Alternatives to HRT >>> Herbal or complementary treatments

• Other drugs >>> GABA , SSRI


Q:1117 1117. A 28yo woman at 34wks GA for her first pregnancy attends antenatal
clinic. Her blood results:
Hgb=10.6, MCV=95 (if low- iron deficient anaemia), MCHC=350. What do you
do for her?
a. Folate
b. Dextran
c. Ferrous sulphate
d. None
e. IV FeSO4
f. Explain this physiologic
hemodynamic anemia
g. Blood transfusion

Clincher(s) 34 WEEKS GA, HB N FOR SECOND /THIRD TRIMESTER, MCV NORMAL, MCHC
HIGH
A
B
C
D
E
KEY F
Additional
Information
Reference
Dr Khalid/Rabia key : F

reason : normal MCV,MCHC + in the second and third trimester anemia is


considered when Hb is less than 10.5

Anemia in pregnancy

definition >>> Hb level <11.0 g/dL at booking. In the second and third
trimesters the diagnostic level for anaemia is an Hb level of <10.5 g/dL.
Postpartum the diagnostic level is 10.0 g/dL.

115
OBS GYN-System Wise 1700-by Sush and Team. 2016
Susmita, Asad, Manu, Saima, Zohaib, Savia, Shanu, Mona, Manisha, Sitara, Samreena, Sami and Komal

Iron-deficiency anaemia accounts for 85% of all cases of anaemia that are
identified and is characterised by MCV, and it's caused by nutritional deficiency
or low iron stores

Presentation >>> often asymptomatic,Fatigue,Dyspnoea m pallor

Investigations

• Hb

• MCV ≤ 76 >>> the probable cause is iron deficiency

• Normal MCV (76-96 fl) with low Hb is typical of pregnancy

• Serum ferritin 10-50 μg/L needs monitoring and <10 μg/L requires
treatment.

Management:

• Women with known haemoglobinopathy >>> check serum ferritin


>>> if low give oral supplements

• Women with unknown haemoglobinopathy status with a


normocytic or microcytic anaemia>>> oral supplements and
haemoglobinopathy screening

• Non-anaemic women at increased risk of iron deficiency >>> check


serum ferritin >>> if low give oral supplements

• Women with established iron deficiency anaemia >>> give 100-200


mg elemental iron daily, continue for at least three months

* Referral to secondary care in case of >>> significant symptoms ,severe


anaemia (Hb<70 g/dL) ,late gestation (>34 weeks) or failure to respond to a
trial of oral iron.


- Hb < 11.5 g/dl

- 85% of the times the cause is Iron deficiency

- Other causes are less common

- Investigations

- Hb

- MCV: If less than 76, it is most probably due to Iron deficiency. If it is normal
then it is the typical dilutional anemia of pregnancy.

- Serum Ferritin: 10-50 ug/L needs monitoring; less than 10 ug/L requires
treatment

116
OBS GYN-System Wise 1700-by Sush and Team. 2016
Susmita, Asad, Manu, Saima, Zohaib, Savia, Shanu, Mona, Manisha, Sitara, Samreena, Sami and Komal

- Management

- Routine Iron Replacement is not recommended in the UK.

- Treat with oral Ferrous Sulphate if Hb is less than 11.5 g/dl


Q:1118 1118. A 34yo woman who never had fits or high BP developed fits 6h after
delivery of a term healthy child. What is the most likely dx?
a. Eclampsia
b. Preeclampsia
c. Epilepsy (as she has no other symptoms- except fits)
d. Pulmonary embolism
e. Pregnancy induced HTN


Clincher(s) Previously healthy, fits after 6hrs of birth- eclampsia (upto 24 hrs after preg)
A
B
C
D
E
KEY C (as no other symptom during pregnancy; eg BP)
Additional
Information
Reference
Dr Khalid/Rabia

reason: postpartum fits>> occurs in 45% of cases of eclampsia

Eclampsia

Eclampsia is defined as the occurrence of one or more convulsions


superimposed on pre-eclampsia. 44% of seizures occur postnatally, the
remainder being antepartum (38%) or intrapartum (18%).

Management of eclampsia:

• Resuscitation >> The patient should be placed in the left lateral


position and the airway secured + O2

• Treatment and prophylaxis of seizures >> Magnesium sulfate is the


anticonvulsant drug of choice, intubation may become necessary in
women with repeated seizures

• Treatment of hypertension >> (blood pressure >160/110 mmHg or


mean arterial pressure >125 mmHg) ,,,Intravenous hydralazine or
labetalol are the two most commonly used drugs ( SE >> fetal

117
OBS GYN-System Wise 1700-by Sush and Team. 2016
Susmita, Asad, Manu, Saima, Zohaib, Savia, Shanu, Mona, Manisha, Sitara, Samreena, Sami and Komal

distress,so continuous fetal heart rate monitoring)

• Fluid therapy

• Delivery >>>The definitive treatment of eclampsia is delivery

Complications :

• haemolysis, HELLP syndrome (3%), disseminated intravascular


coagulation (3%), renal failure (4%) and adult respiratory distress
syndrome (3%).

• Cerebrovascular haemorrhage is a complicating factor in 1-2%.


- Symptoms of pre-eclampsia+Seizure = Eclampsia

- NOTE: If a woman has a fit within a few days after delivery, it is always
eclampsia until proven otherwise.

- Management

- ABC

- MgSO4 IV bolus 4g, then 1g IV infusion for 24 hours, and if seizure recurs give
IV bolus.

- Monitor BP, pulse, RR and Oxygen saturation every 15 minutes.

- If BP> 160/110, give antihypertensive (Hydralazine, Labetalol, Nifedipine)

- NOTE: If the patient has been given MgSO4 and experiences another fit,
repeat MgSO4.


Q:1119 A 30yo lady who already has one child through a prv C-section demands a
reversible contraception. She presently experiences heavy and painful periods.
What is the most appropriate contraceptive you will recommend for her?
a. COCP
b. POP
c. Implanon
d. Danazol
e. Mirena
f. IUCD

Clincher(s) Heavy painful periods, reversible contraception,
A
B
C

118
OBS GYN-System Wise 1700-by Sush and Team. 2016
Susmita, Asad, Manu, Saima, Zohaib, Savia, Shanu, Mona, Manisha, Sitara, Samreena, Sami and Komal

D
E
KEY E (for first 7 days alternative contraception used as it takes 7 days to work)
Additional
Information
Reference
Dr Khalid/Rabia key : E

reason: Women with idiopathic menorrhagia, Consider the LNG-IUS(Merina)


as the first-line option

Contraception in special groups

• Menorrhagia >>> Women with idiopathic


menorrhagia>>> Consider the LNG-IUS(Merina) as the first-line
option, COCP as the second-line option, and POP and progestogen-
only injectables as third-line options.

• Fibroid >>> With distortion of the uterine cavity: cu-IUCDs and the
LNG-IUS should NOT be used

• obese >>> COCP shouldn NOT be used

• Migraine :

-WITH AURA >>> don't use COCP ---- use POP, progestogen-only
implants and injectables, and the LNG-IUS BUT they are not recommended to
continue if the woman developed migraine with aura while using them

-WITHOUT AURA (<35 years of age) >>> NOT usually recommended


>>> continuation of COCP, the combined contraceptive patch, and combined
contraceptive vaginal ring

-WITHOUT AURA ( ≥35 years of age) >>> don't use COCP

• DM >>> Methods that should not usually be used: progestogen-only


injectables. COCP, the combined contraceptive patch and the
combined contraceptive vaginal ring

• Hypertension >>> Methods that are not usually recommended:


COCP, the combined contraceptive patch and combined
contraceptive vaginal ring.

• Irregular menses >>> COCP

• Smoking >>> Methods that should not be used: COCP, combined


contraceptive patch and combined contraceptive vaginal ring.

• Multiple risk factors for cardiovascular disease>>> Methods that


should not be used: COCP, combined contraceptive patch and
combined contraceptive vaginal ring AND POP are not usually

119
OBS GYN-System Wise 1700-by Sush and Team. 2016
Susmita, Asad, Manu, Saima, Zohaib, Savia, Shanu, Mona, Manisha, Sitara, Samreena, Sami and Komal

recommended

• Venous thromboembolism or risk of thromboembolism>>>


Methods that should not be used: COCP, the combined
contraceptive patch, and the combined contraceptive vaginal ring.

• Sickle cell disease >>> intrauterine devices are not recommended,


as they may be associated with uterine bleeding and infection ,
Depot contraceptive (Depo-Provera®) is safe and has been found to
improve the blood picture and reduce pain crises.

• Women taking anticoagulants for VTE >>> Methods that should not
be used: COCP, combined contraceptive patch and combined
contraceptive vaginal ring AND POP are not usually recommended

• Sexually transmitted infection or pelvic inflammatory disease


>>> insertion of a cu-IUCD or the LNG-IUS is not recommended.

• Contraception for those with learning disabilities >>> use of


injectable contraceptives and IUCDs is high


• Contraception for those also taking enzyme enhancers >>>COCP -
all women should be advised to switch to a contraceptive method
unaffected by enzyme inducers (eg progestogen-only injectable,
copper IUCD (Cu-IUCDs) or LNG-IUS).or to cover with another
method eg. condoms

POP>>> Advise alternative contraceptive methods

Progestogen-only implants >>> May continue with progestogen-only


implants with additional
contraceptive protection, such as condoms, when taking liver enzyme-inducers
and for four weeks after they are stopped.

drugs which induce liver enzymes include:

Antifungals: griseofulvin.

Antibiotics: rifampicin and rifabutin.

Anti-epileptics: carbamazepine, eslicarbazepine, phenytoin, phenobarbital,


primidone, oxcarbazepine, topiramate.

Central nervous system stimulant: modafinil.

Antiretroviral drugs: nelfinavir, nevirapine, ritonavir.

St John's wort.


- Mirena, also known as Intrauterine System, is a Levonorgestril-containing coil

120
OBS GYN-System Wise 1700-by Sush and Team. 2016
Susmita, Asad, Manu, Saima, Zohaib, Savia, Shanu, Mona, Manisha, Sitara, Samreena, Sami and Komal

which is inserted into the uterus.

- Local effect: reversible endometrial atrophy, makes implantation less likely


and periods lighter (20% reversible amenorrhea)

- Less risk of ectopic pregnancy

- Risk of STDs is reduced

- May benefit women with endometriosis, adenomyosis, fibroids or


endometrial hyperplasia

- NOTE: Avoid if breast cancer.


Q:1120 (studied 1120. A 32yo woman comes with intermenstrual bleeding. Her last cervical
from here) smear was 1y ago and was negative. What test would you recommend for her
initially?
a. Colposcopy
b. Cervical smear
c. Endocervical swab
d. Transvaginal US
e. Pelvic CT

Clincher(s)
A
B
C
D
E
KEY B (to rule out Cervical ca)
Additional
Information
Reference
Dr Khalid/Rabia key : b

reason : to exclude cervical carcinoma

- In this patient there is no indication for colposcopy. Indications for


colposcopy include:

- Smear showing mild, moderate or severe Dyskaryosis

- Any suggestion of malignancy

- 3 consecutive inflammatory smears

121
OBS GYN-System Wise 1700-by Sush and Team. 2016
Susmita, Asad, Manu, Saima, Zohaib, Savia, Shanu, Mona, Manisha, Sitara, Samreena, Sami and Komal

- Glandular abnormal cells

- 2 consecutive atypical smears

- 3 consecutive borderline or inadequate smears

- Post-coital bleeding

- Endocervical Swab is not advisable at this point due to absence of any signs
of infection

- Initially, a cervical smear should be taken as the previous one was negative
one year ago (25-50 years, take cervical smear every 3 years)


Q:1121 1121. A 20yo woman has had abdominal pain in the LIF for 6wks duration.
Over the past 48h, she has
severe abdominal pain and has a fever of 39.1C. Pelvic US shows a complex
cystic 7cm mass in
the LIF. What is the most likely dx?
a. Endometriosis
b. Dermoid cyst
c. Ovarian ca
d. Tubo-ovarian abscess
e. Ectopic pregnancy

Clincher(s) Abdominal pain LIF, fever, complex cystic mass lif- abscess…localised.
A ENDOMETRIOSIS- GENERALISED, CYCLICAL,NO FEVER.
B Dermoid Cyst- localized, no fever, painless
C Ovarian ca- no fever pain
D Tubo Ovarian abscess- localized, fever, complex cystic mass
E Ectopic, fever, complex cystic mass. No amenorrhoea
KEY D
Additional
Information
Reference
Dr Khalid/Rabia key : D

reason : fever 39.1 +cystic pelvic mass + localized abdominal pain

Tubo-ovarian abscess

A tubo-ovarian abscess is one type of pelvic abscess which is found in women


of reproductive age, and may be a complication of pelvic inflammatory
disease. In this case it is an inflammatory mass which involves the ovary and
Fallopian tube

122
OBS GYN-System Wise 1700-by Sush and Team. 2016
Susmita, Asad, Manu, Saima, Zohaib, Savia, Shanu, Mona, Manisha, Sitara, Samreena, Sami and Komal

Presentation

• Systemic features of toxicity: fever, malaise, anorexia, nausea,


vomiting, pyrexia.

• Local effects: eg, pain, deep tenderness , diarrhoea, tenesmus,


mucous discharge per rectum, urinary frequency, dysuria, vaginal
bleeding or discharge.

Investigations

• FBC >>> raised white cell count often but not invariably

• Ultrasound

• CT/MRI scanning may be more effective at identifying the origin of


the abscess

Management:

• Hospital admission

• Drainage of the abscess along with antibiotic treatment

• Antibiotics used alone are occasionally effective for very early, small
abscesses

• Procedures used for drainage of the pelvic abscess >>> Ultrasound-


guided aspiration and drainage----CT-guided aspiration and
drainage----Endoscopic ultrasound-guided drainage ----Laparotomy
or laparoscopy with drainage of abscess

• Definitive surgery may be required after initial drainage for some


causes of pelvic abscess, such as salpingo-oophorectomy for tubo-
ovarian abscess.

-General symptomatology of pelvic infections

- Systemic features of toxicity: fever, malaise, anorexia, nausea, vomiting,


pyrexia.

- Local effects: eg, pain, deep tenderness in one or both lower quadrants,
diarrhoea, tenesmus, mucous discharge per rectum, urinary frequency,
dysuria, vaginal bleeding or discharge.

- Rectal or vaginal examination: may reveal tenderness of the pelvic


peritoneum and bulging of the anterior rectal wall.

- Partial obstruction of the small intestine: this may sometimes occur.

-Investigations

123
OBS GYN-System Wise 1700-by Sush and Team. 2016
Susmita, Asad, Manu, Saima, Zohaib, Savia, Shanu, Mona, Manisha, Sitara, Samreena, Sami and Komal

- FBC showing increased WBC count

- U/S

- CT/MRI scanning may help in tracing the origin of the abscess

- Management

- Arrange urgent admission to hospital.

- Management is usually by drainage of the abscess along with antibiotic


treatment. Antibiotics used alone are occasionally effective for very early,
small abscesses

- Antibiotic choice is guided by the likely cause and local resistance patterns
and guidelines, but usually needs to be broad-spectrum until the pathogens
are determined.

- Points in favour of Tubo-Ovarian Abscess

- Fever

- Pain in the LIF

- Severe abdominal pain

- U/S showing a cystic mass in the LIF


Q:1122 1122. A woman is 16wk pregnant and she is worried about abnormal
chromosomal anomaly in her
child. What is the definitive inv at this stage?
a. Amniocentesis
b. CVS
c. Parents karyotyping
d. Coo s test
e. Pre-implantation genetic dx

Clincher(s) 16 WEEKS, ABNORMAL CHROMOSOME
A AMNIOCENTESIS 15TH WEEK ONWARDS
B CVS 11-13 WEEK
C PARENTS KARYOTYPING
D COOMBS TEST (for Rh incompatibility)
E PIGD
KEY
Additional
Information
Reference
Dr Khalid/Rabia key : A

124
OBS GYN-System Wise 1700-by Sush and Team. 2016
Susmita, Asad, Manu, Saima, Zohaib, Savia, Shanu, Mona, Manisha, Sitara, Samreena, Sami and Komal

Prenatal diagnosis

• Amniocentesis : This is normally carried out from 15 weeks of


gestation. A needle is inserted into the amniotic cavity and amniotic
fluid is sampled, allowing culture and assessment of fetal cells.the
most invasive prenatal diagnostic procedure.

• Chorionic villus sampling : carried out at 11-13 weeks. Risk of


miscarriage may be slightly higher than for
amniocentesis. sampling of the developing placenta and the same
type of analysis of fetal cells to detect chromosomal, genetically
inherited and endocrine or metabolic conditions.

• Fetoscopy : This allows visualisation of the fetus, using endoscopic


techniques. at 18-20 weeks of gestation

• Cordocentesis/percutaneous umbilical blood sampling : at 18


weeks.ultrasound guidance to obtain fetal blood cells from the
umbilical cord.It enables karyotyping/chromosome analysis,
Intrauterine blood transfusions, Fetal viral infection confirmation.

• Fetal radiology : in suspected skeletal dysplasia, ultrasound (2D and


3D) and MRI are now the investigations of choice

• Ultrasound-guided percutaneous skin and organ biopsy : This can


also be carried out to allow skin, muscle, liver and other fetal organ
analysis

• Maternal blood tests : eg. Maternal serum alpha-fetoprotein levels


can be measured to aid the diagnosis of neural tube defects
between 15-22 weeks

- Definitive tests

- Pre-implantation Genetic Diagnosis (earliest possible diagnosis; available to


couples at risk of having a child with specific genetic or chromosomal disorder
such as CF, Thalassemia and Huntington’s etc)

- Chorionic Villous Sampling (done between 10 and 13 weeks; enables early


detection and provides the mother with a choice to either continue or
terminate pregnancy)

- Amniocentesis (done between 14-18 weeks; carries a very low risk of


miscarriage)

- The patient is 16 weeks pregnant, so the best definitive testing that can be
carried out at this stage would be Amniocentesis.


Q:1123 1123. A 28yo lady with a fam hx of CF comes for genetic counselling and wants
the earliest possible dx test for CF for the baby she is planning. She is not in

125
OBS GYN-System Wise 1700-by Sush and Team. 2016
Susmita, Asad, Manu, Saima, Zohaib, Savia, Shanu, Mona, Manisha, Sitara, Samreena, Sami and Komal

favor of termination. What would you recommend for her?
a. CVS
b. Amniocentesis
c. Pre-implantation genetic dx
d. Chromosomal karyotyping
e. Maternal serum test (NTD)
f. Reassure

Clincher(s) PRE NATAL, FAM HX CF, NOT IN FAVOUR OF TERMINATION
A
B
C
D
E
KEY C
Additional
Information
Reference
Dr Khalid/Rabia key : C

reason :pre-implantation genetic dx is the earliest diagnostic test

Pre-implantation prenatal diagnosis

This is a technique that allows the analysis of oocytes or embryos conceived


through in vitro fertilisation (IVF). This information then informs the choice of
optimal embryos to be transferred back to the mother. This is an accepted
technique for avoiding the birth of affected children from parents with a
known genetic abnormality

- There is a positive family history of CF, therefore the earliest possible


diagnosis can be made via Pre-implantation genetic testing.

- See above for further details


Q:1124 1124. A 39yo woman in her 36th week GA with acute abdominal pain is rushed
for immediate delivery.
Her report: BP=110/60mmHg, Hgb=low, bilirubin=22 (N=upto 17), AST=35,
Plt=60, APTT=60 (upto 30 is N), PT=30, (N=13-14)
Fibrinogen=0.6 (low). What is the cause?
a. Pregnancy induced hypertension
b. DIC (preg is hypercoag state)
c. HELLP syndrome
d. Acute fatty live
e. Obstetric cholestasis

Clincher(s) 36 week, low hb, bil high(17), ast- 35(n),plt low, apt- 35-45, PT-?

126
OBS GYN-System Wise 1700-by Sush and Team. 2016
Susmita, Asad, Manu, Saima, Zohaib, Savia, Shanu, Mona, Manisha, Sitara, Samreena, Sami and Komal

FIBRINOGEN-
A PIH- BP NORMAL
B DIC, FGN – LOW, APTT HIGH, INC BIL,
C HELLP- PLT LOW, LIVER ENZ N, PT, APTT DERANGED
D Liver enzymes no that high
E Bil not that high
KEY B
Additional
Information
Reference reason : low fibrinogen,increased bilirubin ,normal AST , prolonged PT,PTT

DIC

The diagnosis of DIC should include both clinical and laboratory information:

• (PT) elevated.

• (aPTT) elevated.

• Platelet counts in DIC are typically low

• Fibrinogen level low.

Confirmatory tests :

• The D-dimer test gives strong evidence of DIC.

• Fibrin degradation products (FDPs) are helpful but can occur in


other conditions such as deep vein thrombosis (DVT)

• In acute DIC, PT and aPTT are prolonged, and the platelet count
and fibrinogen decrease. D-dimer, FDP, and fibrin monomer levels
are elevated

Management :

The cornerstone of the the management of DIC is treatment of the underlying


condition. Thus, infection will need antibiotics, and obstetric complications
may need intervention

plasma and /or platelet transfusion

Conditions that may be complicated by DIC include:

* Infections, especially septicaemia, Escherichia coliO157, typhoid fever, Rocky


Mountain spotted fever and parasites. The rash of meningococcal septicaemia
is classical.

* Malignancy, especially leukaemias.

* Major trauma including crush syndrome and, occasionally, burns. * Some

127
OBS GYN-System Wise 1700-by Sush and Team. 2016
Susmita, Asad, Manu, Saima, Zohaib, Savia, Shanu, Mona, Manisha, Sitara, Samreena, Sami and Komal

connective tissue disorders including antiphospholipid syndrome. *
Complications of pregnancy including the placental problem of placental
abruption, amniotic fluid embolism, severe hypertension of pregnancy with
fulminating pre-eclampsia and HELLP syndrome. A retained dead fetus tends
to produce a thrombotic rather than a haemorrhagic state.

* Incompatible blood transfusion.

* Heat stroke.

* Dissecting aortic aneurysm.

* Some snake bites

If DIC is suspected then clotting screen tests are followed by confirmation:

- Prothrombin time (PT) elevated.

- Activated partial thromboplastin time (aPTT) elevated.

- Platelet counts in DIC are typically low, especially in acute sepsis-associated


DIC, but may be increased in malignancy-associated chronic DIC.

- Fibrinogen level low. If two results are positive,DIAGNOSIS is possible; if three


are positive, it is likely; if all four are positive, it is extremely likely.

- Confirmatory tests look for evidence of the simultaneous formation of


thrombin and plasmin.

- The D-dimer test gives strong evidence of DIC.

- Fibrin degradation products (FDPs) are helpful but can occur in other
conditions such as deep vein thrombosis (DVT) and, in severe disease, they
may be negative.

- In acute DIC, PT and aPTT are prolonged, and the platelet count and
fibrinogen decrease. D-dimer, FDP, and fibrin monomer levels are elevated
Dr Khalid/Rabia
Q:1125 1125. A 36wk pregnant woman presents with sudden onset of uterine pain and
bleeding, uterus is
tender, no prv LSCS. What is the most appropriate cause?
a. Preeclampsia
b. DIC
c. Placental abruption
d. Placental previa
e. Ectopic pregnancy
f. Missed abortion
g. Ectropion

Clincher(s) SUDDEN PAIN BLEEDING IN THIRD TRIMESTER- APH

128
OBS GYN-System Wise 1700-by Sush and Team. 2016
Susmita, Asad, Manu, Saima, Zohaib, Savia, Shanu, Mona, Manisha, Sitara, Samreena, Sami and Komal

A PREECLAMPSIA-PAIN BLEEDING
B DIC, NO CLINCHER
C PLACENTAL ABRUPTION- PAIN BLEEDING
D PP- PAIN LESS
E ECTOPIC, GA IS HIGH E, F G IRRELEVENT
KEY C
Additional
Information
Reference
Dr Khalid/Rabia key : C

reason : a case of antepartum haemorrhage in the third trimester >>> pain +


bleeding >>> abruptin ( pain is common with abruption) ( full topic discussed
earlier)

- Bleeding during the third trimester is either painful or painless bleeding.

- Painful bleeding points to Placental Abruption while painless bleeding points


to Placenta Previa

- In this case, the patient is experiencing painful vaginal bleeding in the third
trimester, so this appears to be the case of Placental Abruption

- The investigation of choice in this case is an Ultrasound

- Risk Factors for Abruption

- Hypertension

- Smoking

- Multiple Pregnancy

- Cocaine/Amphetamine Use

- Increased Maternal Age

- Trauma to the abdomen

- Polyhydramnios

- Investigation

- Diagnosis is clinical but U/S is done to exclude Placenta Praevia and to check
the well-being of the baby.

- Management

- Always admit the patient to hospital for assessment and management. Phone
999/112/911 if there are any major concerns regarding maternal or fetal well-
being.

129
OBS GYN-System Wise 1700-by Sush and Team. 2016
Susmita, Asad, Manu, Saima, Zohaib, Savia, Shanu, Mona, Manisha, Sitara, Samreena, Sami and Komal

- The mainstays of management are resuscitation and accurate diagnosis of
the underlying cause.

- Severe bleeding or fetal distress: urgent delivery of the baby, irrespective of


gestational age.

- Admit to hospital, even if bleeding is only a very small amount. There may be
a large amount of concealed bleeding with only a small amount of revealed
vaginal bleeding.

- No vaginal examination should be attempted, at least until a placenta praevia


is excluded by ultrasound. It may initiate torrential bleeding from a placenta
praevia.

- Resuscitation can be inadequate because of underestimation of blood loss


and misleading maternal response. A young woman may maintain a normal
blood pressure until sudden and catastrophic decompensation occurs.

- Take blood for FBC and clotting studies. Crossmatch, as heavy loss may
require transfusion.

- Gentle palpation of the abdomen to determine the gestational age of the


fetus, presentation and position.

- Fetal monitoring.

- Arrange urgent ultrasound.

- With every episode of bleeding, a rhesus-negative woman should have a


Kleihauer test and be given prophylactic anti-D immunoglobulin

- Complications

- Premature Labour

- DIC

- Renal Tubular Necrosis

- PPH

- Placenta Accreta

- Points to look for diagnosis of Abruption

- Shock is out of proportion from visible blood loss

- Constant pain

- Tender tense uterus

- Fetal heart sounds absent/distressed

130
OBS GYN-System Wise 1700-by Sush and Team. 2016
Susmita, Asad, Manu, Saima, Zohaib, Savia, Shanu, Mona, Manisha, Sitara, Samreena, Sami and Komal

- Coagulation problems like DIC


Q:1126 1126. A 28wk pregnant woman presents with uterine bleeding after sexual
intercourse. What is the most appropriate cause?
a. Preeclampsia
b. DIC
c. Placental abruption
d. Placental previa
e. Ectopic pregnancy
f. Missed abortion
g. Ectropion

Clincher(s) 28 WEEKS, UTERINE BLEEDING AFTER INTERCOURSE
A PRE ECLAMPSIA- NO BLEEDING
B DIC NO POINTERS
C PA- NO H/O PAIN
D PP- (PP after 24 weeks of preg more common)
E ECTROPION, COMMONEST CAUSE OF PCB, bleeding on contact
KEY G H/O POST COITAL BLEED
Additional
Information
Reference
Dr Khalid/Rabia key : G

reason :postcoital bleeding >>> mostly ectropion

Causes of postcoital bleeding

• Infection,Trauma.

• Cervical ectropion - especially in those women taking the combined


oral contraceptive pill (COCP).

• Cervical or endometrial polyps.

• Vaginal cancer , Cervical cancer - usually apparent on speculum


examination.

Ectropion

• It is most commonly seen in teenagers, during pregnancy and in


women on combined hormonal contraception.

• It is generally an asymptomatic condition but patients occasionally


present with bleeding or excessive discharge.

• Once a normal cervical smear has been confirmed, it is actively

131
OBS GYN-System Wise 1700-by Sush and Team. 2016
Susmita, Asad, Manu, Saima, Zohaib, Savia, Shanu, Mona, Manisha, Sitara, Samreena, Sami and Komal

managed only if there are symptoms. Over time, vaginal acidity
promotes metaplasia to squamous epithelium when the symptoms
will disappear.

• After stopping any oestrogen-containing contraceptive, treatment


options are controversial but include diathermy, cryotherapy,
surgery with laser treatment and microwave therapy.


- The cervix enlarges under the influence of oestrogen and as a result the
endocervical canal is everted. It is seen on examination as a red ring around
the os and is so common as to be regarded as normal. - It is most commonly
seen in teenagers, during pregnancy and in women on combined hormonal
contraception.

- This seems to be the most appropriate among the given options because the
patient is 28 weeks pregnant and had sex at this point.

- There are no signs of pre-eclampsia, DIC, Placental abruption and Placenta


Praevia. Ectopic pregnancy and missed abortion do not present at this stage.



Q:1127 1127. A 6wk pregnant woman presents with abdominal pain. She has prv hx of
PID. What is the most likely dx?
a. Preeclampsia
b. DIC
c. Placental abruption
d. Placental previa
e. Ectopic pregnancy
f. Missed abortion
g. Ectropion

Clincher(s) 6 WEEKS PREGNANT, ABDOMINAL PAIN, PID (PID causes ectopic)
A PREECLAMPSIA, HIGH BP >20
B DIC-
C PLACENTAL ABRUPTION- 1ST TM
D
E
KEY E- H/O PID, ABD PAIN 6 WEEKS
Additional
Information
Reference
Dr Khalid/Rabia key : E

reason : Ectopic: hx of PID + early pregnancy + abdominal pain

132
OBS GYN-System Wise 1700-by Sush and Team. 2016
Susmita, Asad, Manu, Saima, Zohaib, Savia, Shanu, Mona, Manisha, Sitara, Samreena, Sami and Komal

Ectopic pregnancy

The majority of ectopic pregnancies occur in the ampullary or isthmic portions


of the Fallopian tubes

Risk factors

• IUCD

• PID

• in tubes that have been divided in a sterilisation operation and


where they have been reconstructed to reverse on

Presentation :

• 30% of ectopic pregnancies present before a period has been


missed.

• Abdominal pain.

• Pelvic pain.

• Amenorrhoea or missed period.

• Vaginal bleeding (with or without clots)

Investigations:

• The most accurate method to detect a tubal pregnancy is


transvaginal ultrasound.

• hCG levels are performed in women with pregnancy of unknown


location who are clinically stable

Management :

• Medical management: systemic methotrexate is offered first-line


to those women who are able to return for follow-up and who have
the following:

• No significant pain,Unruptured ectopic pregnancy with an adnexal


mass <35 mm and no visible heartbeat, No intrauterine pregnancy
seen on ultrasound scan ,Serum hCG <1500 IU/L.

• Surgical management: offered to those women who can not


return for follow-up after methotrexate or to those who have any
of the following:

• Significant pain.

• Adnexal mass ≥35 mm.

133
OBS GYN-System Wise 1700-by Sush and Team. 2016
Susmita, Asad, Manu, Saima, Zohaib, Savia, Shanu, Mona, Manisha, Sitara, Samreena, Sami and Komal

• Fetal heartbeat visible on scan.

• Serum hCG level ≥5000 IU/L.

• A laparoscopic approach is preferable. A salpingectomy should be


performed, unless the woman has other risk factors for infertility, in
which case a salpingotomy should be undertaken.

- Symptoms may develop at any time between 4 and 10 weeks of pregnancy. *


- Pain on one side of the lower tummy (abdomen). It may develop sharply, or
may slowly get worse over several days. It can become severe. * Vaginal
bleeding often occurs, but not always. It is often different to the bleeding of a
period. For example, the bleeding may be heavier or lighter than a normal
period. The blood may look darker. However, you may think the bleeding is a
late period.

* Shoulder tip pain due to irritation of the diaphragm.

- Risk factors

- Women over the age of 35

- Previous ectopic pregnancy

- Scarring, damage or other structural abnormality of the fallopian tube

- PID (This patient in question has a history of PID)

- IUCDs

- Investigations

- Urine pregnancy test

- Transvaginal U/S

- Beta-HCG

- Diagnostic Laproscopy

- Treatment options

- Ruptured ectopic pregnancy

Emergency surgery is needed if a Fallopian tube ruptures with heavy bleeding.


The main aim is to stop the bleeding. The ruptured Fallopian tube and remnant
of the early pregnancy are then removed. The operation is often life-saving.

- Early ectopic pregnancy - before rupture

Ectopic pregnancy is most often diagnosed before rupture. Your doctor will
discuss the treatment options with you and, in many cases, you are able to

134
OBS GYN-System Wise 1700-by Sush and Team. 2016
Susmita, Asad, Manu, Saima, Zohaib, Savia, Shanu, Mona, Manisha, Sitara, Samreena, Sami and Komal

decide which treatment is best for you. These may include the following:

* Surgery. Removal of the tube (either the whole tube or part of it) and the
ectopic pregnancy is most commonly performed by keyhole surgery (a
laparoscopic operation). Removal of the Fallopian tube containing the ectopic
pregnancy (salpingectomy) is usually performed if the other tube is healthy.
Removal of only a section of the tube with the ectopic pregnancy in it
(salpingotomy) is usually performed if the other tube is unhealthy; for
example, scarred from a previous infection. However, many women with an
ectopic pregnancy do not need to have an operation.

* Medical treatment. Medical treatment of ectopic pregnancies is now more


common and avoids the need for surgery. A medicine called methotrexate is
often given, usually as an injection. It works by killing the cells of the
pregnancy growing in the Fallopian tube. It is normally only advised if the
pregnancy is very early. The advantage is that you do not need an operation.
The disadvantage is that you will need close observation for several weeks
with repeated blood tests andSCANS to check it has worked. You will need to
have a blood test for hCG every 2-3 days until your levels are low. Scans are
usually repeated weekly. Methotrexate can cause side-effects which include
nausea and vomiting in some women. It can be common for some abdominal
pains to develop 3-7 days after having methotrexate.


Q:1129 1129. A 25yo lady at her 28th week GA came for check up. Her
BP=160/95mmHg, protein in urine=6g/d. (more than 0.3g)
What is the most likely dx?
a. Essential HTN
b. Gestational HTN
c. Chronic HTN
d. Preeclampsia

Clincher(s) HYPERTENSION, 2ND TM PROTIEN IN URINE
A
B
C
D
E
KEY D
Additional
Hypertension in pregnancy
Information
NICE published guidance in 2010 on the management of hypertension in
pregnancy. They also made recommendations on reducing the risk of
hypertensive disorders developing in the first place. Women who are at high
risk of developing pre-eclampsia should take aspirin 75mg od from 12 weeks

135
OBS GYN-System Wise 1700-by Sush and Team. 2016
Susmita, Asad, Manu, Saima, Zohaib, Savia, Shanu, Mona, Manisha, Sitara, Samreena, Sami and Komal

until the birth of the baby. High risk groups include:

• hypertensive disease during previous pregnancies


• chronic kidney disease
• autoimmune disorders such as SLE or antiphospholipid syndrome
• type 1 or 2 diabetes mellitus


The classification of hypertension in pregnancy is complicated and varies.
Remember, in normal pregnancy:

• blood pressure usually falls in the first trimester (particularly the


diastolic), and continues to fall until 20-24 weeks
• after this time the blood pressure usually increases to pre-pregnancy
levels by term


Hypertension in pregnancy in usually defined as:

• systolic > 140 mmHg or diastolic > 90 mmHg


• or an increase above booking readings of > 30 mmHg systolic or > 15
mmHg diastolic


After establishing that the patient is hypertensive they should be categorised
into one of the following groups

Pregnancy-induced hypertension
Pre-existing hypertension (PIH, also known as gestational Pre-eclampsia
hypertension)
Hypertension (as defined above)
occurring in the second half of Pregnancy-induced
A history of hypertension
pregnancy (i.e. after 20 weeks) hypertension in
before pregnancy or an
association with
elevated blood pressure >
No proteinuria, no oedema proteinuria (> 0.3g / 24
140/90 mmHg before 20 weeks
hours)
gestation
Occurs in around 5-7% of

pregnancies Oedema may occur but
No proteinuria, no oedema
is now less commonly

Resolves following birth (typically used as a criteria
Occurs in 3-5% of pregnancies
after one month). Women with PIH
and is more common in older
are at increased risk of future pre- Occurs in around 5% of
women
eclampsia or hypertension later in pregnancies
life


Reference

136
OBS GYN-System Wise 1700-by Sush and Team. 2016
Susmita, Asad, Manu, Saima, Zohaib, Savia, Shanu, Mona, Manisha, Sitara, Samreena, Sami and Komal

Dr Khalid/Rabia key : D

reason : Pre-eclamsia: HTN + protienuria +/- edema >>> preeclampsia

Pre-eclampsia

Pre-eclampsia is pregnancy-induced hypertension in association with


proteinuria (>0.3 g in 24 hours) with or without oedema.

Severe pre-eclampsia is defined as diastolic blood pressure of at least 110 mm


Hg, or systolic blood pressure of at least 160 mm Hg, and/or symptoms, and/or
biochemical and/or haematological impairment

Presentation:

• New hypertension ,New and/or significant proteinuria.

• Other clinical features of severe pre-eclampsia include:

Severe headache - usually frontal.

Platelet count falling to below 100 x 109/

Abnormal liver enzymes (ALT or AST rising to above 70 IU/L).

HELLP syndrome: H (haemolysis) EL (elevated liver enzymes) LP (low


platelets).

Investigations :

• Urinalysis

• Frequent monitoring of FBC, LFTs, renal function, electrolytes and


serum urate

• Clotting studies if there is severe pre-eclampsia or


thrombocytopenia

• 24-hour urine collections for protein quantification and creatinine


clearance.

• Assessment of fetus -ultrasound

Management:

• Control Blood pressure

• Prevention of seizures >>> Magnesium sulfate

• Fluid balance

• Delivery

137
OBS GYN-System Wise 1700-by Sush and Team. 2016
Susmita, Asad, Manu, Saima, Zohaib, Savia, Shanu, Mona, Manisha, Sitara, Samreena, Sami and Komal

- Pre-eclampsia: BP > 140/90 and 300 mg proteinuria in 24-hour urine
collection

- Mild to moderate: BP <160/110 with significant proteinuria and no maternal


complications

- Severe: BP >160/110 with significant proteinuria or if maternal complications


occur

- Risk factors

- Previous history of pre-eclampsia

- Maternal age > 40 years

- Family History

- DM, HTN, Renal Disease

- This is a case of severe pre-eclampsia as the BP of the patient in question


suggests. This should be managed along the following lines.

- Anti-hypertensives to bring BP down to less than 160/110

- IV Hydralazine is the first choice

- Labetolol

- MgSO4 to prevent eclampsia

- CTG and U/S to monitor the baby

NOTE: If less than 34 weeks gestation, give steroids to help production of


surfactant.

- Complications

- Eclampsia

- HELLP Syndrome

- DIC

- Renal Failure

- Placental Abruption


Q:1130 1130. A 32yo woman has a hx of spontaneous abortions at 6wks, 12wks, and
20wks. She is now keen
to conceive again. Which of the following would you prescribe for the next
pregnancy?
a. MgSO4

138
OBS GYN-System Wise 1700-by Sush and Team. 2016
Susmita, Asad, Manu, Saima, Zohaib, Savia, Shanu, Mona, Manisha, Sitara, Samreena, Sami and Komal

b. Aspirin
c. Warfarin
d. Mefenemic acid
e. Heparin

Clincher(s)
A MgSo4
B Aspirin (started in first day of conception, and then heparin in phospholipid
syndrone)
C Warfarin
D Mefenamic Acid
E Heparin
KEY B
Additional
Information
Reference
Dr Khalid/Rabia key : B

reason : antiphospholipid syndrome is the most important treatable cause of


recurrent miscarriage >>>> Rx >>> Asprin

recurrent miscarriage

def. >>> the loss of three or more consecutive pregnancies from the time of
conception until 24 weeks of gestation in the UK.

Aetiology :

• Antiphospholipid syndrome >> investigation : anticardiolipin


antibodies , lupus anticoagulant >>> Rx : Aspirin

• structural >>> uterine anomalies , fibroids , cervical incompetence


( late miscarriage) >>> investigation : pelvic u/s >>> Rx in cervical
incompetence : cerclage (complication : uterine rupture)

• endocrine >>> PCOS , uncontrolled diabetes

• immune

• thrombophilia

• genetic abnormality

• infection >>> bacterial vaginosis in the first trimester is a risk of


second trimester miscarriage and preterm delivery

- Recurrent miscarriage is defined as the loss of three or more consecutive


pregnancies. - Miscarriage, the most common complication of pregnancy, is
the spontaneous loss of a pregnancy before the fetus has reached viability. The
term therefore includes all pregnancy losses from the time of conception until

139
OBS GYN-System Wise 1700-by Sush and Team. 2016
Susmita, Asad, Manu, Saima, Zohaib, Savia, Shanu, Mona, Manisha, Sitara, Samreena, Sami and Komal

24 weeks of gestation in the UK. - Antiphospholipid syndrome (APS):

- This is the most important treatable cause of recurrent miscarriage.

- Investigations

- Antiphospholipid antibodies:

The presence of these is associated with early miscarriages and maternal


morbidity and is referred to as primary APS. There is requirement for two tests
at least six weeks apart showing either lupus anticoagulant or anticardiolipin
antibodies at significant levels.

- Women with recurrent first-trimester miscarriage and all women with one or
more second-trimester miscarriages should be screened for antiphospholipid
antibodies before pregnancy. And pelvic ultrasound to assess uterine
anatomy.

- Women with second-trimester miscarriage should be screened for inherited


thrombophilias including factor V Leiden, factor II (prothrombin) gene
mutation and protein S.

- All women with recurrent first-trimester miscarriage and all women with one
or more second-trimester miscarriages should have pelvic ultrasound to assess
uterine anatomy.

- If uterine anomalies are detected then further investigations, such as


hysteroscopy and/or laparoscopy, may be required.

- Management

- General advice

- Reassurance should be given about the high probability of a successful


outcome. In a large trial that included women with 4.2 consecutive
miscarriages and an average age of 32.7 years, the placebo group was shown
to have a live birth rate of 65%.

Pharmacological treatment

- In primary APS patients, heparin combined with low-dose aspirin improves


live birth rate to 70%.[5] There ARE only limited data supporting the use of
heparin in women without APS

- There is some evidence suggesting that use of metformin during pregnancy is


associated with a reduction in the miscarriage rate in women with polycystic
ovarian syndrome

- However, the RCOG DOES NOT recommend its use in pregnancy at present
until further randomised prospective study results are available to provide

140
OBS GYN-System Wise 1700-by Sush and Team. 2016
Susmita, Asad, Manu, Saima, Zohaib, Savia, Shanu, Mona, Manisha, Sitara, Samreena, Sami and Komal

adequate evidence of safety and efficacy of its use.

- A Cochrane review found evidence of benefit for progestogen therapy in


women with a history of recurrent miscarriage. There was no statistically
significant difference in rates of adverse effects.

- However, there is currently a large randomised, double-blind, placebo-


controlled multicentre trial underway - the Progesterone in recurrent
miscarriage (PROMISE) study - which aims to provide a definitive answer
regarding progesterone use in women with recurrent miscarriages.

Surgical

- Cervical cerclage is used where cervical incompetence is suspected. However,


it is overdiagnosed as a cause of second-trimester miscarriage. The cerclage
procedure also carries a risk of stimulating uterine contractions.

- Cerclage benefit increases as the cervix shortens to less than 25 mm. It has
also been shown to be beneficial in those women with a shortened cervical
length of less than 25 mm.


Q:1132 1132. A 42yo woman who smokes 20 cigarettes/d presents with complains of
heavy bleeding and prolonged menstrual period. What is the most appropriate
tx for her?
a. Tranexemic acid (antifibrinolytic)
b. COCP
c. Mefenemic acid
d. IUCD
e. Norethisterone

Clincher(s)
A Tranexemic acid
B COCP
C Mefenamic Acid
D IUCD
E Norethisterone
KEY D
Additional
Information
Reference
Dr Khalid/Rabia reason : smoker >>> so no use of COCP ,,, heavy bleeding >>> IUCD ( FULL
TOPIC DICUSSED EARLIER)

The orginial key says IUCD (D) but it is not the first choice in

141
OBS GYN-System Wise 1700-by Sush and Team. 2016
Susmita, Asad, Manu, Saima, Zohaib, Savia, Shanu, Mona, Manisha, Sitara, Samreena, Sami and Komal

heavy/prolonged menstrual periods.

1.Mirena Coil is.

2. The second line is Tranexamic Acid.

3. Third line is COCPs.

4. Fourth line is Endometrial ablation or Hysterectomy

(If there is no desire to conceive). IUCD, according to patient.info actually


causes heavy or painful periods. The logical choice in this case would be
Tranexamic Acid since Mirena is not mentioned in the given options.

1. merena or IUS> Tranexamic> ICOCP>IV progesterone> endo ablation>


hysterectomy

Q:1133 1133. A 17yo senior school girl with complain of prolonged irregular menstrual
period and heavy blood losses. What is the most appropriate tx for her?
a. Mefenemic acid
b. COCP
c. POP
d. IUCD
e. Mirena
First line for regular menorrhagagia – mefenemic acid (prostogl)
If irregular: COCP
Clincher(s)
A
B
C
D
E
KEY
Additional
Information
Reference
Dr Khalid/Rabia KEY : B

reason : irregular menses + heavy bleeding >>> COCP can treat both (Full topic
discussed earlier)


- COCPs are widely used for irregular menstrual periods.

142
OBS GYN-System Wise 1700-by Sush and Team. 2016
Susmita, Asad, Manu, Saima, Zohaib, Savia, Shanu, Mona, Manisha, Sitara, Samreena, Sami and Komal

- They also carry the advantage of causing a decrease in bleeding and
menstrual pain (can be used for dysmenorrhea and menorrhagia)


Q:1134 1134. A 32yo presents with heavy blood loss, US: uterine thickness>14mm.
What is the most
appropriate tx for her?
a. Mefenemic acid
b. COCP
c. POP
d. IUCD
e. IU system (mirena)
Endo hyperplasia: Mirena; causes atrophy(?) of endometrium. Avoided if ca
Clincher(s)
A
B
C
D
E
KEY
Additional
Information
Reference
Dr Khalid/Rabia key :E

reason : heavy bleeding >>> IUS is the first line (full topic discussed earlier)

- Mirena, also known as Intrauterine System, is a Levonorgestrel-containing


coil which is inserted into the uterus.

- Local effect: reversible endometrial atrophy, makes implantation less likely


and periods lighter (20% reversible amenorrhea)

- Less risk of ectopic pregnancy

- Risk of STDs is reduced

- May benefit women with endometriosis, adenomyosis, fibroids or


endometrial hyperplasia

- NOTE: Avoid if breast cancer.


Q:1135 1135. A 37yo woman presents with heavy bleeding. Inv show subserosal
fibroid=4cm and intramural fibroid=6cm. Which is the most appropriate tx?
a. UAE
b. Abdominal hysterectomy

143
OBS GYN-System Wise 1700-by Sush and Team. 2016
Susmita, Asad, Manu, Saima, Zohaib, Savia, Shanu, Mona, Manisha, Sitara, Samreena, Sami and Komal

c. Hysteroscopic Myomectomy
d. Vaginal Hysterectomy
e. Abdominal myomectomy

Clincher(s)
A
B
C
D
E
KEY
Additional
Information
Reference
Dr Khalid/Rabia key : e

reason :Fibroids: abdominal myomectomy is the best alternative for


hysterectomy in patients who want to keep their fertility ,, pt is 37 ys and this
procedure can treat both intramural and subserosal fibroids (full topic
discussed earlier)

- Fibroids are responsive to estrogen and therefore increase in size, which in


turn increases the size of the uterus.

- Other symptoms

- Pelvic pain (Compression on to adjacent structures)

- Infertility/Recurrent Miscarriages

- Pelvic Mass

- The investigation of choice is an U/S

- Management

- Mirena Coil is the first choice if the fibroids are not big enough to restrict its
insertion.

- If < 3 cm: Trial of pharmacologic treatment first (Tranexamic Acid) first

- If it fails and uterus is not bigger than 10-week pregnancy, do endometrial


ablation (UAE) and wishes to retain uterus and/or wants to avoid surgery

- Go for Uterine Artery Embolization

- If > 3 cm and wishes to retain uterus, go for a hysteroscopic myomectomy or


a myomectomy (or If the above fails)

144
OBS GYN-System Wise 1700-by Sush and Team. 2016
Susmita, Asad, Manu, Saima, Zohaib, Savia, Shanu, Mona, Manisha, Sitara, Samreena, Sami and Komal

-

NOTE: This patient has a subserosal fibroid so an abdominal approach should


be adopted
Q:1136 1136. A woman with sickle cell disease complains of heavy menstrual blood
loss. What is the most
appropriate tx?
a. COCP
b. Mirena
c. Depot provera
d. Copper IUS
e. Transdermal patch

Clincher(s)
A
B
C Reduces incidence - sickle cell crisis, - OHCS- pg 304
D Copper IUs can cause infection
E
KEY C
Additional
Information
Reference
Dr Khalid/Rabia key : c

reason : sickle cell disease :

intrauterine devices are not recommended, as they may be associated with


uterine bleeding and infection.

Combined hormonal methods are not recommended , because of the risk of


thromboembolism in sickle cell patients

Depot contraceptive (Depo-Provera®) is safe and has been found to improve


the blood picture and reduce pain crises.[15] (amenorrhea)

The choice of contraceptive method needs to be considered carefully. The coil


(intrauterine contraceptive device) may cause particularly heavy painful
periods. The use of injectable contraceptives (such as Depo-Provera) has been
reported to provide some protection against sickling episodes.


v
Q:1150 1150. A 25yo woman presents with a painful shallow ulcer on the vulva. What
inv has to be done?
a. HSV antibodies
b. Syphilis serology
c. Swab for hemophilus ducreyi

145
OBS GYN-System Wise 1700-by Sush and Team. 2016
Susmita, Asad, Manu, Saima, Zohaib, Savia, Shanu, Mona, Manisha, Sitara, Samreena, Sami and Komal

d. Urine culture
e. Blood culture

Clincher(s)
A
B
C
D
E
KEY
Additional Both Painful multiple ulcer:
Information
HSV- systemic symtoms present
Hemophilus ducreyi – no sym symtoms
Reference key : c

reason : shallow painful ulcer >> chancroid

chancroid:

Chancroid is a sexually transmitted disease (STD) caused by haemophilus


ducreyi characterized by painful shallow with soft ragged margins necrotizing
genital ulcers that may be accompanied by inguinal lymphadenopathy.

- The symptoms point towards the causative organism being Haemophilus


Ducreyi as it causes painful shallow ulcers.

- Mnemonic: YOU CRY WITH DUCREYI

- The other conditions cannot explain the symptoms above as they do not
cause painful shallow ulcers. HSV cause vesicles while Syphilis causes painless
ulcers.

- Urine and Blood Culture are not required for the same reason as stated
above.


Dr Khalid/Rabia
Q:1164 1164. A 17yo girl comes to see her GP after having unprotected sex 2d ago.
She asks if her GP can explain to her how this prescribed procedure would
work by helping her not to get pregnant.
a. It helps to prevent implantation
b. It helps in preventing or delaying ovulation
c. It causes an early miscarriage
d. It releases progesterone and stops ovulation
e. It causes local enzymatic reaction

Clincher(s)

146
OBS GYN-System Wise 1700-by Sush and Team. 2016
Susmita, Asad, Manu, Saima, Zohaib, Savia, Shanu, Mona, Manisha, Sitara, Samreena, Sami and Komal

A
B
C
D
E
KEY
Additional
Information
Reference
Dr Khalid/Rabia key:A

reason : CU IUD prevents implantation

emergency contraception: OHCS- 299 as per protocol seq:

• IUCD >>> inhibitory effect on both fertilisation and implantation


( direct toxicity effects of the copper on both ovum and sperm). up
to five days after UPSI (prevents 99% - most effective)

• ulipristal acetate >>> inhibits ovulation. effective up to 120 hours


after UPSI. Pregnancy or suspected pregnancy should be excluded
before use

• Progestogen-only-PCP - levonorgestrel >>> early in the


cycle inhibits ovulation. later in the cycle, it is unclear how it has its
effect. use within 72 hours of UPSI


Q:1573 1573. A woman became acutely SOB in the recovery bay and is coughing after
GA. Auscultation:
reduced air entry at the right lung base and diffuse wheeze. Observation:
HR=88bpm,
BP=112/76mmHg, temp=37.8C and sat=91% in air. Choose among the options
which C-section
complication has she developed?
a. Aspiration pneumonitis
b. Spontaneous pneumothorax
c. Endometritis
d. Pulmonary embolism
e. Tension pneumothorax

Clincher(s)
A
B
C

147
OBS GYN-System Wise 1700-by Sush and Team. 2016
Susmita, Asad, Manu, Saima, Zohaib, Savia, Shanu, Mona, Manisha, Sitara, Samreena, Sami and Komal

D
E
KEY A
Additional
Information
Reference
Dr Khalid/Rabia Key: Aspiration pneumonitis (A)
Reason: Everything aside, look at the presentation of this illness. The woman
presented acutely after GA and with different vitals to the great mimicker in
this period of illness which would be Pulmonary Embolism. Vitally stable with
mildly reduced SpO2 and coughs after GA. The wheeze also points to
aspiration. Pneumothorax wouldn’t present this way and neither would a
tension pneumothorax. Endometritis is just stupid here.

Q:1578 1578. A 32yo woman of 38wks GA attends the antenatal day unit with pain in
the suprapubic area that radiates to the upper thighs and perineum. It is worse
on walking. Her urine dipstick showed a trace of protein but no white cells,
nitrates or blood. What s the ost likel d ?
a. Braxton hicks contractions
b. Round ligament stretching
c. Symphasis pubis dysfunction
d. Labor
e. Complicated femoral hernia

Clincher(s)
A
B
C
D
E
KEY
Additional
Information
Reference
Dr Khalid/Rabia Key: Symphysis Pubic Dysfunction (C)
Reason: It is a condition that causes excessive movement of the pubic
Symphysis and pain or discomfort in the pelvic region. The pain in the supra-
pubic area radiating to the legs and perineum combined with worse symptoms
on walking would point to only one thing – Symphysis Pubis Dysfunction. The
Urine exam is a detractor. Braxton-Hick’s contractions or Labour would not
present this way, neither in intensity nor radiation. Round ligament stretching
doesn’t make sense here and femoral hernia is not associated with these
symptoms.

Q: 1610 1610. A 32yo woman of 40wks gestation attends the antenatal day unit with
sudden onset epigastric pain with nausea and vomiting. She is clinically

148
OBS GYN-System Wise 1700-by Sush and Team. 2016
Susmita, Asad, Manu, Saima, Zohaib, Savia, Shanu, Mona, Manisha, Sitara, Samreena, Sami and Komal

jaundiced. Her biochemistry results show a raised BiliRubin, abnormal liver
enzymes, high uric acid and hypoglycemia. What’s the most likely
dx?
a. Acute fatty liver of pregnancy
b. Obstetric cholestasis
c. Cholecystitis
d. HELLP syndrome
e. Acute hepatitis

Clincher(s)
A
B No pruritis
C No gall stones
D No thrombocytopenia, amylodisis
E
KEY
Additional
Information
Reference
Dr Khalid/Rabia Key: Acute Fatty Liver of Pregnancy (AFLP) (A)
Reason: Pain, nausea, vomiting, jaundice, fever with elevated liver enzymes
and bilirubin is clinically indicative of AFLP. Also can have elevated INR, TLC and
hypoglycaemia. It isn’t cholestasis because there is no pruritis, cholecystitis
isnt the answer because there is no history of evidence of gallstones, HELLP
isn’t the answer because there isn’t any hemolysis or thrombocytopenia and
acute hepatitis would present subclinically or with very less symptoms like
diarrhoea and vomiting alongwith clinical history of food poisoning.
Discussion:
Epidemiology:
* It is a rare condition with an incidence of 5 in 100,000 pregnancies.
* Acute fatty liver of pregnancy (AFLP) tends to occur in late pregnancy.
* Risk factors include first pregnancies, pre-eclampsia, twin pregnancies and
male fetuses.
* It may be associated with a mutant gene producing a defect in mitochondrial
fatty acid oxidation and infants born to mothers with AFLP should be screened
for defects in this system.
Presentation:
* This usually presents acutely with nausea, vomiting and abdominal pain,
fevers, headache and pruritus, beginning typically at about 35 weeks of
gestation but can occur much earlier. It may also appear immediately after
delivery.
* Jaundice appears soon after onset of symptoms and can become intense in a
large proportion of patients. Fulminant liver failure may follow.
Investigations:
* The white cell count is often elevated. There may also be neutrophilia and
thrombocytopenia.

149
OBS GYN-System Wise 1700-by Sush and Team. 2016
Susmita, Asad, Manu, Saima, Zohaib, Savia, Shanu, Mona, Manisha, Sitara, Samreena, Sami and Komal

* Liver transaminases are moderately high.
* Raised serum bilirubin.
* Abnormal clotting with coagulopathy (prolongation of prothrombin and
partial thromboplastin times with depression of fibrinogen levels).
* Biopsy would be diagnostic but coagulation problems often preclude it.
CT/MRI scanning may show reduced attenuation in the liver.
Management:
Consider early delivery, as the condition usually resolves afterwards with
complete recovery. Supportive ITU care is frequently required.
Complications:
AFLP is a life-threatening condition with a reported 1.8% maternal and 23%
fetal mortality rate. Serious complications include: * Disseminated
intravascular coagulation (DIC) and gastrointestinal bleeding.
* Hepatic coma. * Acute kidney injury. * Pancreatitis. * Hypoglycaemia.


Q:1615 1615. A 29yo woman presents to her GP with troublesome heavy periods. The
med tx that she has tried have made little difference. She is known to have
large uterine intramural fibroids. You confirm that she is currently trying for
more children. Select the most appropriate management for menorrhagia in
this pt?
a. Danazol (anti androgen- causes amenorrhea)
b. Endometrial ablation (older women- family complete)
c. Hysterectomy
d. Hysteroscopic resection of fibroid (small fibroid- eg submucosal)
e. Myomectomy

Clincher(s)
A
B
C
D
E
KEY
Additional
Information
Reference
Dr Khalid/Rabia Key: Myomectomy (E)
Reason: She is currently trying for more children, hysterectomy will not be
useful in this case obviously. OHCS pg. 276 states that chance of subsequent
pregnancies is better after myomectomy and it is the best treatment in this
case. Endometrial ablation will not affect the fibroids and danazol causes a
post-menopausal state which would not help her in conceiving.

150
OBS GYN-System Wise 1700-by Sush and Team. 2016
Susmita, Asad, Manu, Saima, Zohaib, Savia, Shanu, Mona, Manisha, Sitara, Samreena, Sami and Komal

Q:1620 1620. A 30yo woman had an IUCD inserted 8-9m ago. Now on routine follow
up the thread is missing.Uterine US showed no IUCD in the uterus. What is the
best management?
a. Laparoscopy
b. Pelvic CT
c. Laparotomy
d. Pelvic XR

Clincher(s)
A
B
C
D
E
KEY
Additional
Information
Reference
Dr Khalid/Rabia Key: Pelvic XR (D)
Reason: Thread is missing and it isn’t seen in the uterus, Xray is the logical
choice. Laparotomy and laparoscopy are not needed unless it perforates an
organ which is highly unlikely. USG is the first choice which has been
performed, Xray after that to check location and then advice surgical retrieval
if needed.


Q:1631 1631. A 28yo woman who has had a prv pulmonary embolism in pregnancy
wishes to discuss
contraception. She has menorrhagia but is otherwise well. What is the SINGLE
most suitable
contraceptive method for this patient?
a. COCP
b. Copper IUCD
c. Levonorgestrel intra-uterine system
d. Progestogen implant
e. POP

Clincher(s)
A
B
C
D
E
KEY
Additional In embolism- no progesterone

151
OBS GYN-System Wise 1700-by Sush and Team. 2016
Susmita, Asad, Manu, Saima, Zohaib, Savia, Shanu, Mona, Manisha, Sitara, Samreena, Sami and Komal

Information
Reference
Dr Khalid/Rabia Key: Levonorgestrel Intra-Uterine System (C)
Reason: The woman has a history of thromboembolic disease, which
Studied till here essentially rules out COCPs. POPs, Copper IUCD (Copper – T) and Progestogen
implants would not help the menorrhagia. LNG-IUS (Mirena) is the hormone
releasing device that is most suitable in this patient and would be the
contraceptive of choice.


Q:1640 1640. A 27yo woman who takes the COCP has had painless vaginal spotting
and discharge for 3 days.
Her last menstrual period, which lasted four days, finished 10 days ago. Her
last cervical smear
two years ago was normal. Abdominal and vaginal examinations are normal
apart from a mild
ectropion with contact bleeding. What is the SINGLE most appropriate initial
inv?
a. Cervical smear
b. Colposcopy
c. Endocervical swab
d. Endometrial biopsy
e. Pelvic US

Clincher(s)
A
B
C
D
E
KEY
Additional
Information
Reference
Dr Khalid/Rabia Key is C: Endocervical swab (infection rule out)
As her cervical smear and examination of abdomen and vagina are normal,
next would be to exclude a STD for which Endocervical swab is taken.
· Chlamydia is an sexually transmitted infection caused by a germ
(bacterium) calledChlamydia trachomatis
· About 1 in 20 sexually active women in the UK are infected with
chlamydia. It is most common in women aged under 25. (About 1 in
12 women aged 20 are infected with chlamydia.)
Presentation:
· Vaginal discharge. This is due to the neck of the womb (cervix)
becoming inflamed.

152
OBS GYN-System Wise 1700-by Sush and Team. 2016
Susmita, Asad, Manu, Saima, Zohaib, Savia, Shanu, Mona, Manisha, Sitara, Samreena, Sami and Komal

· Pain or burning when you pass urine.
· Vaginal bleeding or spotting between periods. In particular,
bleeding after you have sex.
· Pain or discomfort in the lower tummy (abdomen) area (the pelvic
area)
Diagnosis
· Chlamydia can be confirmed by a swab taken from the neck of the
womb (cervix) in womenhave sex
Treatment
· A short course of an antibiotic medicine usually clears chlamydial
infection
· Note: antibiotics can interfere with the combined oral contraceptive
pill (COCP). If you take the COCP you should use alternative methods
of contraception until seven days after finishing a course of antibiotics

Q:1664 1664. A primigravida in the 17th week of her symptomless gestation is found,
on US, to have evidence
of placental tissue covering the cervical os. By the end of her pregnancy she is
likely to develop?
a. Placental migration (most common)
b. Uterine myoma
c. Uterine rupture
d. Choriocarcinoma
e. Chorangioma
f. Vasa previa
g. Subplacental abruption
placenta
h. Subchorionic abruption
placenta
i. Placenta accrete
j. Placenta previa (second choice)

Clincher(s)
A
B
C
D
E
KEY
Additional
Information
Reference
Dr Khalid/Rabia Ans: A
In 90% of pregnancies, an initial low lying placenta will be pulled upwards by
the growing uterus and assume a normal position in the upper segment. This
phenomenon is referred to as Migration

153
OBS GYN-System Wise 1700-by Sush and Team. 2016
Susmita, Asad, Manu, Saima, Zohaib, Savia, Shanu, Mona, Manisha, Sitara, Samreena, Sami and Komal


Q:1690 1690. A pregnant woman in an early stage of labour expresses the wish to
have pain relief during
labour. The anesthetist describes that if the patient wishes he can use
medication as a local
anesthetic to block the pain sensations of labour. Into which space should the
local anaesthetic
be normally injected?
a. Anterior pararenal space
b. Aryepiglottic space
c. Vestibule space
d. Epidural space
e. Sub-arachnoid space
f. Space of Disse
g. Middle ear
h. Posterior pararenal space
i. Supraglottic space j. Lesser sac

Clincher(s)
A
B
C
D
E
KEY D
Additional
Information
Reference
Dr Khalid/Rabia
Q:736 736. A 28yo woman who is 32 wks pregnant in her 3rd pregnancy is diagnosed
as a case of placental
abruption. After all the effective measures, she is still bleeding. What is the
underlying
pathology?
a. Clotting factor problem
. Clauser’s syndrome
c. Platelet problem
d. Succiturate lobe
e. Villamentous insertion of placenta

Clincher(s)
A key
B
C
D
E

154
OBS GYN-System Wise 1700-by Sush and Team. 2016
Susmita, Asad, Manu, Saima, Zohaib, Savia, Shanu, Mona, Manisha, Sitara, Samreena, Sami and Komal

KEY
Additional 57 OHCS- Thomboplastin from P abruption release may cause DIC
Information
Reference
Dr Khalid/Rabia Ans: Placental abruption has been defined as the complete or partial
separation of a normally implanted placenta from its uterine site before the
delivery of the fetus. This definition differentiates this process from placenta
previa, in which the placenta is implanted in an abnormal anatomical position
covering the internal cervical os.

Grade I: Mild. This group accounts for 40% of all cases; it includes antepartum
hemorrhage of uncertain cause. There may be slight vaginal bleeding and
uterine irritability. Maternal blood pressure usually is normal, and there is no
maternal coagulopathy or fetal distress. The diagnosis of this class of abruptio
placentae is confirmed on postpartum detection of a small retroplacental clot.
Grade II: Intermediate. This accounts for 45% of all cases. This diagnosis is
based on the classic features of abruptio placentae with uterine hypertonicity,
but the fetus still is alive. There is a greater amount of vaginal bleeding (mild to
moderate), hypofibrinogenemia, and fetal distress. Blood pressure is
maintained, but the pulse rate may be elevated and postural blood volume
deficits may be present.
Grade III: Severe. This accounts for 15% of all cases. In such cases, the fetus is
always dead. Usually, heavy vaginal bleeding occurs, although in some cases
this may be concealed. Maternal hypotension, hypofibrinogenemia, and
thrombocytopenia are present, along with a tetanic uterus. This type is further
subdivided into grade IIIA, in which overt coagulopathy is not present, and
grade IIIB, when an overt coagulopathy results.2
This grading system may be helpful in establishing a therapeutic plan.

Clinicians and investigators have observed the presence of a bleeding diathesis
accompanying some cases of premature separation of the placenta. In addition
to its association with severe placental abruption, acquired
hypofibrinogenemia also has been reported in cases of amniotic fluid
embolism, long-standing fetal death in utero, septic abortion, eclampsia, and
delayed postpartum hemorrhage.

The phenomenon of consumption coagulopathy leads to patient injury
because of two problems: the bleeding diathesis caused by diminished
coagulation factors and elevated FDP and the localized tissue necrosis in target
organs because of fibrin deposition in small blood vessels. As a protective
mechanism, the fibrinolytic system is activated secondarily, and dissolution of
the fibrin clots by plasmin may protect the local tissue from anoxia. This
delicate balance between fibrin deposition and degradation is present in the
body at these times. If this secondary fibrinolysis and fibrinogenolysis become
excessive, the decreased coagulation factors resulting from consumption are
augmented by destruction of the same factors by plasmin, and the

155
OBS GYN-System Wise 1700-by Sush and Team. 2016
Susmita, Asad, Manu, Saima, Zohaib, Savia, Shanu, Mona, Manisha, Sitara, Samreena, Sami and Komal

hemorrhagic situation is made worse. The degradation products of fibrin and
fibrinogen also interfere with fibrinogen conversion to fibrin. Fibrinolytic
problems in abruptio placentae are mostly secondary, with progressive
activation of the system occurring only after an initial phase of intravascular
coagulation.

The complications of hypofibrinogenemia have been detected in one third to
one half of patients having signs and symptoms of severe placental separation.
Most patients with abruptio placentae have the milder form and exhibit no
clinical difficulties with the clotting mechanism.

So as it says in the mcq that all other measures to stop the bleeding have failed
then we should start thinking on the lines of clotting factor problems. Even in
the most severe cases, clinically evident coagulopathy usually resolves by 12
hours after delivery.


Q:759 759. A 28yo pregnant lady presents with severe lower abdominal pain with
excessive per vaginal bleeding at 34wks gestation. What should be the initial
inv of choice?
a. Coagulation profile
b. US abdomen
c. CT pelvis
d. D-dimer
e. Kleiuber test

Clincher(s)
A
B
C
D
E
KEY B
Additional
Information
Reference
Dr Khalid/Rabia
Q:791 791. A 35yo primigravida post C-section complains of inability to void. She
denies dysuria but complains of fullness. She was treated with an epidural for
analgesia. What is the single most appropriate inv?
a. MSU
b. US abdomen
c. US KUB
d. Serum calcium

Clincher(s)

156
OBS GYN-System Wise 1700-by Sush and Team. 2016
Susmita, Asad, Manu, Saima, Zohaib, Savia, Shanu, Mona, Manisha, Sitara, Samreena, Sami and Komal

A
B
C
D
E
KEY
Additional
Information
Reference
Dr Khalid/Rabia Answer= C. US KUB. This is the case of postoperative urinary retention (the risk
factors for it include operation itself and spinal anesthesia) so US KUB should
be done to measure the urinary volume of bladder. it will guide in the
treatment plan. if the urine volume is <400ml observe the patient and if the
urine volume is > 600ml catheterize the patient.

Q:950 950. A pregnant woman presents with knee pain on movements. The pain
becomes worse at the end of the day. Radiology shows decreased joint space.
Labs: CRP=12. What is the 1st line med?
a. Paracetamol
b. NSAIDs
c. Oral steroid
d. Intra articular steroid
e. DMARDs

Clincher(s)
A
B
C
D
E
KEY
Additional
Information
Reference
Dr Khalid/Rabia a. Paracetamol
Osteoarthritis... First line is paracetamol safe in pregnancy
second-line treatment is oral NSAIDs/COX-2 inhibitors, opioids, capsaicin
cream and intra-articular corticosteroids.

Q:1227 1227. A 16yo girl who is normally fit and well attends her GP complaining of
heavy and painful periods.
She is requesting tx for these complaints. She denies being sexually active.
Select the most appropriate management for her menorrhagia?
a. Antifibrinolytics (tranexamic acid)
b. COCP
c. Endometrial ablation

157
OBS GYN-System Wise 1700-by Sush and Team. 2016
Susmita, Asad, Manu, Saima, Zohaib, Savia, Shanu, Mona, Manisha, Sitara, Samreena, Sami and Komal

d. IUS progestrogens (mirena)
e. NSAIDS (mefenamic acid

Clincher(s)
A
B
C
D
E
KEY E
Additional Antifibrinolyic favoured over mefenamic. Latter better for only painful
Information
Reference
Dr Khalid/Rabia Heavy and Painful periods (A)
- First-line treatment for menorrhagia is Mirena coil, but the patient in
question does not need contraception and she is having painful periods. The
second line is Mefenamic acid or Tranexamic Acid. Third line is COCPs. Fourth
line is Endometrial ablation or Hysterectomy (If there is no desire to conceive).
IUCD, according to patient.info actually causes heavy or painful periods.
- Therefore the best choice in this case is Mefenamic acid (NSAIDs)

Q:1258 1258. A 36yo woman came with uterine bleeding. Vaginal US reveals uterine
thickness=12mm. what is the most probable dx?
a. Cervical ca
b. Endometrial ca
c. Ovarian ca
d. Breast ca
e. Vaginal ca
(Normal – 20-22mm in proliferative phase)
Clincher(s)
A
B
C
D
E
KEY
Additional
Information
Reference
Dr Khalid/Rabia Answer: B.Endometrial CA. The increased uterine thickness points towards the
diagnosis.
Endometrial cancer is classically seen in postmenopausal women but around
25% of cases occur before the menopause. It usually carries a good prognosis
due to early detection

158
OBS GYN-System Wise 1700-by Sush and Team. 2016
Susmita, Asad, Manu, Saima, Zohaib, Savia, Shanu, Mona, Manisha, Sitara, Samreena, Sami and Komal

The risk factors for endometrial cancer are as follows*:
• obesity
• nulliparity
• early menarche
• late menopause
• unopposed oestrogen. The addition of a progesterone to oestrogen
reduces this risk (e.g. In HRT). The BNF states that the additional risk is
eliminated if a progestogen is given continuously
• diabetes mellitus
• tamoxifen
• polycystic ovarian syndrome


Features
• postmenopausal bleeding is the classic symptom
• pre-menopausal women may have a change intermenstrual bleeding
• pain and discharge are unusual features


Investigation
• first-line investigation is trans-vaginal ultrasound - a normal
endometrial thickness (< 4 mm) has a high negative predictive value
• hysteroscopy with endometrial biopsy


Management
• localised disease is treated with total abdominal hysterectomy with
bilateral salpingo-oophorectomy. Patients with high-risk disease may
have postoperative radiotherapy
• progestogen therapy is sometimes used in frail elderly women not
consider suitable for surgery

Q:1259 1259. A 30yo woman has PID which was treated with metronidazole and
cephalosporin. It is getting
worse. What is the next best inv?
a. Endocervical swab
b. US
c. Laparotomy
d. High vaginal swab
Nyseria- cephalosporin, and metro for chlamidiya


Clincher(s)
A
B
C
D

159
OBS GYN-System Wise 1700-by Sush and Team. 2016
Susmita, Asad, Manu, Saima, Zohaib, Savia, Shanu, Mona, Manisha, Sitara, Samreena, Sami and Komal

E
KEY
Additional
Information
Reference
Dr Khalid/Rabia Answer: B. US. US is done to rule out tubo ovarian abscess. (needs draining-
antibiotic wont work)


Pelvic inflammatory disease (PID) is a term used to describe infection and
inflammation of the female pelvic organs including the uterus, fallopian tubes,
ovaries and the surrounding peritoneum. It is usually the result of ascending
infection from the endocervix

Causative organisms
• Chlamydia trachomatis - the most common cause
• Neisseria gonorrhoeae
• Mycoplasma genitalium
• Mycoplasma hominis


Features
• lower abdominal pain
• fever
• deep dyspareunia
• dysuria and menstrual irregularities may occur
• vaginal or cervical discharge
• cervical excitation


Investigation
• screen for Chlamydia and Gonorrhoea


Management
• due to the difficulty in making an accurate diagnosis, and the potential
complications of untreated PID, consensus guidelines recommend
having a low threshold for treatment
• oral ofloxacin + oral metronidazole or intramuscular ceftriaxone + oral
doxycycline + oral metronidazole
• RCOG guidelines suggest that in mild cases of PID intrauterine
contraceptive devices may be left in. The more recent BASHH
guidelines suggest that the evidence is limited but that ' Removal of the
IUD should be considered and may be associated with better short term
clinical outcomes'

Complications

160
OBS GYN-System Wise 1700-by Sush and Team. 2016
Susmita, Asad, Manu, Saima, Zohaib, Savia, Shanu, Mona, Manisha, Sitara, Samreena, Sami and Komal

• infertility - the risk may be as high as 10-20% after a single episode
• chronic pelvic pain
• ectopic pregnancy
• perihepatitis
• tubo ovarian abscess
• reiter's syndrome


Q:1260 1260. A pregnant woman had hit her chest 3wks ago. Now she is 24wks
pregnant and presents with left upper quadrant mass with dimpling. What is
the most probable dx?
a. Breast ca
b. Carcinoma
c. Fibroadenoma
d. Fibroadenosis
e. Fatty necrosis of breast
All oma’s excluded as short hx and hx of injury
Clincher(s)
A
B
C
D
E
KEY
Additional
Information
Reference Answer:E. Fatty necrosis of breast. H/o of trauma to the breast and lump with
dimpling point towards the diagnosis.


Fat necrosis

It tends to be large, fatty breasts in obese women that have this problem:

• It usually follows trauma.


• The lump is usually painless and the skin around it may look red,
bruised or dimpled.
• Biopsy may be required, but if the diagnosis is confirmed, no
further management is indicated.


Dr Khalid/Rabia
Q: 1261. A pregnant pt with Rh –Ve who has’nt been prv sensitized delivers her
first baby without any prbs. What would be the latest time to administer anti-
sensitization?

161
OBS GYN-System Wise 1700-by Sush and Team. 2016
Susmita, Asad, Manu, Saima, Zohaib, Savia, Shanu, Mona, Manisha, Sitara, Samreena, Sami and Komal

a. 6h PP
b. 24h PP
c. 48h PP
d. 72h PP
e. 5d PP

Clincher(s)
A
B
C
D
E
KEY
Additional
Information
Reference
Dr Khalid/Rabia Answer: D.72h pp.
Anti-D immunoglobulin should be given as soon as possible (but always within
72 hours) in the following situations:
• delivery of a Rh +ve infant, whether live or stillborn
• any termination of pregnancy
• miscarriage if gestation is > 12 weeks
• ectopic pregnancy
• external cephalic version
• antepartum haemorrhage
• amniocentesis, chorionic villus sampling, fetal blood sampling


Q:

Clincher(s)
A
B
C
D
E
KEY
Additional
Information
Reference
Dr Khalid/Rabia
Q:

Clincher(s)
A
B

162
OBS GYN-System Wise 1700-by Sush and Team. 2016
Susmita, Asad, Manu, Saima, Zohaib, Savia, Shanu, Mona, Manisha, Sitara, Samreena, Sami and Komal

C
D
E
KEY
Additional
Information
Reference
Dr Khalid/Rabia
Q:

Clincher(s)
A
B
C
D
E
KEY
Additional
Information
Reference
Dr Khalid/Rabia
Q:

Clincher(s)
A
B
C
D
E
KEY
Additional
Information
Reference
Dr Khalid/Rabia


Q:330 A lady presents with abdominal pain, dysuria, dyspareunia and
vaginal discharge. What is your next step?
a. Laparoscopy
b. High vaginal swab
c. Hysteroscopy
d. Laparotomy
e. US

163
OBS GYN-System Wise 1700-by Sush and Team. 2016
Susmita, Asad, Manu, Saima, Zohaib, Savia, Shanu, Mona, Manisha, Sitara, Samreena, Sami and Komal

Clincher(s) Abd pain,dysuria,dyspareunia,vaginal discharge (points to infection)
A Not needed
B For detecting the cause of infection.
C
D Not needed
E
KEY B
Additional High Vaginal Swab (HVS) is a technique used in Obstetrics and Gynaecology to
Information obtain a sample of discharge from the vagina. This is then sent for culture and
sensitivity. It is commonly used to test for the presence of candidiasis infection,
bacterial vaginosis and trichomonas vaginalis.

In vaginal candidiasis (thrush), women may notice discomfort, labial swelling,
itchiness and a white/cream lumpy, musty smelling discharge, often causing
dyspareunia but no systemic upset.

In bacterial vaginosis, Presentation is variable, from minimal symptoms to


more extensive discomfort with a watery, fishy-smelling grey-green discharge.

The anaerobic single-cell protozoan Trichomonas vaginalis can cause a heavy,


frothy, yellow-green, fishy-smelling vaginal discharge, with dysuria and even
abdominal pain.Diagnosis is confirmed by high vaginal swab and the infection
should respond to treatment with metronidazole.


Reference Go online.com
Dr Khalid/Rabia B high vaginal swab


Q:351 A 35yo lady presents with painful ulcers on her vulva, what is the appropriate
inv which will lead to the dx?
a. Anti-HSV antibodies
b. Dark ground microscopy of the ulcer
c. Treponema palladium antibody test
d. Rapid plasma regain test
e. VDRL


Clincher(s) Painful shallow ulcers on her vulva (all options except A is for syphilis)
A This is done for Herpes simplex virus (painful ulcer- as well as chancroid)
B It's done for detection of early syphilis and ulcers are painless.
C Syphilis as mentioned cause painless ulcers.
D The rapid plasma reagin (RPR) test refers to a type of rapid diagnostic test
that looks for non-specific antibodies in the blood of the patient that may

164
OBS GYN-System Wise 1700-by Sush and Team. 2016
Susmita, Asad, Manu, Saima, Zohaib, Savia, Shanu, Mona, Manisha, Sitara, Samreena, Sami and Komal

indicate a syphilis infection.
E Screening of syphilis.
KEY A
Additional
Information
Reference
Dr Khalid/Rabia Genital Herpes may be asymptomatic or may remain dormant for months or
even years. When symptoms occur soon after a person is infected, they tend
to be severe.
They may start as multiple small blisters that eventually break open and
produce raw, painful sores that scab and heal over within a few weeks. The
blisters and sores may be accompanied by flu-like symptoms with fever and
swollen lymph nodes.
There are three major drugs commonly used to treat genital herpes
symptoms: acyclovir (Zovirax), famciclovir (Famvir), and
valacyclovir(Valtrex). These are all taken in pill form. Severe cases
may be treated with the intravenous (IV) drug acyclovir].


Q:355 A 28yo pregnant woman with polyhydramnios (excessive fluids) and SOB
comes for an anomaly scan at 31 wks. US= absence of gastric bubble. What is
the most likely dx?
a. Duodenal atresia
b. Esophageal atresia
c. Gastrochiasis
d. Exomphalos
e. Diaphragmatic hernia


Clincher(s) Polyhydrominos,SOB, absence of gastric bubble
A The diagnosis of duodenal atresia is usually confirmed by radiography. An X-
ray of the abdomen shows two large air filled spaces, the so-called "double
bubble" sign
B On antenatal USG, the finding of an absent or small stomach in the setting of
polyhydramnios was considered a potential symptom of esophageal atresia
C Abdominal defect in children. It is protrusion of abdominal contents through a
defect in the ant abd wall to the right of umbilicus.
D Congenital abnormality. Abdominal contents are found outside the abdomen
covered in a three layer membrane consisting of peritoneum, Wharton jelly
and amnion.
E Symptoms will be tachpnoea,tachycardia, difficulty breathing , bowel sounds
in the chest area and diminished or absent breath sounds.
KEY B
Additional Other birth defects may co-exist with Esophageal atresia are particularly in
Information the heart, but sometimes also in the anus, spinal column, or kidneys. This is
known as VACTERL association because of the involvement of Vertebral

165
OBS GYN-System Wise 1700-by Sush and Team. 2016
Susmita, Asad, Manu, Saima, Zohaib, Savia, Shanu, Mona, Manisha, Sitara, Samreena, Sami and Komal

column, Anorectal, Cardiac, Tracheal, Esophageal, Renal, and Limbs. It is
associated with polyhydramnios in the third trimester.
Reference Wikipedia
Dr Khalid/Rabia Oesophageal atresia


Q:359 A woman has numerous painful ulcers on her vulva. What is the cause?
a. Chlamydia
b. Trichomonas
c. Gardenella
d. HSV
e. EBV


Clincher(s) Painful ulcers on vulva
A Painless ulcer
B Bubbly fishy smell discharge with dysuria and abd pain
C Fishy smell discharge
D Painful ulcer
E Not related to vulva ulcers
KEY D
Additional Please scroll up for HSV information.
Information
Reference
Dr Khalid/Rabia


Q:376 Pt with widespread ovarian carcinoma has bowel obstruction and severe colic
for 2h and was normal in between severe pain for a few hours (e.g. when she
eats). What is the most appropriate management?
a. PCA (morphine)
b. Spasmolytics
c. Palliative colostomy
d. Oral morphine
e. Laxatives

Clincher(s) Widespread ovarian cancer with intestinal obstruction and colic
A It will only relieve the pain
B No role here
C This will relieve the intestinal obstruction and hence episodic pain will
disappear
D Again will only relieve pain
E No role
KEY C
Additional

166
OBS GYN-System Wise 1700-by Sush and Team. 2016
Susmita, Asad, Manu, Saima, Zohaib, Savia, Shanu, Mona, Manisha, Sitara, Samreena, Sami and Komal

Information
Reference
Dr Khalid/Rabia Cancer or chemotherapy induced obstructions are unlikely
to respond to conservative management [NBM, IV fluid, nasogastric
suction] and hence only analgesia will not relieve it. So in such
cases we have to go for palliative colostomy.

Q:385 A 65yo female pt was given tamoxifen, which of the following
side effect caused by it will concern you?
a. Fluid retention
b. Vaginal bleeding
c. Loss of apetite
d. Headache and dizziness
e. Anorgasm

Clincher(s) Tamoxifen hazardous side effect tamoxifen+ vaginal = endo Ca
A Common side effect.
B Less common but worrisome SE and pt need to seek immediate medical
advice.
C Not related
D Less common SE
E Less common
KEY B (endometrial hyperplasia)
Additional
Information
Reference Drugs.com
Dr Khalid/Rabia Tamoxifen can promote development of endometrial
carcinoma. So vaginal bleeding will be of concern for us.

Q:542 A 23yo woman is being followed up 6wks after a surgical procedure to
evacuate the uterus following a miscarriage. The histology has shown changes
consistent with a hydatidiform mole. What is the single most appropriate inv in
this case?
a. Abdominal US
b. Maternal karyotype
c. Paternal blood group
d. Serum B-HCG
e. Transvaginal US


Clincher(s) Hyadatiform mole on histology
A In pregnant females it presents as snowstorm effect and large for dates.
B No role here
C Same as above
D This is usually done to monitor the levels of serum HCG

167
OBS GYN-System Wise 1700-by Sush and Team. 2016
Susmita, Asad, Manu, Saima, Zohaib, Savia, Shanu, Mona, Manisha, Sitara, Samreena, Sami and Komal

E No role after miscarriage.
KEY D
Additional If it keeps going up after delivery: could be choriocarcinoma
Information
If usg= snowstorm appearance= hydatiform mole


Reference
Dr Khalid/Rabia Complete hydatidiform mole

Occurs when an empty egg is fertilized by a single sperm that then duplicates
its own DNA, hence the all 46 chromosomes are of paternal origin

Features
· bleeding in first or early second trimester
· exaggerated symptoms of pregnancy e.g. hyperemesis
· uterus large for dates
· very high serum levels of human chorionic gonadotropin (hCG)
· hypertension and hyperthyroidism* may be seen

Management
· urgent referral to specialist centre - evacuation of the uterus is performed
· effective contraception is recommended to avoid pregnancy in the next 12
months

Around 2-3% go on to develop choriocarcinoma

In a partial mole a normal haploid egg may be fertilized by two sperms, or by
one sperm with duplication of the paternal chromosomes. Therefore the DNA
is both maternal and paternal in origin. Usually triploid - e.g. 69 XXX or 69 XXY.
Fetal parts may be seen

*hCG can mimic thyroid-stimulating hormone (TSH)



Q:552 A 38yo female G4 at 32wks of pregnancy presented with thick white marks on
the inside of her mouth for 3wks. Her mouth including her tongue appeared
inflamed on examination. She smokes 20 cigarettes/day despite advice to quit.
She attends her ANC regularly. What is the most probable dx?
a. Lichen planus
b. Aphthous ulcer
c. Smoking
d. Candidiasis
e. Leukoplakia

168
OBS GYN-System Wise 1700-by Sush and Team. 2016
Susmita, Asad, Manu, Saima, Zohaib, Savia, Shanu, Mona, Manisha, Sitara, Samreena, Sami and Komal

Clincher(s) Thick white mArks inside her tongue and hx of smoking.
A It has lacy appearance not thick. It's painful and less itchy.
B Not the case here (not white)
C X
D Pregnancy is a risk factor along with smoking and white marks making the
diagnosis of candidiasis more sound. Cherry tongue, marks
E Raised white plaques which can't be rubbed out and normally assc with HIV.
Cant remove it. Patch. No inflammation
KEY D
Additional
Information
Reference
Dr Khalid/Rabia lichen planus may have lace like appearance and not thick white mark.
Aphthous ulcer has yellowish floor and surrounded by erythematous halo.
Smoking may cause tongue coating but not like thick white mark on the inside
of mouth.
Leukoplakia is with raised edges/Bright white patches and sharply defined and
cannot be rubbed out like candida patch;
here also inflamed tongue points towards infection. So candidiasis is the
probable option].



Q:557 7. A 28yo woman has been admitted at 38wks gestation. Her
BP=190/120mmHg and proteinuria +++. Immediately following admission she
has a grand-mal seizure. What is the single most appropriate initial
management? `(most appropriate- immediate delivery)
a. Diazepam IV
b. Fetal CTG
c. Hydralazine IV
d. Immediate delivery
e. Magnesium sulphate

Clincher(s) High BP, proteinuria and seizure
A Not the first choice
B Not the tx
C Tx for HTN in pregnant females
D Delivery is the only cure for females with severe pre eclampsia and other
symptoms.
E Initial management of seizures. Intially, loading dose, then maintence dose,
then dolus and IV, then diazepam, then ventilations.
KEY E
Additional This applies to those with BP >160/110 with proteinuria or BP ≥140/90 with
Information
proteinuria plus one or more of:

169
OBS GYN-System Wise 1700-by Sush and Team. 2016
Susmita, Asad, Manu, Saima, Zohaib, Savia, Shanu, Mona, Manisha, Sitara, Samreena, Sami and Komal

• Seizures • Headache or epigastric pain • Platelets <100 ≈ 109/L

• Visual disturbance • Papilloedema • ALT >70U/L

• Clonus (>3 beats) • Liver tenderness or HELLP (p26)

Continuously monitor maternal oxygen saturation, and BP.

Use prophylactic magnesium sulfate: 4g (8mL of 50% solution) IVI over 15min
in 100mL 0.9% saline; then maintenance .

• Catheterize: measure urine output (eg use urometer) & T° hourly; FBC,
U&E,
• LFTs, creatinine every 12–24h. If platelets <100 ≈ 109/L do clotting
studies.
• Monitor fetal heart rate; assess liquor volume and fetal growth by scan;

umbilical cord Doppler if possible. Use monitoring in labour.

• Delivery is the only cure for these women. When a decision is made to
de-liver, contact on-call consultant, anaesthetist, and senior labour
ward mid-wife. Deliver appropriately (eg <34 weeks usually by caesar).
Give steroids
• if <34–36 wks, (p51). At 3rdstage of labour give 5U oxytocin IM/IV
slowly.


Reference OHCS page 49
Dr Khalid/Rabia


Q:

Clincher(s)
A
B
C
D
E
KEY
Additional
Information
Reference
Dr Khalid/Rabia

170
OBS GYN-System Wise 1700-by Sush and Team. 2016
Susmita, Asad, Manu, Saima, Zohaib, Savia, Shanu, Mona, Manisha, Sitara, Samreena, Sami and Komal


Q: 273 A 24yo woman presents with deep dyspareunia and severe pain in every cycle.
What is the
initial inv?
a. Laparoscopy
b. Pelvic US
c. Hysteroscopy
d. Vaginal Swab




Clincher(s) Deep dysperunia, cyclical pain
A
B
C
D
E
KEY Key = B (Pelvis ultrasound)

Additional Deep dysperunia could be due to pid, endometriosis etc
Information
Reference
Dr Khalid/Rabia To rule out cervical abnormalities, endometriosis. ovarian cysts etc.
Treatment: There is no cure for endometriosis, but a number of treatments
may improve symptoms. This may include pain medication [NSAIDs such as
naproxen], hormonal treatments [COCP, or mirena], or surgery [Surgical
removal of endometriosis when other measures fail].



Q: 274 A 38yo woman, 10d postpartum presents to the GP with hx of passing blood
clots per vagina
since yesterday. Exam: BP=90/40mmhg, pulse=110bpm, temp=38C, uterus
tender on palpation
and fundus 2cm above umbilicus, blood clots +++. Choose the single most
likely dx/
a. Abruption of placenta 2nd to pre-eclampsia
b. Concealed hemorrhage
c. Primary PPH
d. Secondary PPH
e. Retained placenta
f. Scabies


Clincher(s) Pph 10 days post partum

171
OBS GYN-System Wise 1700-by Sush and Team. 2016
Susmita, Asad, Manu, Saima, Zohaib, Savia, Shanu, Mona, Manisha, Sitara, Samreena, Sami and Komal

A irrelevant
B --
C Primary is within first 24 hours
D
E Causes of secondary pph.
KEY The key is D. Secondary PPH.


Additional
Information
Reference
Dr Khalid/Rabia Secondary PPH: Secondary PPH is defined as abnormal or excessive bleeding
from the birth canal between 24 hours and 12 weeks postnatally.
[www.rcog.org.uk/en/guidelines-research-services/guidelines/gtg52/].



Q: 275 A 32yo female with 3 prv 1st trimester miscarriages is dx with antiphospholipid
syndrome.
Anticardiolipin
antibodies +ve. She is now 18wks pregnant. What would be the most
appropriate
management?
a. Aspirin
b. Aspirin & warfarin
c. Aspirin & heparin
d. Heparin only
e. Warfarin only

1st trimester recurrent miscarriage: anti Phospho;
2nd trimester common cause : thrombophilia
Clincher(s)
A
B
C
D
E
KEY C
Additional Points in favour = More than 3 prev miscarriages due to APLS - LMWH plus
Information aspirin throughout pregnancy is indicated.

Affected women are treated from conception with aspirin 75mg daily and
heparin eg enoxaparin 40mg sc/24h from when fetal heart identified (about
6 weeks) until 34 weeks. Those who have suffered prior thromboses receive
heparin throughout pregnancy.
Ohcs page 30.

172
OBS GYN-System Wise 1700-by Sush and Team. 2016
Susmita, Asad, Manu, Saima, Zohaib, Savia, Shanu, Mona, Manisha, Sitara, Samreena, Sami and Komal


Reference
Dr Khalid/Rabia Antiphospholipid syndrome (APS) is an autoimmune disorder characterised by
arterial and venous thrombosis, adverse pregnancy outcomes (for mother and
fetus), and raised levels of antiphospholipid (aPL) antibodies.

Presentation:
APS has varied clinical features and a range of autoantibodies. Virtually any
system can be affected, including:[1][4][5]
• Peripheral artery thrombosis, deep venous thrombosis.
• Cerebrovascular disease, sinus thrombosis.
• Pregnancy loss: loss at any gestation - recurrent miscarriage or
prematurity can be seen in APS.
• Pre-eclampsia, intrauterine growth restriction (IUGR).
• Pulmonary embolism, pulmonary hypertension.
• Livedo reticularis (persistent violaceous, red or blue pattern of the skin
of the trunk, arms or legs; it does not disappear on warming and may
consist of regular broken or unbroken circles), purpura, skin ulceration.
• Thrombocytopenia, haemolytic anaemia.
• Libman-Sacks endocarditis and cardiac valve disease:
o Usually mitral valve disease or aortic valve disease and usually
regurgitation with or without stenosis.
o Mild mitral regurgitation is very common and is often found
with no other pathology. There may also be vegetations on the
heart and valves.
• Myocardial infarction.
• Retinal thrombosis.
• Nephropathy: vascular lesions of the kidneys may result in chronic
kidney disease.
• Adrenal infarction.
• Avascular necrosis of bone.

Investigations
Young adults (≤50 years old) with ischaemic stroke and women with recurrent
pregnancy loss (≥3 pregnancy losses) before 10 weeks of gestation should be
screened for aPL antibodies.[3]
• Levels of aCL, anti-beta2 GPI or lupus anticoagulant (LA) on two
occasions at least 12 weeks apart.
• FBC; thrombocytopenia, haemolytic anaemia.
• Clotting screen.
• CT scanning or MRI of the brain (cerebrovascular accident), chest
(pulmonary embolism) or abdomen (Budd-Chiari syndrome).
• Doppler ultrasound studies are recommended for possible detection of
deep vein thrombosis.

173
OBS GYN-System Wise 1700-by Sush and Team. 2016
Susmita, Asad, Manu, Saima, Zohaib, Savia, Shanu, Mona, Manisha, Sitara, Samreena, Sami and Komal

• Two-dimensional echocardiography may demonstrate asymptomatic
valve thickening, vegetations or valvular insufficiency.

Management in Pregnancy :
APS in pregnancy may affect both mother and fetus throughout the entire
pregnancy and is associated with high morbidity. Clinical complications are
variable and include recurrent miscarriage, stillbirth, IUGR and pre-
eclampsia.
• For women with APS with recurrent (≥3) pregnancy loss, antenatal
administration of low molecular weight heparin combined with low-
dose aspirin is recommended throughout pregnancy. Treatment
should begin as soon as pregnancy is confirmed.
• For women with APS and a history of pre-eclampsia or IUGR, low-
dose aspirin is recommended.
• Women wit aPL antibodies should be considered for postpartum
thromboprophylaxis.





Q: 276 A 23yo presents with vomiting, nausea and dizziness. She says her menstrual
period has been delayed 4 weeks as she was stressed recently. There are no
symptoms present. What is the next appropriate management?
a. Refer to OP psychiatry
b. Refer to OP ENT
c. CT brain
d. Dipstick for B-hCG


Clincher(s)
A
B
C
D
E
KEY D
Additional Test for pregnancy first in case of amenorrhea, next appropriate
Information management.
Reference
Dr Khalid/Rabia


Q: 277 . A 16yo girl came to the sexual clinic. She complains of painful and heavy
bleeding. She says she
does Not have a regular cycle. What is the ost appropriate management?

174
OBS GYN-System Wise 1700-by Sush and Team. 2016
Susmita, Asad, Manu, Saima, Zohaib, Savia, Shanu, Mona, Manisha, Sitara, Samreena, Sami and Komal

a. Mini pill
b. Combined pill
c. IUS
d. Anti-prostoglandins
e. Anti-fibrinolytics


Clincher(s) 16 years old, painful heavy bleeding irregular cycle
A
B
C
D
E
KEY The key is B. Combined pill.
Additional COCP are best for treating meonorragia plus dysmeonoehea and makes theb
Information cycle regular, plus gives contraception too.
Reference
Dr Khalid/Rabia

Q: 279 Which method of contraception can cause the risk of ectopic pregnancy?
a. COCP
b. IUCD
c. Mirena
d. POP

IUCD causes ascending infection and hence causes PID, and makes endo less
favourable for implantation and so ectopic
Clincher(s)
A
B
C
D
E
KEY B
Additional IUCD pose a risk of ectopic pregnancy.
Information
Reference
Dr Khalid/Rabia

Q: 282 A 31yo woman who is 32weeks pregnant attends the antenatal clinic. Labs:
Hgb=10.7, MCV=91.
What is the most appropriate management for this pt?
a. Folate supplement
b. Ferrous sulphate 200mg/d PO
c. Iron dextran

175
OBS GYN-System Wise 1700-by Sush and Team. 2016
Susmita, Asad, Manu, Saima, Zohaib, Savia, Shanu, Mona, Manisha, Sitara, Samreena, Sami and Komal

d. No tx req



Clincher(s)
A First trimester
B First trimester, if anemic
C Second trimester if anemic and can not tolerate oral iron
D
E
KEY D
Additional
Information
Reference
Dr Khalid/Rabia [According to NICE, cut offs for iron supplements:
at booking (8-10 weeks)- if less than 11
at 28 weeks and further- if less than 10.5
if less than these values=> give iron].



Q: 284 A 32yo woman of 39wks gestation attends the antenatal day unit feeling very
unwell with
sudden onset of epigastric pain a/w nausea and vomiting. Temp 36.7C. Exam:
RUQ tenderness.
Bloods: mild anemia, low plts, elevated LFT and hemolysis. What is the most
likely dx?
a. Acute fatty liver of pregnancy
b. Acute pyelonephritis
c. Cholecystitis
d. HELLP syndrome
e. Acute hepatitis


Clincher(s)
A
B
C
D
E
KEY D (HELLP syndrome)
Points in favour = hemolysis, elevated LFTs and low platelets


Additional
Information

176
OBS GYN-System Wise 1700-by Sush and Team. 2016
Susmita, Asad, Manu, Saima, Zohaib, Savia, Shanu, Mona, Manisha, Sitara, Samreena, Sami and Komal

Reference
Dr Khalid/Rabia
Presentation
• HELLP syndrome is a serious form of pre-eclampsia and patients may
present at any time in the last half of pregnancy.
• One third of women with HELLP syndrome present shortly after
delivery.
• Symptoms of HELLP syndrome are usually nonspecific.
• Initially, women may report nonspecific symptoms including malaise,
fatigue, right upper quadrant or epigastric pain, nausea, vomiting, or
flu-like symptoms.
• Hepatomegaly can occur.
• Some women may have easy bruising/purpura.
• On examination, oedema, hypertension and proteinuria are present.
• Tenderness over the liver can occur.

Investigations
• There needs to be a high index of clinical suspicion in order to avoid
diagnostic delay and improve outcome.
• Haemolysis with fragmented red cells on the blood film
• Raised LDH >600 IU/L with a raised bilirubin.
• Liver enzymes are raised with an AST or ALT level of >70 IU/L.
Levels of AST or ALT >150 IU/L are associated with increased
• morbidity and mortality.

Management
• The main treatment is to deliver the baby as soon as possible, even if
premature, since liver function in the mother gets worse very quickly.
• Problems with the liver can be harmful to both mother and child.
• Definitive treatment of HELLP syndrome requires delivery of the fetus
and is advised after 34 weeks of gestation if multisystem disease is
present.
• There is no clear evidence of any effect of giving corticosteroids on
clinical outcomes for women with HELLP syndrome.[2]
• Transfusion of red cells, platelets, fresh frozen plasma and
cryoprecipitate or fibrinogen concentrate are required as indicated
clinically and by blood and coagulation tests.
• Postpartum HELLP syndrome may be treated with steroids and plasma
exchange.
• If the fetus is less than 34 weeks of gestation and delivery can be
deferred, corticosteroids should be given.
• Blood pressure control is very important.
• Women with severe liver damage may need liver transplantation.

177
OBS GYN-System Wise 1700-by Sush and Team. 2016
Susmita, Asad, Manu, Saima, Zohaib, Savia, Shanu, Mona, Manisha, Sitara, Samreena, Sami and Komal


Q: 285 A 57yo woman presents with dysuria, frequency and urinary incontinence. She
complains of
dyspareunia. Urine culture has been done and is sterile. What is the most
appropriate step?
a. Oral antibiotics
b. Topical antibiotics
c. Topical estrogen
d. Oral estrogen
e. Oral antibiotics and topical estrogen


Clincher(s) 57 years old, dysuria, frequency and urinary incontinence, dysperunia. Urine
culture sterile.
A Not needed
B ---
C
D
E
KEY Key = C (topical estrogen)

Additional In menopausal women, atrophy of estrogen-dependant tissues(genitalia,
Information breasts) and skin occur. Vaginal dryness can lead to vaginal and urinary
infection, dyspareunia, traumatic bleeding, stress incontinence, and
prolapse.
Ohcs 256.
Reference
Dr Khalid/Rabia The problem here is vaginal dryness for which the age and symptoms are a
good clue. Topical estrogen or HRT can be given to treat vaginal dryness,
vaginal discharge and recurrent UTIs in post menopausal women.


Q:

Clincher(s)
A
B
C
D
E
KEY
Additional
Information
Reference
Dr Khalid/Rabia

178
OBS GYN-System Wise 1700-by Sush and Team. 2016
Susmita, Asad, Manu, Saima, Zohaib, Savia, Shanu, Mona, Manisha, Sitara, Samreena, Sami and Komal




Q:1262 A 30yo primigravida who is 30wks GA presents to the L&D with absent fetal
movements. She also complains of severe headache, heartburn and seeing
floaters before her eyes for the last
few days. Exam: BP=170/110 mmHg, urine protein=++++, rock hard uterus, no
visible signs of
fetal movements. Choose the single most likely dx?
a. Abruption of placenta 2nd pre-eclampsia
b. Antepartum hemorrhage
c. Placenta previa
d. Primary PPH
e. IUFD
f. Abruption of placenta due to trauma

Clincher(s)
A ABRUPTION OF PLACENTA
Abruption is the premature separation of a normally placed placenta before
delivery of the fetus, with blood collecting between the placenta and the
uterus. It is one of the two most important causes of antepartum haemorrhage
(the other being placenta praevia), accounting for 30% of all cases of
antepartum haemorrhage.

B
C
D
E
IUFD: We need a lot more to conclude IUFD and cannot be based on just rock
hard uterus and no visible signs of fetal movements (such as auscultation,
cardiotocography, real time ultrasonography etc)

KEY Answer: A. Abruption of placenta secondary to preeclampsia. Maternal
Hypertension is the most important cause of placental abruption.
Rigid abdomen/ hard rock uterus here indicates peritoneal irritation due to
bleeding (concealed haemorrhage)
exclusion:

Additional There are two main forms:
Information
• Concealed (20% of cases) - where haemorrhage is confined within
the uterine cavity and is the more severe form. The amount of
blood lost is easily underestimated.
• Revealed (80%) - where blood drains through the cervix, usually
with incomplete placental detachment and fewer associated

179
OBS GYN-System Wise 1700-by Sush and Team. 2016
Susmita, Asad, Manu, Saima, Zohaib, Savia, Shanu, Mona, Manisha, Sitara, Samreena, Sami and Komal

problems.

Risk factors

There are recognised factors that increase the risk - these include:

• Previous abruption carries the highest risk of abruption in current


pregnancy.
• Multiple pregnancy: twice as common with a twin pregnancy than
with a singleton.
• Trauma:
o Road traffic accident.
o Domestic violence.
o Iatrogenic - eg, external cephalic version.
• Threatened miscarriage earlier in current pregnancy.
• Pre-eclampsia and maternal hypertension (most imp risk factor
accounting for approx: 44% of cases)
• Multiparity.
• Previous caesarean section.
• Non-vertex presentations.
• Smoking.
• Cocaine or amphetamine use during pregnancy.
• Thrombophilia.
• Intrauterine infections.
• Polyhydramnios.

The clinical features of placental abruption depend on the size and site of the
bleeding.

The grades of haemorrhage described are:

• mild - in this case there is only a small area of placental separation and
the blood loss is usually less than 200 ml. There may be abdominal
discomfort and the uterus may be tender
• moderate - up to a 1/3 of the placenta separates. There is more severe
bleeding (200-600 ml). The patient complains of abdominal pain. On
examination the patient may have tachycardia but does not have signs
of hypovolemia. The uterus is tender. Fetal heart sounds are present
• severe - in this condition more than half of the placenta separates. The
abdominal pain is more severe. On examination the uterus is tender
and rigid (hard) - it may be impossible to feel the fetus. Fetal heart
sounds are reduced or absent. The patient may be in a state of
hypovolaemic shock

Diagnosis
Abruption is a clinical diagnosis with no available sensitive or reliable

180
OBS GYN-System Wise 1700-by Sush and Team. 2016
Susmita, Asad, Manu, Saima, Zohaib, Savia, Shanu, Mona, Manisha, Sitara, Samreena, Sami and Komal

diagnostic tests.
Management:
Guidance from the Royal College of Obstetricians and Gynaecologists for
moderate or severe placental abruption is to follow ABCD of resuscitation

• Assess Airway and Breathing: high-flow oxygen.


• Evaluate Circulation:
• Access fetus and Decide on Delivery


Reference
Dr Khalid/Rabia


Q:1263 A 38yo woman, 10d post partum, presents to her GP with a hx of passing blood
clots per vagina since yesterday. Exam: BP=90/40 mmHg, pulse=110 bpm,
temp=38C, uterus tender on palpation and fundus is 2 cm above umbilicus,
blood clots +++. Choose the single most likely dx?
a. Abruption of placenta 2nd preeclampsia
b. Concealed hemorrhage (abruption)
c. Primary PPH
d. Secondary PPH
e. Retained placenta
f. Scabies



Primary after 24 hrs of delivery and from 24 hours onwards- secondary
hemorrhage. Placenta previa- cause primary hge
Clincher(s)
A
B
C Occurs after 24hrs
D Secondary PPH-occurs between 24 hours - 12 weeks**

E

181
OBS GYN-System Wise 1700-by Sush and Team. 2016
Susmita, Asad, Manu, Saima, Zohaib, Savia, Shanu, Mona, Manisha, Sitara, Samreena, Sami and Komal

KEY Answer:D. Secondary PPH. The 10day post partum, signs of shock and blood
clots all point towards the diagnosis of secondary pph.

Additional Post-partum haemorrhage
Information

Post-partum haemorrhage (PPH) is defined as blood loss of > 500mls and may
be primary or secondary

Primary PPH
• occurs within 24 hours
• affects around 5-7% of deliveries
• most common cause of PPH is uterine atony (90% of cases). Other
causes include genital trauma and clotting factors

Risk factors for primary PPH include*:
• previous PPH
• prolonged labour
• pre-eclampsia
• increased maternal age
• polyhydramnios
• emergency Caesarean section
• placenta praevia
• macrosomia
• ritodrine (a beta-2 adrenergic receptor agonist used for tocolysis)


Management
• ABC
• IV syntocinon (oxytocin) 10 units or IV ergometrine 500 micrograms
• IM carboprost
• other options include: B-Lynch suture, ligation of the uterine arteries or
internal iliac arteries
• if severe, uncontrolled haemorrhage then a hysterectomy is sometimes
performed as a life-saving procedure

Secondary PPH
• occurs between 24 hours - 12 weeks**
• due to retained placental tissue or endometritis


*the effect of parity on the risk of PPH is complicated. It was previously
thought multiparity was a risk factor but more modern studies suggest
nulliparity is actually a risk factor

Reference
Dr Khalid/Rabia

182
OBS GYN-System Wise 1700-by Sush and Team. 2016
Susmita, Asad, Manu, Saima, Zohaib, Savia, Shanu, Mona, Manisha, Sitara, Samreena, Sami and Komal



Q:1264 IMP A 22yo lady who is in her last trimester of pregnancy comes with hx of
exposure to a child dx with chicken pox 1d ago. She was investigated and was
+ve for varicella antibody. What is the single most appropriate management?
a. Give varicella I/g
b. Quarantine
c. Give varicella vaccination
d. Oral acyclovir
e. Reassure,.


Clincher(s)
A
B
C
D
E
KEY Answer: E. Reassure. Lady is +ve for varicella antibody so no need to give
varicella zoster Igs, just reassure the patient
Additional
Information Chickenpox exposure in pregnancy
Chickenpox is caused by primary infection with varicella zoster virus. Shingles
is reactivation of dormant virus in dorsal root ganglion. In pregnancy there is a
risk to both the mother and also the fetus, a syndrome now termed fetal
varicella syndrome

Risks to the mother
• 5 times greater risk of pneumonitis


Fetal varicella syndrome (FVS)
• risk of FVS following maternal varicella exposure is around 1% if occurs
before 20 weeks gestation
• studies have shown a very small number of cases occurring between
20-28 weeks gestation and none following 28 weeks
• features of FVS include skin scarring, eye defects (microphthalmia),
limb hypoplasia, microcephaly and learning disabilities

Other risks to the fetus
• shingles in infancy: 1-2% risk if maternal exposure in the second or
third trimester
• severe neonatal varicella: if mother develops rash between 5 days
before and 2 days after birth there is a risk of neonatal varicella, which
may be fatal to the newborn child in around 20% of cases

183
OBS GYN-System Wise 1700-by Sush and Team. 2016
Susmita, Asad, Manu, Saima, Zohaib, Savia, Shanu, Mona, Manisha, Sitara, Samreena, Sami and Komal


Management of chickenpox exposure
• if there is any doubt about the mother previously having chickenpox
maternal blood should be urgently checked for varicella antibodies
• if the pregnant women is not immune to varicella she should be given
varicella zoster immunoglobulin (VZIG) as soon as possible. RCOG and
Greenbook guidelines suggest VZIG is effective up to 10 days post
exposure
• consensus guidelines suggest oral aciclovir should be given if pregnant
women with chickenpox present within 24 hours of onset of the rash

Reference
Dr Khalid/Rabia


Q:1265 . A 22yo woman who is 20wk pregnant came with pain and bleeding per
vagina. Exam: os is not open. What is the single most likely dx?
a. Threatened abortion
b. Missed abortion
c. APH
d. Miscarriage
e. Inevitable abortion


Clincher(s)
A Threatened Abortion. Pain and bleeding per vagina and os closed all point
towards it.
B Obsolete term – reference ten teachers book of gyne obs : miscarriage is now
used
C General term – bleeding less than 20weeks
D Incomplete miscarriage: this occurs when the products of conception are
partially expelled. Os not opened. Many incomplete miscarriages can be
unrecognised missed miscarriages
Complete miscarriage is vice versa: os is open
E Inevitable miscarriage: usually presents with heavy bleeding with clots and
pain. The cervical os is open. The pregnancy will not continue and will proceed
to incomplete or complete miscarriage.
• .
Threatened>incomplete>Inevitable>complete

KEY Answer. A. Threatened Abortion. Pain and bleeding per vagina and os closed
all point towards the diagnosis
Additional .
Information MISCARRIAGE
Miscarriage is defined as the loss of a pregnancy before 24 weeks of gestation.

184
OBS GYN-System Wise 1700-by Sush and Team. 2016
Susmita, Asad, Manu, Saima, Zohaib, Savia, Shanu, Mona, Manisha, Sitara, Samreena, Sami and Komal

Classification of miscarriage is as follows:
• .
• Missed miscarriage: the fetus is dead but retained. The uterus is
small for dates. A pregnancy test can remain positive for several
days. It presents with a history of threatened miscarriage and
persistent, dirty brown discharge. Early pregnancy symptoms may
have decreased or gone.
• Habitual or recurrent miscarriage: three or more consecutive
miscarriages.


Reference
Dr Khalid/Rabia


Q:1266 A 32yo lady G1, 28wks GA came to her ANC with a concern about pain relief
during labour. She has no medical illnesses and her pregnancy so far has been
uncomplicated. She wishes to feel her baby being born but at the same time
she wants something to work throughout her labour. What method of pain
relief best matches this lady’s request?
a. C-section
b. Pudendal block
c. Entonox
d. TENS
e. Pethidine

Clincher(s)
A
B Local analgesia : in episiotomy
C Given for all pt
D 1. Transcutaneous electrical nerve stimulation (TENS)
Randomised controlled trials provide no compelling evidence for TENS having
any analgesic effect during labour. so it is not recommended by NICE.

Offered to pt.

E Causes resp depression in fetus
KEY C. Entonox. . 4-Nitrous oxide and oxygen (Entonox®)
This is a 50:50 mixture inhaled during painful contractions during the first and
second stages of labour. It is often used as a supplement to pethidine.

• The main advantages are that it is under the patient's control, it


takes effect within seconds and wears off quickly with no side-
effects.

185
OBS GYN-System Wise 1700-by Sush and Team. 2016
Susmita, Asad, Manu, Saima, Zohaib, Savia, Shanu, Mona, Manisha, Sitara, Samreena, Sami and Komal

• Inhaled analgesia appears to be effective in reducing pain intensity
and in giving pain relief in labour

Additional Pain Relief In Labor:
Information 1- TENS
2. Acupuncture and hypnosis may be beneficial for the management of pain
during labour;
3. Water/birthing pool: Immersion in water during labour is claimed to
increase maternal relaxation and reduce analgesic requirements. It is
supported by the Royal College of Obstetricians and Gynaecologists (RCOG) for
healthy women with uncomplicated pregnancies.

4 entonux
5. Intramuscular opiate: Parenteral opioids provide some relief from pain in
labour but are associated with adverse effects - eg, maternal nausea, vomiting
and drowsiness.

6. Epidural analgesia: Epidural analgesia is a central nerve block technique


achieved by injection of a local anaesthetic close to the nerves that transmit
pain. It is widely used as a form of pain relief in labour. Advantages: It is the
most effective way of relieving pain in labour - providing complete relief in
95% of cases. It also has the benefit of avoiding need for greater
analgesia/general anaesthetic if forceps, vacuum extraction or caesarean
section are required. It is not associated with increase in symptoms related to
perineal trauma and pelvic floor muscle weakness.


7. Ambulatory epidural: This is a low-dose epidural that relieves pain, but
allows women to walk about during labour.

8. Local analgesia: This is used for women who have not had an epidural but
require forceps or vacuum extraction delivery. It is also used for repair of
episiotomy or perineal tear.

Pudendal nerve block: using lidocaine behind each ischial spine of the pelvis
via the vagina.


Reference
Dr Khalid/Rabia

Q:1267 . A primipara at full term in labor has passed show and the cervix is 3cm
dilated. What is the single most appropriate management for her labor?
a. Repeat vaginal examination in 4h
b. CTG
c. IV syntocin drip

186
OBS GYN-System Wise 1700-by Sush and Team. 2016
Susmita, Asad, Manu, Saima, Zohaib, Savia, Shanu, Mona, Manisha, Sitara, Samreena, Sami and Komal

d. Repeat vaginal examination in 2h
e. Induction of labour



Clincher(s)
A Primiparous woman, in labour, cervix at 3cm dilatation points to Latent phase
of 1st Stage of Labor.

The next step is to do a Vagina Examination in 4hours (to assess any
improvement in cervical dilatation).

B
C
D
E
KEY A
Additional It is the first stage of labor since the cervix is only 3cm dilated. First stage of
Information labour ends when the cervix is 10cm dilated.

The first stage of labour
Latent phase (not necessarily continuous):
there are painful contractions, the cervix initially effaces (becomes shorter and
softer) then dilates to 4cm.
Established phase:
contractions with dilatation from 4 cm. A satisfactory rate of dilatation from 4
cm is 0.5cm/h.

The 1st stage generally takes 8–18h in a primip, and 5–12h in a multip.

During the first stage check maternal BP, and T° 4-hourly, pulse hourly;
assess the contractions every 30min, their strength and their frequency (ideally
3–4 per 10min, lasting up to 1 min).
Offer vaginal examination e.g every 4h to assess the degree of cervical
dilatation, the position and the station of the head.
Auscultate fetal heart rate (if not continuously monitored), by Pinard or
Doppler every 15min, listening for 1min after a contraction.


Reference
Dr Khalid/Rabia

Q:1269 . A woman comes to the ED complaining of pain in the right side of the
abdomen, she has 7wks amenorrhea. Her pregnancy test is +ve and US scan

187
OBS GYN-System Wise 1700-by Sush and Team. 2016
Susmita, Asad, Manu, Saima, Zohaib, Savia, Shanu, Mona, Manisha, Sitara, Samreena, Sami and Komal

shows an empty uterus. What is the next step?
a. Laparoscopy
b. HCG measurements
c. US
d. Laparotomy
e. Culdo-centesis

Clincher(s) 7 WEEKS AMENORRHEA , pain , U/S shows empty uterus
A
B SERIAL HCG FOR ECTOPIC
C
D
E
KEY Answer is B.
This is a case of ectopic pregnancy.
Always think of an ectopic in a sexually active woman with abdominal pain;
bleeding; fainting; or diarrhoea and vomiting. There is generally ~8 weeks’
amenorrhoea but an ectopic may present before a period is missed. An early
sign is often dark blood loss (‘prune juice’, as the decidua is lost from the
uterus) or fresh.
Diagnosis: Early diagnosis is vital. Dipstick testing for HCG (human chorionic
gonadotrophin)
is sensitive to values of 25IU/L. do ultrasound. If HCG >6000IU/L and an
intrauterine gestational sac is not seen, ectopic pregnancy is very likely, as is
the case if HCG 1000–1500IU/L and no sac is seen on transvaginal ultrasound.
Additional
Information
Reference
Dr Khalid/Rabia


Q:1270 A 23yo woman who has had several recent partners has experienced post-
coital bleeding on gentle contact. What is the single most likely cause of her
vaginal discharge?
a. Cervical ca
b. Cervical ectropion
c. CIN
d. Chlamydial cervicitis
e. Gonococcal cervicitis

Clincher(s) MULTIPLE PARTNERS , 23yrs, postcoital bleeding , discharge
A • Cervical cancer - usually apparent on speculum examination
B
C
D 50% cause of cervicitis.
E

188
OBS GYN-System Wise 1700-by Sush and Team. 2016
Susmita, Asad, Manu, Saima, Zohaib, Savia, Shanu, Mona, Manisha, Sitara, Samreena, Sami and Komal

KEY D-chlamydia cervicitis
Additional
Information Causes of postcoital bleeding
• Infection.
• Cervical ectropion - especially in those women taking the
combined oral contraceptive pill (COCP).
• Cervical or endometrial polyps.
• Vaginal cancer.
• Cervical cancer - usually apparent on speculum examination.
• Trauma.

In this case the history of several recent partners points towards a sexually
transmitted disease so it is chlamydial cervicitis as chlamydia is transmitted
sexually.

Chlamydial cervicitis:

Risk factors
• Age <25 (the highest prevalence in women occurs between ages
16-19 years and in men between ages 20-24 years).
• Two or more sexual partners in the preceding year.
• A recent change in sexual partner.
• Non-barrier contraception.
• Infection with another STI.
• Poor socio-economic status.
• Genetic predisposition

Symptoms: Vaginal discharge.

• Dysuria (always consider chlamydia as a cause of sterile pyuria).


• Vague lower abdominal pain.
• Fever.
• Intermenstrual or postcoital bleeding.
• Dyspareunia.

Signs:

• A friable, inflamed cervix, sometimes with a follicular or


'cobblestone' appearance, with contact bleeding.
• Mucopurulent endocervical discharge.
• Abdominal tenderness.
• Pelvic adnexal tenderness on bimanual palpation.
• Cervical excitation.

189
OBS GYN-System Wise 1700-by Sush and Team. 2016
Susmita, Asad, Manu, Saima, Zohaib, Savia, Shanu, Mona, Manisha, Sitara, Samreena, Sami and Komal

Treatment: Doxycycline for 7 days or azithromycin single dose.


Reference Table on 273 Oxford clinical speciality- IMP
Dr Khalid/Rabia Follicular or mucorulent with discharge


Q:1271 A 68yo woman presents with post-coital bleeding following her first episode of
sexual intercourse in 10yrs. What is the single most likely cause that has led to
post-coital bleeding?
a. Endometrial ca
b. Atrophic vaginitis
c. Endometrial polyp
d. Cervical ca
e. Cervical ectropion


Clincher(s) 68 yrs , post coital bleed
A In a case of post menopausal bleeding always first rule out endometrial
Carcinoma.
B
C
D
E
KEY Answer is Atrophic vaginitis.
Here since most likely cause is asked it is atrophic vaginitis.

Aetiology
Vaginal atrophy. The most common cause of PMB.

Use of HRT.

Endometrial hyperplasia; simple, complex, and atypical.

Endometrial cancer. The probability of a woman presenting with

PMB having endometrial cancer is 10%. However, 90% of women
with endometrial cancer present with PMB.[2]
• Endometrial polyps or cervical polyps.
• Cervical cancer; remember to check if the cervical smear is up-to-
date.
• Uterine sarcoma (rare).
• Ovarian cancer, especially oestrogen-secreting (theca cell) ovarian
tumours.
• Vaginal cancer (very uncommon).
• Vulval cancer may bleed, but the lesion should be obvious.
Non-gynaecological causes including trauma or a bleeding disorder
Additional

190
OBS GYN-System Wise 1700-by Sush and Team. 2016
Susmita, Asad, Manu, Saima, Zohaib, Savia, Shanu, Mona, Manisha, Sitara, Samreena, Sami and Komal

Information
Reference
Dr Khalid/Rabia


Q:1272 A 28yo woman 8wks GA had PID treated prvly and now comes with vaginal
bleeding, rigid abdomen, BP=80/50 mmHg, pulse=140 bpm. What is the most
probable dx?
a. Threatened abortion
b. Miscarriage
c. Missed abortion
d. Tubal pregnancy
e. Inevitable abortion

Clincher(s) Hypotension , rigid abdomen – peritonitis ---- rupture of tubal preg
A
B Miscarriage is defined as the loss of a pregnancy before 24 weeks of gestation
C
D PID – salpingitis leads to tubal preg – ectopic
E
KEY D- tubal pregnancy
Additional MISCARRIAGE
Information Miscarriage is defined as the loss of a pregnancy before 24 weeks of gestation.
Classification of miscarriage is as follows:
• Threatened miscarriage: mild symptoms of bleeding. Usually little
or no pain. The cervical os is closed.
• Inevitable miscarriage: usually presents with heavy bleeding with
clots and pain. The cervical os is open. The pregnancy will not
continue and will proceed to incomplete or complete miscarriage.
• Incomplete miscarriage: this occurs when the products of
conception are partially expelled. Many incomplete miscarriages
can be unrecognised missed miscarriages.
• Missed miscarriage: the fetus is dead but retained. The uterus is
small for dates. A pregnancy test can remain positive for several
days. It presents with a history of threatened miscarriage and
persistent, dirty brown discharge. Early pregnancy symptoms may
have decreased or gone.
• Habitual or recurrent miscarriage: three or more consecutive
miscarriages.


Reference
Dr Khalid/Rabia

191
OBS GYN-System Wise 1700-by Sush and Team. 2016
Susmita, Asad, Manu, Saima, Zohaib, Savia, Shanu, Mona, Manisha, Sitara, Samreena, Sami and Komal


Q: 148 A 41yo woman who has completed her family, has suffered from extremely
heavy periods for many years. No medical tx has worked. She admits that she
would rather avoid open surgery.
After discussion, you collectively decide on a procedure that wouldn’t require
open surgery or
GA. Select the most appropriate management for this case.
a. Endometrial ablation (vaginal)
b. Hysterectomy
c. Fibroid resection (vaginal)
d. Myomectomy (both)
e. Uterine artery embolization (through femoral artery- Local?)



Clincher(s)
A
B
C
D
E it is done with a catheter under local anesthesia. because blood supply
blocked to uterus decrease size of myomas.

(Khalid) Uterine artery embolization is preferred when there is uterine
fibroid and in case of uterine fibroid endometrial ablation is avoided! In the
given history drug failure indicates organic disease like fibroid and that is why
UAE is preferred and endometrial ablation is not chosen.


KEY Ans. The key is uterine artery embolization.

Additional
Information
Reference
Dr Khalid/Rabia Treating menorrhagia
Drugs Progesterone-containing IUCDs, eg Mirena
should be considered 1st line treatment for those wanting contraception.
effective for bleeding and also reduce the size of fibroid uterus.

2nd line recommended drugs are antifibrinolytics, antiprostaglandins or the
Pill. Antifibrinolytics Taken during bleeding these reduce loss (by 49%)—eg
tranexamic acid CI: thromboembolic disease—
Antiprostaglandins eg mefenamic acid 500mg/8h PO pc (CI: peptic ulceration)
taken during
days of bleeding particularly help if there is also dysmenorrhoea. COCP can
also be used if they are not contraindicated.

192
OBS GYN-System Wise 1700-by Sush and Team. 2016
Susmita, Asad, Manu, Saima, Zohaib, Savia, Shanu, Mona, Manisha, Sitara, Samreena, Sami and Komal


3rd line recommendation is progestogens IM or norethisterone
Rarely gonadotrophin (LHRH) releasing hormones are used

Surgery Endometrial resection is suitable for women who have completed
their families and who have <10wk size uterus and fibroids <3cm.
Contraception will be required. For women wishing to retain fertility who have
fibroids >3cm consider uterine
artery embolization or myomectomy
Women not wishing to retain fertility, with a uterus >10wk size and fibroids
>3cm may benefit from hysterectomy, vaginal hysterectomy being the
preferred route.



Q: 159 A 25yo had an LSCS 24h ago for fetal distress. She now complains of
intermittent vaginal
bleeding. Observations: O2 sat=98% in air, BP=124/82mmHg, pulse=84bpm,
temp=37.8C. The
midwife tells you that she had a retained placenta, which required manual
removal in the OT.
Choose the most appropriate C-Section complication in this case?
a. Retained POC
b. Aspiration pneumonitis
c. Endometritis
d. Uterine rupture
e. DIC



Clincher(s)
A
B
C More handling of tissue like manual removal of placenta, intermittent vaginal
bleeding and raised temperature points toward infective process like
endometritis. Secondary hemorrhage. Most common cause endometritis

D
E
KEY The key is C. Endometritis.
Additional
Information
Reference
Dr Khalid/Rabia This is secondary PPH.
Secondary PPH: This is excessive blood loss from the genital tract after 24h
from delivery. It usually occurs between 5 and 12 days and is due to infections

193
OBS GYN-System Wise 1700-by Sush and Team. 2016
Susmita, Asad, Manu, Saima, Zohaib, Savia, Shanu, Mona, Manisha, Sitara, Samreena, Sami and Komal

(most common cause) (endometritis) or retained placenta.
Look for history of extended labour, difficult third stage, ragged placenta,
PPH.
Symptoms: Abdominal pain. Offensive smelling lochia. Abnormal vaginal
bleeding - PPH. Abnormal vaginal discharge. Dyspareunia. Dysuria.
Signs: are those of sepsis. Tachycardia, fever, rigors, suprapubic tenderness.
Treatment: For endometritis: IV antibiotics if there are signs of severe sepsis.
If less systemically unwell, oral treatment may be sufficient. Piperacilin and
tazobectum may be used.
If RPOC are suspected, elective curettage with antibiotic cover may be
required. Surgical measures should be undertaken if there is excessive or
continuing bleeding, irrespective of ultrasound findings



Q: 175 A 25yo woman with T1DM has delivered a baby weighing 4.5kg. Her uterus is
well contracted. Choose the single most likely predisposing factor for PPH from
the options?
a. Atonic uterus
b. Cervical/vaginal trauma
c. Retained POC (product of conception)
d. Large placental site
e. Rupture uterus



Clincher(s)
A
B The baby is a big baby. If patient’s uterus was not well contracted we would
fear of atonic uterus! But as uterus is well contracted it is not atonic uterus.
Rather most likely cause is trauma dring delivery of this big baby.
C
D
E
KEY The key is B. Cervical/vaginal trauma
Additional
Information
Reference
Dr Khalid/Rabia Primary PPH is the loss of greater than 500mL (definitions vary) in the first 24h
after delivery
Causes: uterine atony (90%), genital tract trauma (7%), clotting disorders—
(3%)
Risks: Antenatal • Previous PPH or retained placenta BMI>35kg/m2 •
Maternal Hb<8.5g/dl at onset of labour • Antepartum haemorrhage
Multiparity 4+ • Maternal age 35y+ • Uterine malformation or fibroids A large
placental site (twins, severe rhesus disease, large baby) • Low placenta,

194
OBS GYN-System Wise 1700-by Sush and Team. 2016
Susmita, Asad, Manu, Saima, Zohaib, Savia, Shanu, Mona, Manisha, Sitara, Samreena, Sami and Komal

Overdistended uterus (polyhydramnios, twins) • Extravasated blood in the
myometrium (abruption).
In labour • Prolonged labour (1st, 2nd or 3rd stage) • Induction or oxytocin use
• Precipitant labour • Operative birth or caesarean section. Book mothers with
risk factors for obstetric unit delivery.
Treatment: Give oxytocin 5U slowly IV for atonic uterus.
Attach oxygen, Give IV fluids, maintain systolic >100mmHg, Transfuse blood.
Is the placenta delivered? If it is, is it complete? If not, explore the uterus. • If
the placenta is complete, put the patient in the lithotomy position with
adequate analgesia and good lighting. Check for and repair trauma.
• If the placenta has not been delivered but has separated, attempt to deliver
it by controlled cord traction after rubbing up a uterine contraction. If this fails,
ask an experienced obstetrician to remove it under general
anaesthesia.Beware renal shut down.



Q: 181 A 28yo woman at 39wk gestation is in labor. She develops abdominal pain and
HR=125bpm, BP=100/42mmHg, temp=37.2C and saturation=99%. Exam: lower
abdomen is exquisitely tender. CTG=prv normal, now showing reduced
variability and late deceleration develops with slow recovery. She has had 1
prev LSCS for a breech baby. Choose the most appropriate CS
complication for this lady?
a. Endometritis
b. UTI
c. Urinary tract injury
d. Pleurisy
e. Uterine rupture



Clincher(s) tachycardia and going hypotensive, painful abdomen, fetal
compromise and h/o Csection,

A Endometritis– no fever

B little fever but no dysuria, no bacteriuria or anything so not an option.

C No urological symtom like urinary retention

D Sat is 99%
E
KEY The key is E. Uterine rupture.
Additional
Information
Reference

195
OBS GYN-System Wise 1700-by Sush and Team. 2016
Susmita, Asad, Manu, Saima, Zohaib, Savia, Shanu, Mona, Manisha, Sitara, Samreena, Sami and Komal

Dr Khalid/Rabia Uterine rupture: Its an obstetrical emergency
Causes: ~70% of UK ruptures are due to dehiscence of caesarean section scars.
Other risk factors: • Obstructed labour in the multiparous, especially if
oxytocin is used • Previous
cervical surgery • High forceps delivery • Internal version • Breech extraction.
Rupture is usually during the third trimester or in labour.

Vaginal birth after caesarean (trial of scar): Vaginal birth will be successful in
72–76%. Endometritis, need for blood transfusion, uterine rupture and
perinatal death are commoner than repeated elective C section.

Signs and symptoms Rupture is usually in labour. In a few (usually a caesarean
scar dehiscence) rupture precedes labour. Pain is variable, some only having
slight pain and tenderness over the uterus. In others pain is severe.
Vaginal bleeding is variable and may be slight (bleeding is intraperitoneal).
Unexplained
maternal tachycardia, sudden maternal shock, cessation of contractions,
disappearance of the presenting part from the pelvis, and fetal distress are
other presentations. Postpartum indicators of rupture: continuous PPH with a
well-contracted uterus; if bleeding continues postpartum after cervical repair;
and whenever shock is present.

Management If suspected in labour, perform laparotomy, deliver the baby by
caesarean section, and explore the uterus. If rupture is small Repair or if vagina
or cervix are involved in the tear hysterectomy may be needed.



Q: 190 190. A 45yo waitress complains of pelvic pain which worsens pre-menstrually
and on standing and walking. She also complains of post-coital ache. Select the
most likely cause leading to her
symptoms?
a. PID
b. Endometriosis (other –dysparunia)
c. Pelvic congestion syndrome
d. Adenomyosis
e. Premature ovarian failure
Ans.




Clincher(s)
A PID mostly presents with pelvic pain, fever, spasm of lower abdominal muscles

196
OBS GYN-System Wise 1700-by Sush and Team. 2016
Susmita, Asad, Manu, Saima, Zohaib, Savia, Shanu, Mona, Manisha, Sitara, Samreena, Sami and Komal

and cervicitis with profuse, purulent, or bloody vaginal discharge. Heavy
menstrual loss suggests endometritis.

B Endometriosis can present as given in this question but it will have menstrual
abnormalities too and the pain will not be related to any specific activity.

C
PCS as its common in people who have a standing jobs like waitress etc


D
E
KEY The key is C. Pelvic congestion syndrome.
Additional
Information
Reference
Dr Khalid/Rabia Pelvic Congestion Syndrome:

Condition is characterised by the presence of dilated pelvic veins associated
with stasis
Aetiology
Considered that ovarian dysfunction is responsible for the excessive
production of local oestrogen, causing dilatation and stasis in the pelvic veins,
which leads to pelvic pain

Presentation:
women with this condition commonly complain of a dull, aching pain,
exacerbated by activities that increase intra-abdominal pressure; the pain is
relieved by lying down.
other clinical features may also be deep dyspareunia, congestive
dysmenorrhoea and post-coital ache condition usually occurs in the
reproductive age group, with a mean age of 33 years.

Examination may reveal tenderness that is maximal over the ovaries. Vaginal
and cervical examination may reveal an apparent blue colouration due to
congestion of the pelvic veins. The patient may also have varicose veins of the
legs

Investigations for endometriosis and pelvic inflammatory disease must be
instigated
Venography is still considered the definitive radiological investigation for
women with pelvic congestion syndrome
Radiological features: dilated uterine and ovarian veins with reduced venous
clearance of contrast medium

Management

197
OBS GYN-System Wise 1700-by Sush and Team. 2016
Susmita, Asad, Manu, Saima, Zohaib, Savia, Shanu, Mona, Manisha, Sitara, Samreena, Sami and Komal

Medical treatment options include:
Medroxyprogesterone acetate (MPA) suppresses ovarian function and
therefore reduces pelvic congestion and pain, however benefit was not
sustained after discontinuing treatment (1)

Gonadorelin analogues goserelin 3.6 mg per month given for 6 months
provided an alleviation of symptoms, an improvement in sexual functioning
and a reduction of anxiety and depressive states in women with pelvic
congestion

Other possible treatment options include:
Bilateral ovarian vein ligation
Hysterectomy plus bilateral salpingo-oophrectomy (with post-operative
hormone replacement therapy)


Q: 193 A 35yo lady who has been using IUCD for one year now complains of pelvic
pain and heavy
painful periods. Select the most likely cause leading to her symptoms?
a. PID
b. Endometriosis
c. Adenomyosis
d. Fibroids
e. Asherman syndrome



Clincher(s)
A The given picture may have D/D of PID or fibroid. As IUCD is a risk factor for
PID, it is the most likely diagnosis of given picture. iucd increases the risk of pid
in the 1st 20 days but other answers excluded

B
Endometriosis will also have Cyclical or chronic pelvic pain
along with Dysmenorrhoea. ( endometriosis has a multifactorial aetiology,
involving possible genetic, immunological, and endocrinological factors. So its
not just IUCD use.)

Pain of endometriosis characteristically increase as the cycle progresses...


C
D Fibroids are not related to IUCD. Also in fibroids there will be findings on
examination such as mass in lower abdomen.

E Due to D & C- leisons- leading to secondary amenorrhea

198
OBS GYN-System Wise 1700-by Sush and Team. 2016
Susmita, Asad, Manu, Saima, Zohaib, Savia, Shanu, Mona, Manisha, Sitara, Samreena, Sami and Komal

KEY The key is A. PID.
Additional
Information
Reference

199
OBS GYN-System Wise 1700-by Sush and Team. 2016
Susmita, Asad, Manu, Saima, Zohaib, Savia, Shanu, Mona, Manisha, Sitara, Samreena, Sami and Komal

Dr Khalid/Rabia

200
OBS GYN-System Wise 1700-by Sush and Team. 2016
Susmita, Asad, Manu, Saima, Zohaib, Savia, Shanu, Mona, Manisha, Sitara, Samreena, Sami and Komal


Q: 196 . A 64yo woman has been on HRT for 9yrs. She had regular withdrawal bleeds
until 3 yrs ago and since then has been taking a no bleed prep. Recently she
noticed a brown vaginal discharge.
Choose the single most appropriate initial inv?
a. Cervical smear
b. High vaginal swab
c. TFT
d. Transvaginal US

e. Endometrial sampling




Clincher(s)
A
B
C
D
Brown vaginal discharge is endometrial Ca. Do a tvs us then asses the thickness
and then endometrial sampling .

a no bleed prep is prob due to estrogen alone HRT, which would predispose to
endometrial hyperplasia and cancer. *post menopausal bleed is always
endometrial cancer until and unless proved otherwise.

So transvaginal USS and endometrial sampling. Initial would be Tvs USS to
check endometrial thickness and then definitive/best is endometrial sampling





E
KEY The key is D. Transvaginal US.
Additional
Information
Reference
Dr Khalid/Rabia To determine the endometrial thickness!

In a postmenopausal woman with vaginal bleeding, the risk of cancer is
approximately 7.3% if her endometrium is thick (> 5 mm) and < 0.07% if her
endometrium is thin (≤ 5 mm).
In postmenopausal women without vaginal bleeding, the risk of cancer is
approximately 6.7% if the endometrium is thick (> 11 mm) and 0.002% if the

201
OBS GYN-System Wise 1700-by Sush and Team. 2016
Susmita, Asad, Manu, Saima, Zohaib, Savia, Shanu, Mona, Manisha, Sitara, Samreena, Sami and Komal

endometrium is thin (≤ 11 mm).

Investigate postmenopausal vaginal bleeding promptly as the cause may be
endometrial cancer.

Endometrial Carcinoma:
Most are adenocarcinomas, and are related to excessive exposure to
oestrogen unopposed
by progesterone.

Risk Factors: Obesity • Unopposed oestrogen • Functioning ovarian tumour •
Family History of breast, ovary, or colon cancer • Nulliparity • Late menopause
• Diabetes mellitus • Tamoxifen, tibolone • Pelvic irradiation • Polycystic
ovaries.

Presentation This is usually as postmenopausal bleeding (PMB). It is initially
scanty and occasional (± watery discharge). Then bleeding gets heavy and
frequent. Premenopausal women may have intermenstrual bleeding, but 30%
have only menorrhagia.

Diagnosis: TVUS scan is an appropriate first-line procedure to identify which
women with PMB are at higher risk of endometrial cancer. Endometrial
thickness of >5mm warrants biopsy. The definitive diagnosis is made by
uterine sampling or curettage. All parts of the uterine cavity must be sampled;
send all material for histology. Hysteroscopy enables visualization of abnormal
endometrium to improve accuracy of sampling.

Staging The tumour is…
Stage I in the body of the uterus only.
Stage II in the body and cervix only.
Stage. III advancing beyond the uterus, but not beyond the pelvis.
Stage: IV extending outside the pelvis (eg to bowel and bladder).

Treatment: Stages I and II may be cured by total hysterectomy with bilateral
salpingo-
oophorectomy and/or radiotherapy if unfit for surgery. In advanced diseases
consider radiotherapy and/or high dose progesterone which shrinks the
tumor.



Q: 268 A 20yo young lady comes to the GP for advice regarding cervical ca. she is
worried as her mother
past away because of this. She would like to know what is the best method of
contraception in
her case?

202
OBS GYN-System Wise 1700-by Sush and Team. 2016
Susmita, Asad, Manu, Saima, Zohaib, Savia, Shanu, Mona, Manisha, Sitara, Samreena, Sami and Komal

a. POP (prog only)
b. Barrier method
c. IUCD
d. COCP (The combined oral contraceptive pill )

e. IUS



Clincher(s)
A Pills increase ca risk
B Points in favour = barrier method can help prevent catching HPV infection
which is the main etiology behind CA cervix. Other methods may provide
with better contraception but are not good means of preventing hpv
infections.

C
D
E
KEY Key = B (barrier method)

Additional
Information
Reference
Dr Khalid/Rabia


Q: 271 A 44yo woman complains of heavy bleeding per vagina. Transvaginal US was
done and normal.
Which of the following would be the most appropriate inv for her?
a. Hysterectomy
b. Endometrial biopsy
c. CBC
d. High vaginal swab
e. Coagulation profile


Clincher(s)
A Before considering steps like hysterectomy, systemic causes of bleeding must
be ruled out by checking coagulation profile.
B Endometrial biopsy will be needed if ultrasound shows some endometrial
abnormality
C CBC and high vaginal swab will not help much in finding the cause of bleeding.

D
E Points in favour = After normal vaginal US coagulation profile should be done

203
OBS GYN-System Wise 1700-by Sush and Team. 2016
Susmita, Asad, Manu, Saima, Zohaib, Savia, Shanu, Mona, Manisha, Sitara, Samreena, Sami and Komal

to rule out systemic causes of heavy bleeding first..

KEY Key = E (coagulation profile)

Additional FB: can be Von willebrand factor deficiency which is an inherited autosomal
Information dominant coagulopathy that can present in later stages with epistaxis, purpura
or menorrhagia as in this case.

Debate: supported by abu khattab:



E would have been done if we had known Platelet count and they were
normal.


Reference
Dr Khalid/Rabia


Q:

Clincher(s)
A
B
C
D
E
KEY
Additional
Information
Reference
Dr Khalid/Rabia


204
OBS GYN-System Wise 1700-by Sush and Team. 2016
Susmita, Asad, Manu, Saima, Zohaib, Savia, Shanu, Mona, Manisha, Sitara, Samreena, Sami and Komal

Q: 1390 A 7yo boy presents with proptosis and periorbital edema. What is the
immediate action that needs to be taken?
a. IV morphine and immediate ophthalmoscopy
b. IV morphine
c. Observation only


Clincher(s) proptosis and periorbital edema
A Periorbital cellulitis
B
C
D
E
KEY A- IV morphine and immediately ophthalmoscopy.
Additional
Information


Reference OHCS PG 420
Dr Khalid/Rabia


205
OBS GYN-System Wise 1700-by Sush and Team. 2016
Susmita, Asad, Manu, Saima, Zohaib, Savia, Shanu, Mona, Manisha, Sitara, Samreena, Sami and Komal

Q: 1409 A 19yo female dx with trichomonas vaginalis. LMP was 10d ago. What is the
best antibiotic tx? a. Erythromycin

b. Vancomycin
c. Metronidazole
d. Penicillin
e. Clarithromycin
f. Doxycycline
g. Fluconazole
h. Clotrimazole


Clincher(s) Trichomonas vaginalis infection- treatment.
A
B
C
D
E
KEY C- metronidazole

206
OBS GYN-System Wise 1700-by Sush and Team. 2016
Susmita, Asad, Manu, Saima, Zohaib, Savia, Shanu, Mona, Manisha, Sitara, Samreena, Sami and Komal

Additional
Information


Reference OHCS PG 284
Dr Khalid/Rabia


207
OBS GYN-System Wise 1700-by Sush and Team. 2016
Susmita, Asad, Manu, Saima, Zohaib, Savia, Shanu, Mona, Manisha, Sitara, Samreena, Sami and Komal

Q: 1411 A 28yo woman comes with sudden onset vomiting and pain per abdomen.
Exam: mobile swelling in the right iliac fossa. What is the most probable dx?
a. Ectopic pregnancy
b. Tubo-ovarian abscess
c. Acute appendicitis
d. Ovarian torsion
e. Diverticulitis


Clincher(s) sudden onset vomiting and pain abdomen.
A There is no history amenorrhea.
B There is no history of fever or septic signs.
C Acute appendicits is not mobile and there is a rigid abdomen.
D only suitable option
E This occurs on the left side of abdomen.
KEY D- ovarian torsion.
Additional
Information


Reference OHG&O PG 96

208
OBS GYN-System Wise 1700-by Sush and Team. 2016
Susmita, Asad, Manu, Saima, Zohaib, Savia, Shanu, Mona, Manisha, Sitara, Samreena, Sami and Komal

Dr Khalid/Rabia



Q: 1419 A 32yo woman with prv hx of PID now presents with severe abdominal pain.
Her LMP was 8wks ago. What is the most probable dx?
a. Ectopic pregnancy
b. Ovarian torsion
c. Hematometrium
d. Chronic PID
e. Cholecystitis


Clincher(s) H/o PID and ammenorrhea-8weeks.
A Patients with PID are on increased risk of developing ectopic pregnancy. (due
to adhesion of fallopian tubes). 90% are tubal .

Clinchers: irreg bleeding, abd pain, amenorrhea: ectopi.

Anti D prophylaxis given as not sure about grouping of fetus: chance of feto
maternal mixing, if needed.


B
C
D Dull pain, discharge
E
KEY A- ectopic pregnancy.

209
OBS GYN-System Wise 1700-by Sush and Team. 2016
Susmita, Asad, Manu, Saima, Zohaib, Savia, Shanu, Mona, Manisha, Sitara, Samreena, Sami and Komal

Additional
Information

210
OBS GYN-System Wise 1700-by Sush and Team. 2016
Susmita, Asad, Manu, Saima, Zohaib, Savia, Shanu, Mona, Manisha, Sitara, Samreena, Sami and Komal

Reference OHCS PG 262
Dr Khalid/Rabia



Q: 1420 A 25yo who is 38wks pregnant presents to the labour ward with a hx of fewer
fetal movements than usual during the evening. She also says that abdominal
contractions are coming veery few minutes and she is having a blood stained
show per vagina for the last few minutes. Exam: cervix
is fully affaced, 9cm dilated, cephalic presentation and station is +1. Choose the
single most
likely dx?
a. APH
b. Concealed haemorrhage (placental previa-
c. Labour

d. IUFD (demise)
e. IUGR


Clincher(s) blood stained show, cervix dilation 9cm, 38 weeks.
A no sign of blood loss.
B irrelevant
C best option
D Irrelevant
E irrelevant
KEY C- labour

211
OBS GYN-System Wise 1700-by Sush and Team. 2016
Susmita, Asad, Manu, Saima, Zohaib, Savia, Shanu, Mona, Manisha, Sitara, Samreena, Sami and Komal

Additional
Information


Reference OHCS PG 58

212
OBS GYN-System Wise 1700-by Sush and Team. 2016
Susmita, Asad, Manu, Saima, Zohaib, Savia, Shanu, Mona, Manisha, Sitara, Samreena, Sami and Komal

Dr Khalid/Rabia


Q: 1526 A mother brings her newborn to the hosp concerned about a blue patch on the
buttocks. The newborn is of mixed race and was delivered normally. What is
the most appropriate
management?
a. Reassurance
b. CBC
c. XR
d. Plt count


Clincher(s) blue patch with normal delivery and normal growth.
A Reassurance as this is a birth mark.
B
C
D
E
KEY A- reassurance
Additional Mongolian spot, also known as Mongolian blue spot, congenital dermal
Information melanocytosis. It is a benign, flat, congenital birthmark with wavy borders and
irregular shape. It normally disappears three to five years after birth and almost
always by puberty. The most common color is blue, although they can be blue-
gray, blue-black or even deep brown.

Reference
Dr Khalid/Rabia

213
OBS GYN-System Wise 1700-by Sush and Team. 2016
Susmita, Asad, Manu, Saima, Zohaib, Savia, Shanu, Mona, Manisha, Sitara, Samreena, Sami and Komal

Q: 1537 An obese woman with hx of migraine presented with heavy bleeding during
menstruation which is painful and needs contraception too. What is the best
possible management for this pt?
a. COCP
b. Mirena coil
c. Copper T
d. UAE
e. Depo provera


Clincher(s)
A contracindicated in migraine. (oestrogen increase migraine)
B best treatment for dysfunctional uterine bleeding.
C No role.
D 2nd line treatment after failure of medical management.
E not advised.
KEY B- mirena coil

214
OBS GYN-System Wise 1700-by Sush and Team. 2016
Susmita, Asad, Manu, Saima, Zohaib, Savia, Shanu, Mona, Manisha, Sitara, Samreena, Sami and Komal

Additional
Information


Reference OHC O & G. PG 513
Dr Khalid/Rabia


215
OBS GYN-System Wise 1700-by Sush and Team. 2016
Susmita, Asad, Manu, Saima, Zohaib, Savia, Shanu, Mona, Manisha, Sitara, Samreena, Sami and Komal

Q: 1556 A 43yo woman has suffered with heavy periods for many years and has tried
many medical tx without succ ess. She is con stan tl y floodi ng an d at
tim es ca n’ t leav e her house due to hea vy bleeding. She has
completed her family of 5 children and her last blood test showed Hgb=8.9g/dl.
She feels that she can’ t cope w ith the b leedi g a nymor e a d her
husb an d is askin g for a tx that can guarantee success. What is the most
appropriate management to improve menorrhagia in this pt?
a. Endometrial ablation
b. Hysterectomy
c. Hysteroscopic/Laser resection of fibroids
d. Myomectomy
e. UAE


Clincher(s) heavy bleeding and completed family.
A Other ones for those to not completed family
B Above 40, unsuccessful medical tx, - hysterectomy
C
D
E
KEY B hysterectomy.
Additional
Information
Reference
Dr Khalid/Rabia


216
OBS GYN-System Wise 1700-by Sush and Team. 2016
Susmita, Asad, Manu, Saima, Zohaib, Savia, Shanu, Mona, Manisha, Sitara, Samreena, Sami and Komal

Q: 1562 A 48yo nulliparous woman feels tired all the time. Her periods are regular but
have always lasted for at least 10d. Choose the single most appropriate intial
inv?
a. High vaginal swab
b. Serum Hgb conc
c. TFT
d. None
e. Abdominal US


Clincher(s) showing signs(tiredness and lethargy) of anaemia
A
B
C
D
E
KEY B serum Hb level
Additional
Information
Reference
Dr Khalid/Rabia



Q:

Clincher(s)
A
B
C
D
E

217
OBS GYN-System Wise 1700-by Sush and Team. 2016
Susmita, Asad, Manu, Saima, Zohaib, Savia, Shanu, Mona, Manisha, Sitara, Samreena, Sami and Komal

KEY
Additional
Information
Reference
Dr Khalid/Rabia






218

You might also like